Download as pdf or txt
Download as pdf or txt
You are on page 1of 79

Contents of IBPS RRB PO & Clerk Prelims 2020 | Capsule | 100% Selection Plan

Guidelines by IBPS to conduct the Exam .......................................................................................................................................... 2


IBPS RRB PO & Clerk Prelims Mini Mock Test Schedule ............................................................................................................. 5
IBPS RRB PO & Clerk Prelims Detailed Syllabus 2020 ................................................................................................................ 7
Must Watch Videos Sessions for IBPS RRB Prelims for Quick Revision ................................................................................ 8
IBPS RRB PO Prelims 2019| Memory Based Paper | For Practice ........................................................................................... 9
IBPS RRB PO Prelims 2018| Memory Based Paper | For Practice .........................................................................................19
IBPS RRB PO Prelims 2017| Memory Based Paper | For Practice .........................................................................................30
IBPS RRB Clerk Prelims 2019 | Memory Based Paper | For Practice ...................................................................................40
IBPS RRB Clerk Prelims 2018 | Memory Based Paper | For Practice ...................................................................................48
IBPS RRB Clerk Prelims 2017 | Memory Based Paper | For Practice ...................................................................................56
Most Asked Topics in IBPS RRB PO Prelims Exam 2020 - Based on Previous Year’s Exam Analysis .......................64
Most Asked Topics in IBPS RRB Clerk 2020 Exam- Based On Previous Year’s Paper ....................................................66
What Should Be Expected Good Attempts for IBPS RRB PO And Clerk Prelims Exams? ...............................................68
Most Important Questions | Quantitative Aptitude | IBPS RRB Prelims 2020 ..................................................................71
Most Important Questions | Reasoning Ability | IBPS RRB Prelims 2020 ..........................................................................76

1 Adda247 | No. 1 APP for Banking & SSC Preparation


Website: bankersadda.com | sscadda.com | store.adda247.com | Email: contact@bankersadda.com
Guidelines by IBPS to conduct the Exam
IBPS RRB Exam Day Guidelines 2020
The IBPS RRB PO & Clerk Prelims exam will begin from 12th September and this year there are 10,000+
vacancies for the IBPS RRB 2020 Recruitment. If you are going to appear for the exam then you must go
through the exam conduct guidelines given below, and if you do not follow the social distancing mode of
conduct guidelines you can also be denied entry to the centre.

IBPS Guidelines: Social Distancing Mode of conduct of Exam Related Instructions


Candidates are required to report to their exam centre 15 minutes before the reporting time indicated on
their call letters. Candidates will be permitted to take along the following items with themselves in the
exam centre:
1. Mask (WEARING A MASK is COMPULSORY)
2. Gloves
3. Personal transparent water bottle
4. Personal hand sanitizer (50 ml)
5. A simple pen
6. Exam related documents (Call Letter/Admit Card, ID card in Original, Photocopy of ID Card, etc)
7. Call Letter/Admit Card should be brought with the Photocopy of the Photo ID stapled with it. Original
ID (same as Photocopy) is also to be brought for verification. The name on the ID and on the Call Letter/
Admit Card should be exactly the same.
8. In case of Scribe Candidates – Scribe form duly filled and signed with Photograph affixed.
No other item apart from those mentioned above will be permitted in the exam venue as per the IBPS RRB
2020 Recruitment notification. And IBPS has clearly mentioned that wearing a mask is compulsory to
attend the examination.

Along with this, there are also certain other measures mentioned in the guidelines:
1. Candidate should not share any of their personal belonging/material with anyone
2. Candidate should maintain safe social distance with one another.
3. Candidate should stand in the row as per the instructions provided at venue.
4. If candidate is availing the scribe, then scribe also should bring their own Gloves, N95 Mask, sanitizer
(50ml) and water bottle. Wearing a mask is compulsory. Both candidate and Scribe will require to be
wearing N95 Mask.
5. A Candidate must have Aarogya Setu App installed on his mobile phone. The Aarogya Setu status
must show candidate’s risk factor. A candidate will have to display this status to the Security Guard at
the entry into the exam venue. In case a candidate does not have a smart phone, he/she will have
to bring in a signed declaration to this effect (declaration form provided in Annexure- IV) and
show the same to the Security Guard at the entry into the exam venue.
6. Candidates with Moderate or High Risk Status on Aarogya Setu will not be allowed entry. In case
any of the responses in declaration suggest COVID 19 infection/symptoms, the candidate will not be
permitted inside the exam venue.

2 Adda247 | No. 1 APP for Banking & SSC Preparation


Website: bankersadda.com | sscadda.com | store.adda247.com | Email: contact@bankersadda.com
7. After Aarogya Setu status display at the entry gate, candidates will be required to switch off their mobile
phones, and deposit it at the designated location, to be collected while exiting
8. All candidates will be checked with Thermo guns at the entry point for temperature. In case, any
person is observed to be having above normal temperature (> 99.14° F) or displaying any symptoms of
the virus, they will not be allowed entry into the venue

What are the conditions in which you can be declined entry to the exam centre and will not
be allowed to appear in the exam?
As per the instructions given by Institute of Banking Personnel Selection, a candidate will not be allowed
to appear in the IBPS RRB 2020 Exam under the following circumstances:
1. If the candidate is not wearing a mask, as wearing a mask is compulsory to attend the examination by
both the candidate and scribe (in case if you will be using a scribe).
2. If candidate fails to show his or her risk factor on the Aarogya Setu App. In case you don’t have a mobile
phone, you must have to bring in a signed declaration to this effect (declaration form provided in
Annexure- IV) and show the same to the Security Guard.
3. If candidate has Moderate or High Risk Status on Aarogya Setu App.
4. If the candidate’s body temperature is recorded above normal (> 99.14° F) at the temperature check
point at the entry gate.
5. If the candidate is displaying symptoms of COVID-19

Frequently Asked Questions by Students on COVID-19 Guidelines for IBPS Exams


Many students have been asking questions related to the guidelines on Adda247 App. Here we are
providing you the answers to some frequently asked questions related to what you should bring to the
exam centre and other COVID-19 guidelines.

Q. Which N95 mask to use?


Ans. Do not wear masks with valve as recently the government has said that the valve can cause risk of
transmitting and catching infections.

Q. If a candidate has a fever, will he or she be allowed to take the test?


Ans. As fever is amongst the primary symptom of COVID-19, IBPS has mentioned in the notice that the
temperatures of all the candidates will be checked and if a candidate has a fever he or she can be
denied entry in the examination Centre.

Q. I don’t have a mobile, or do not have means to install Arogya Setu App, what should I do?
Ans. Then you must bring a signed declaration as mentioned above in the article.

Q. If a candidate has flu like symptoms will he or she be allowed to take the test?
Ans. No, Candidates with flu like symptoms can be denied entry in order to keep others safe.

Q. Do I have to bring sanitizer to the exam centre?


Ans. Yes all candidates must bring sanitizers with them.

3 Adda247 | No. 1 APP for Banking & SSC Preparation


Website: bankersadda.com | sscadda.com | store.adda247.com | Email: contact@bankersadda.com
Q. What should I bring to the exam centre?
Ans. All the details of the items you must bring to the exam centre will be mentioned in your admit card
you must also wear a mask and bring sanitizer and you can also bring a clear water bottle for yourself.

Self-Declaration Form for IBPS RRB 2020 Exam


Under the Annexure IV of the IBPS RRB Notification, Institute of Banking Personnel Selection has given a
Self-Declaration that a candidate must fill and bring to the exam centre in case they do not have a
smartphone with Aarogya Setu App.

4 Adda247 | No. 1 APP for Banking & SSC Preparation


Website: bankersadda.com | sscadda.com | store.adda247.com | Email: contact@bankersadda.com
IBPS RRB PO & Clerk Prelims Mini Mock Test Schedule
Here is the Mini Mock Test Schedule for IBPS RRB PO and Clerk Prelims exam preparation and you will
get the mini mock tests and practice questions daily on bankersdadda.com and Adda247 App. You
can also make a schedule according to your strengths and weakness to score maximum in the upcoming
exam and speed up your preparation for IBPS RRB Prelims Exams.

S.
Date Quantitative Aptitude Reasoning Ability
No.
1 31 August Arithmetic Word Problems Puzzles and Seating Arrangement
2 1 September Data Interpretation Coding-Decoding
3 2 September Number Series Inequalities and Syllogism
Puzzle, Direction Sense and Alphanumeric
4 3 September Simplification
Series
Seating Arrangement, Blood Relation,
5 4 September Approximation
Alphabet/Number based Questions
6 5 September Quadratic Inequalities Puzzle, Syllogism, Order and Ranking
Caselet, Quantity Based and Data Seating Arrangement, Inequalities, Coding-
7 6 September
Sufficiency Decoding
8 7 September Arithmetic Word Problems Puzzles, Direction Sense, Miscellaneous
Seating Arrangement, Alphanumeric Series
9 8 September Number Series, Simplification
and Syllogism
10 9 September Data Interpretation Puzzle, Blood Relation, Order and Ranking
Quadratic Inequalities, Seating Arrangement, Inequalities and
11 10 September
Approximation Coding-Decoding
12 11 September Full Length Test Full Length Test
13 12 September Memory Based Paper Test
Practice Set- 1 (based on questions Practice Set- 1 (based on questions asked
14 13 September
asked on the Day 1 of Exam) on the Day 1 of Exam)
Practice Set- 2 (based on questions Practice Set- 2 (based on questions asked
15 14 September
asked on the Day 2 of Exam) on the Day 2 of Exam)
16 15 September Data Interpretation, Caselet Puzzle, Coding-Decoding, Syllogism

5 Adda247 | No. 1 APP for Banking & SSC Preparation


Website: bankersadda.com | sscadda.com | store.adda247.com | Email: contact@bankersadda.com
S.
Date Quantitative Aptitude Reasoning Ability
No.
Seating Arrangement, Alphanumeric Series,
17 16 September Arithmetic Word Problems
Inequalities
Puzzle, Direction Sense, Alphabet/Number
18 17 September Number Series, Approximation
based Questions
Seating Arrangement, Blood Relation,
19 18 September Simplification
Miscellaneous
Puzzle, Coding-Decoding, Order and
20 19 September Quadratic Inequalities
Ranking
Practice Set- 3 (based on questions Practice Set- 3 (based on questions asked
21 20 September
asked on the Day 3 of Exam) on the Day 3 of Exam)
Practice Set- 4 (based on questions Practice Set- 4 (based on questions asked
22 21 September
asked on the Day 4 of Exam) on the Day 4 of Exam)
23 22 September Practice Set - 5 Practice Set - 5
24 23 September Practice Set - 6 Practice Set - 6
25 24 September Practice Set - 7 Practice Set - 7
Expected Questions for the Next
26 25 September Expected Questions for the Next Shift
Shift

6 Adda247 | No. 1 APP for Banking & SSC Preparation


Website: bankersadda.com | sscadda.com | store.adda247.com | Email: contact@bankersadda.com
IBPS RRB PO & Clerk Prelims Detailed Syllabus 2020
IBPS RRB Prelims Exam Syllabus 2020
One of the most important steps in the recruitment process is to know the syllabus of that exam. IBPS RRB
2020 Exam has only two sections in prelims namely: Quantitative Ability & Reasoning ability and
candidates also have the option to choose their regional language as the language medium for the test.
The following table has the complete syllabus of topics that you must practice for IBPS RRB PO and IBPS
RRB Clerk Preliminary Exams 2020.

Reasoning Ability Prelims Syllabus


 Puzzles
 Seating Arrangement
 Alphabet/Number/Symbol Series
 Inequalities
 Syllogism
 Blood-Relation
 Direction-sense
 Coding-Decoding
 Order-Ranking and Odd-one-Out
 Alphabet/ Number Based Questions

Quantitative Aptitude Prelims Syllabus


 Number Series – Missing Series and Wrong Series
 Simplification
 Approximation
 Quadratic Inequalities
 Data Interpretation – Table, Bar, Pie and Line
 Number System
 Simple Interest & Compound Interest
 Time & Work
 Pipes & Cisterns
 Speed Time Distance
 Boat & Stream
 Trains
 Profit & Loss
 Mensuration
 Probability
 Permutation & Combination
 Average & Ages
 Percentage
 Ratio & Proportion
 Mixture & Allegation
 Partnership

7 Adda247 | No. 1 APP for Banking & SSC Preparation


Website: bankersadda.com | sscadda.com | store.adda247.com | Email: contact@bankersadda.com
Must Watch Videos Sessions for IBPS RRB Prelims for
Quick Revision
For the revision Purpose, we are sharing the Complete Playlist for IBPS RRB Prelims. This playlist includes
all the Topics for all the Sections i.e Quantitative Aptitude and Reasoning Ability. These topics are taken by
the top educators from Adda247. So, do subscribe the channel as well so that you can attend the Live
Classes of our team for IBPS RRB PO and Clerk Prelims as well.

S.no Name of the Section Link of the Playlist


1. Reasoning Ability Click Here
2. Quantitative Aptitude Click Here

Follow the Complete Live Sessions Schedule by the Top Faculties, Subscribe
Adda247 You tube Channel Now.

8 Adda247 | No. 1 APP for Banking & SSC Preparation


Website: bankersadda.com | sscadda.com | store.adda247.com | Email: contact@bankersadda.com
IBPS RRB PO Prelims 2019| Memory Based Paper |
For Practice
REASONING ABILITY
Direction (1-5): Study the following information 6. Statements: Only a few Palace is Home.
carefully and answer the questions given below: All Home is Office.
No Office is Building.
Nine persons i.e. P, Q, R, S, T, U, V, W, X were born on Conclusion I. All Palace is Home is a possibility.
different months i.e. January, March, April, May, July, II. Some Palace is Building.
August, September, October, November but not
7. Statements: All Men is Women. Some Child is
necessarily in same order. Women. No Men is Boy.
Four persons were born between P and T. P was born Conclusion I. Some Men is Child.
before T. Q was born in the month of 30 days after July. T II. No Men is Child.
was born after Q and before R. There were as many 8. Statements: No Professor is Student. Only a few
persons born before X as after R. one person was born Student is Lecturer. All Lecturer is
between U and V. S was born before U and after W. Principal.
Conclusion I. All Professor is Principal is a
1. How many persons were born between X and V? possibility.
(a) Two (b) Three (c) One II. All Student is Lecturer is a
(d) Four (e) More than four possibility.
2. Who among the following was born on August? 9. Statements: Only a few Palace is Home.
(a) R (b) S (c) T All Home is Office.
(d) P (e) None of these No Office is Building.
Conclusion I. Some Home is Building.
3. In which of the following month S was born? II. No Home is Building.
(a) March (b) April (c) June
10. Statements: No Professor is Student.
(d) October (e) None of these Only a few Student is Lecturer.
4. If W is related to April, V is related to July then, P is All Lecturer is Principal.
related to which of the following? Conclusion: I. Some Student is Principal.
II. Some Lecturer is Professor.
(a) March (b) May (c) June
(d) August (e) October Direction (11-15): Study the following information
carefully and answer the questions given below:
5. Four of the following five are alike in certain way
based from a group, find the one which does not Eight persons are sitting around a square table. Four
belong to that group? persons are sitting at middle of the sides of the square
and all are facing towards inside. Remaining four are
(a) R (b) S (c) T
sitting at corners and they face outside.
(d) U (e) V
Two persons are sitting between P and U. R who is an
Direction (6-10): In each of the questions below are immediate neighbor of P, sits opposite to S. T sits 3rd to
given some statements followed by two conclusions. You the right of V. W sits immediate right of T. Q faces W.
have to take the given statements to be true even if they
11. Who among the following sits opposite to T?
seem to be at variance with commonly known facts. Read (a) P (b) R (c) S
all the conclusions and then decide which of the given (d) W (e) None of these
conclusions logically follows from the given statements
disregarding commonly known facts. 12. How many persons are sitting between P and V
when counted from left of P?
(a) If only conclusion I follows.
(a) Two
(b) If only conclusion II follows. (b) Three
(c) If either conclusion I or II follows. (c) Four
(d) If neither conclusion I nor II follows. (d) Either (a) or (c)
(e) If both conclusions I and II follow. (e) None of these
9 Adda247 | No. 1 APP for Banking & SSC Preparation
Website: bankersadda.com | sscadda.com | store.adda247.com | Email: contact@bankersadda.com
13. What is the position of Q with respect to R? T and U are seated (but not necessarily in the same
(a) Immediate right order) and all of them are facing south. In row 2 – A, B, C,
(b) Immediate left D, E and F are seated (but not necessarily in the same
(c) 2nd to the right order) and all of them are facing North. Therefore, in the
(d) 2nd to the left given seating arrangement each member seated in a row
(e) None of these faces another member of the other row. P faces D. U does
not face A, who sits left to E but not immediate left. R sit
14. Who among the following person sit 3rd to the right
at one of the ends and diagonally opposite to B. Three
of Q?
persons sit between B and F, who does not face U. C sits
(a) P (b) U (c) R
immediate left to D but does not faces S. Two persons sit
(d) S (e) None of these
between Q and U, none of them sits at the end. The one
15. Four of the following five are alike in certain way who faces T sits 2nd right to A.
based from a group, find the one which does not
21. Who among the following faces A?
belong to that group?
(a) S (b) T (c) Q
(a) Q (b) R (c) S
(d) R (e) none of these
(d) T (e) U
22. How many persons sit to the right of R?
Direction (16-17): Study the following information
(a) No One (b) one (c) two
carefully and answer the questions given below:
(d) three (e) four
Eight members are living in a family. Q is the only son of
P. T is wife of U. T is sister of Q and R. V is daughter in law 23. Four of the following five form a group ,who among
of W. S is son of T. W is the mother of Q. the following does not belongs to that group?
(a) U (b) T (c) E
16. How is S related to R? (d) F (e) A
(a) Son (b) Daughter (c) Nephew
(d) Niece (e) Can’t be determined 24. If in a certain way R is related to C, T is related to E,
then who among the following is related to D?
17. How many male members are in the family? (a) U (b) T (c) E
(a) Four (b) Five (c) Three (d) F (e) Q
(d) Six (e) None of these
25. Who among the following sit 3rd right to U?
18. How many such numerals are there in the number (a) R (b) T (c) P
‘254136987’ which will remain at the Same position (d) S (e) Q
when arranged in ascending order from left to
right? Directions (26-30): Study the following information
(a) one (b) two (c) three carefully and answer the questions given below:
(d) four (e) None of these In a certain code language
‘left right centre’ is written as ‘yo vo na’,
19. How many pairs of letters are there in the word ‘ahead below behind’ is written as ‘sa ra la’,
‘EDUCATION’, each of which have as many letters ‘above centre right’ is written as ‘ha vo na’, and
between then in the word as they have between ‘behind below above’ is written as ‘ha ra la’.
them in the English alphabet?
(a) one (b) two (c) three 26. What is the code for ‘left’?
(d) four (e) more than four (a) sa (b) ha (c) yo
(d) na (e) None of these
20. If four letter word is formed from 1st, 3rd, 5th and 6th
letter of TRANSLATE then what is the 3rd letter of 27. ‘behind’ will be written as?
newly formed word? If more than one meaningful (a) ra (b) ha (c) la
word is formed, then the answer will be Z. (d) Either (a) or (c) (e) None of
(a) L (b) T (c) A these
(d) S (e) Z 28. What is the code for ‘ahead’?
Directions (21-25): Read the following information (a) sa (b) yo (c) la
carefully and answer the questions given below: (d) ha (e) Can’t be determined

Twelve people are sitting in two parallel rows containing 29. What does ‘ha’ stand for?
six people each in such a way that there is an equal (a) behind (b) below (c) ahead
distance between adjacent persons. In row 1 – P, Q, R, S, (d) above (e) None of these

10 Adda247 | No. 1 APP for Banking & SSC Preparation


Website: bankersadda.com | sscadda.com | store.adda247.com | Email: contact@bankersadda.com
30. What is the code for ‘centre’? 34. Number of boxes above K is one less than the
(a) la (b) yo (c) sa number of boxes below ________?
(d) ha (e)Can’t be determined (a) S (b) R (c) F
Directions (31-35): Study the following information (d) D (e) None of these
and answer the questions given below:
35. How many boxes are there between M and H?
There are eleven boxes placed one above the other. Five (a) One (b) Two (c) Three
boxes are placed between F and T. Not more than five (d) None (e) More than three
boxes are kept above T. Two boxes are kept between T
and M. Three boxes are kept between M and S and M is Directions (36-40): In each of the question,
kept at one of the positions above S. There are only three relationships between some elements are shown in the
boxes kept above the box J. One box is kept between R statements. These statements are followed by
and S. Two boxes are kept between R and H. Box D is kept conclusions numbered I and II. Read the statements and
at one of the positions below box K and at one of the give the answer.
positions above box C which is not above R. Box E is kept
(a) If only conclusion I follows.
immediately above K.
(b) If only conclusion II follows.
31. How many boxes are placed between J and R? (c) If either conclusion I or II follows.
(a) 5 (b) 6 (c) 3 (d) If neither conclusion I nor II follows.
(d) 4 (e) None of these (e) If both conclusions I and II follow.
32. Which of the following statement is true regarding
36. Statements: C ≤ L = E ≤ R ≤ K = P ≥ O
C?
(a) C is placed at one of the positions above D Conclusions: I. P = C II. C < P
(b) C is placed immediately below F. 37. Statements: W > A = S ≥ H < I ≤ N ≤ G
(c) R is placed just above C Conclusions: I. H < W II. G > H
(d) C is placed at the bottom most position
(e) None of these 38. Statements: C < O ≤ D = S > A ≥ P ≥ Q
33. Which of the following is not true regarding J? Conclusions: I. Q < D II. C < A
(a) J is immediately below box T 39. Statements: F ≤ B = I ≤ C = A ≥ S > E
(b) One of the boxes below J is D Conclusions: I. S ≥ B II. F > E
(c) Number of boxes between J and S is four
(d) One of the boxes above J is K 40. Statements: I ≥ N = T ≥ E > L ≥ G > M
(e) One box is kept between J and M Conclusions: I. G < N II. I ≥ L

Quantitative Aptitude

Directions (41-45): Study the table given below and 43. If total no. of employee in E is 25% more than D and
answer the following Question no. of employee in HR dept is same as in company C,
then employee other than HR dept in company E is
what % of other dept employee in company B.
(a) 60% (b) 80% (c) 75%
(d) 50% (e) 55%

44. Find the difference between males of HR dept in


company C and D together and females of HR dept
41. Find the average no. of Females in HR department in company B and C together ?
together ? (a) 36 (b) 42 (c) 48
(a) 54 (b) 46 (c) 49 (d) 40 (e) 30
(d) 50 (e) 52
45. Find the average no. of employee other than HR
42. Females in the HR dept of company C is what %
dept. in A, B and C together ?
more than male in HR department of company A ?
(a) 280 (b) 270 (c) 220
(a) 250% (b) 200% (c) 100%
(d) 300% (e) 150% (d) 300 (e) 240

11 Adda247 | No. 1 APP for Banking & SSC Preparation


Website: bankersadda.com | sscadda.com | store.adda247.com | Email: contact@bankersadda.com
46. If there are total 150 females in company C then (a) 10000 (b) 14000 (c) 12000
how many female employees are there other than (d) 8000 (e) 11000
females of HR department 54. If ratio of ages of P and Q before 4 year ago is 5 : 4
(a) 90 (b) 100 (c) 80 and after 12 years sum of their ages will be 68 years,
(d) 110 (e) 120 their what was P’s age 2 years ago ?
Directions (47-51): Find the missing term in the (a) 24 years (b) 22 years (c) 18 years
following number series: (d) 26 years (e) 20 years
47. 1864, 1521, 1305, ? , 1116, 1089 55. If Pipes A and B can fill a tank in 15 min and 20 mins
(a) 1160 (b) 1180 (c) 1095 respectively and pipe C empties the tank in 12 mins.
(d) 1205 (e) 1220 what will be the time taken by A, B and C together
48. 18, ?, 9, 18, 72, 576 to fill the tank completely?
(a) 12 (b) 9 (c) 18 (a) 25 min (b) 30 min (c) 40 min
(d) 10 (e) 6 (d) 20 min (e)35 min
49. 12, 6.5, 7.5, 12.75, 27.5, ? Directions (56-60): Solve the given quadratic equations
(a) 66.5 (b) 68.75 (c) 63.75 and mark the correct option based on your answer—
(d) 71.25 (e) None of these (a) x > y
50. 5 , 15, 50, ?, 1030, 6185 (b) x < y
(a) 210 (b) 205 (c) 225 (c) x ≥ y
(d) 200 (e) 195 (d) x ≤ y
(e)x = y or there is no relationship
51. 130, 154, 186 , ? , 274, 330
(a) 216 (b) 220 (c) 240 56. (i) x² = 81 (ii) y² – 18y + 81 = 0
(d) 226 (e) 230 57. (i) 4x² - 24x + 32 = 0 (ii) y² - 8y + 15 = 0
52. If a boat travels 18 km more in downstream than in
58. (i) x² - 21x + 108 = 0 (ii) y² – 17y + 72 = 0
upstream in 3 hr. and if the speed of the Boat in still
water is 20 km/hr. find the distance travelled by 59. (i) x² – 11x + 30 = 0 (ii) y² - 15y + 56 = 0
boat in downstream in 4 hr. ?
60. (i) x³ = 512 (ii) y² = 64
(a) 86 (b) 92 (c) 68
(d) 96 (e) None of these 61. If a shopkeeper marks an item 50% above its CP and
53. If A invested Rs. 12000 at some rate of interest of S.I if 12% discount is given on the marked price and
and B joined him after 3 months investing 16000 at the shopkeeper makes profit of 256 Rs, then what
same rate of interest if A leaves before 2 month of will be the actual cost price of the item?
completion, then what will be the share of B’s profit (a) 1000 Rs. (b) 800 Rs. (c) 750 Rs.
after 1 year if total profit is 22000 Rs. ? (d) 1200 Rs. (e) 900 Rs.
Directions (62-67): The line graph shows the data of five seller selling an items (in units) on Monday and Tuesday.

450

400

350

300

250

200

150
A B C D E
Tuesday Monday

12 Adda247 | No. 1 APP for Banking & SSC Preparation


Website: bankersadda.com | sscadda.com | store.adda247.com | Email: contact@bankersadda.com
62. The no. of item sold by A and C together is how 70. Total students in art stream in A is what percent
much more or less then items sold by B and D more than total students in science stream in B?
together on both days? (a) 75% (b) 70% (c) 90%
(a) 250 (b) 280 (c) 300 (d) 100% (e) 110%
(d) 320 (e) 350
71. Find the ratio of total students in commerce stream
63. What is the average no. of items sold by all five
sellers on Monday ? in B to total students in science stream in A?
(a) 298 (b) 305 (c) 280 (a) 8 : 15 (b) 8 : 17 (c) 8 : 13
(d) 300 (e) 315 (d) 8 : 11 (e) 8 : 9
64. Items sold by B and C on Tuesday together is what 72. If in school C total students are720 students and
% more than same sellers on Monday together ? total students in science stream of school Care 25%
(a) 25% (b) 30% (c) 20% more than total students in commerce stream in
(d) 15% (e) 24%
school B, then find total students of art & commerce
65. Find the difference between items sold by B, D, E on stream in school C is how much less than total
Monday together items sold by B and E on Tuesday students in art and commerce stream in school A?
together (a) 120 (b) 110 (c) 150
(a) 150 (b) 180 (c) 160
(d) 100 (e) 140
(d) 120 (e) 200
66. Item sold On Monday by C and E together is 73. Find the average number of students in science
approximately what percentage of total items sold stream in school A & B?
by A and B together on Tuesday ? (a) 250 (b) 270 (c) 240
(a) 71% (b) 80% (c) 55% (d) 200 (e) 225
(d) 85% (e)65%
74. If out of total students in art stream of school A & B
67. Find the difference between the average items sold ratio of boys to girl is 5 : 3 and 7 : 4 respectively,
by A and B together on Monday and average of then find difference between boys and girls in art
items sold by B and C together on Tuesday?
stream of school A & B together?
(a) 45 (b) 35 (c) 25
(d) 40 (e) 50 (a) 220 (b) 225 (c) 240
(d) 248 (e) 224
68. If A start from P with speed 60 km/hr at 8:00 am
and B starts with speed 70 km/hr. at 8 : 30 am from 75. P invested 60% more than Q and R invested 20%
Q and total distance between P and Q is 680 km, find more than Q. If ratio of investment time-period (P:
at what time they will cross each other? Q: R) is 2: 4: 3 and the sum of profit shares of Q and
(a) 2 : 30pm (b) 1 : 30pm (c) 12 : 30pm R is Rs. 8550 then find the profit share of P.
(d) 3 : 00pm (e) 4 : 00pm
(a) Rs. 3200 (b) Rs. 4000 (c) Rs. 2400
69. If a person invested 6000 at T% S.I for 3 year and (d) Rs. 3600 (e) Rs. 3000
same amount at (T + 5)% CI for 2 year and
difference between both interest is 60 Rs. then find 76. When a person sold an article, his profit percent is
T ?(in %) 60% of the selling price. If the cost price is increased
(a) 15 (b) 18 (c) 20 by 75% and the selling price remains the same, then
(d) 24 (e) 25 find decrement in the profit is what percent of the
Direction (70−𝟕𝟒): Read the data carefully and answer selling price of the article?
the question. (a) 25% (b) 30% (c) 40%
(d) 27.5% (e) None of these
There are 1800 students in two school ‘A’ & ‘B’ and three
streams in each school i.e. art, science & commerce.18 77. Area of Istcircle and circumference of IInd circle is
3
4
% of total students in school A arein commerce stream 1386 cm2 and 176 cm respectively. There is a
4 5
and 28 7 % of total students in school B arein science square whose side is 35 7 % of twice of sum of the
stream. Sum of total students in commerce stream in A & radius of both the circles. Find the perimeter of the
1
science stream in B is 420. 19 21 % of total students in square (in cm)?
school B are in commerce stream and 50% of total (a) 132 (b) 136 (c) 140
students in school A are in Art stream. (d) 116 (e) 124
13 Adda247 | No. 1 APP for Banking & SSC Preparation
Website: bankersadda.com | sscadda.com | store.adda247.com | Email: contact@bankersadda.com
78. There are 5 red, 6 black and 5 blue balls in a bag. Out 1
(a) 48 %
1
(b) 53 % (c) 58 %
1
3 3 3
of these balls, four balls are picked at random from 1
the bag. Then, what is the probability that one is red, (d) 63 % (e) 60 %
3
two are black and one is blue ball? 80. A train travelling at 72 km/hr. classes a platform of
75 75 71
(a) 362 (b) 364 (c) 362 160 m in 18 second and another train travelling at
70 5
(d) 363 (e) 26 90 km/hr crosses the same platform in 15 second.
2 Find the length of another train?
79. An article is marked 66 3 % above the cost price and
(a) 160 m (b) 180 m (c) 140 m
loss incurred on selling that article is 25% of the
discount given on it. Then, find the discount % (d) 200 m (e) 215 m
given?

Solutions

REASONING ABILITY
Direction (1-5): 10. (a);
Months Persons
January X
March W
April P
May V Direction (11-15):
July S
August U
September Q
October T
November R
1. (a); 2. (e); 3. (e);
4. (b); 5. (a); 11. (e); 12. (b); 13. (b);
6. (d); 14. (d); 15. (a);
Direction (16-17):

7. (c);

16. (c); 17. (a);


18. (b);
8. (a);

9. (b); 19. (e);

20. (e);

14 Adda247 | No. 1 APP for Banking & SSC Preparation


Website: bankersadda.com | sscadda.com | store.adda247.com | Email: contact@bankersadda.com
Directions (21-25): Directions (31-35):
Boxes
E
K
T
J
H
M
D
21. (d); 22. (a); 23. (e); R
F
24. (e); 25. (e);
S
Directions (26-30): C

Word Code 31. (c); 32. (d); 33. (c);


Right/centre vo/na 34. (c); 35. (d);
Left yo
Direction (36-40):
Below/behind ra/la 36. (c); I. P = C (False) II. C < P (False)
Ahead sa
37. (e); I. H < W (True) II. G > H (True)
above ha
38. (a); I. Q < D (True) II. C < A (False)
26. (c); 27. (d); 28. (a);
39. (d); I. S ≥ B (False) II. F > E (False)
29. (d); 30. (e);
40. (a); I. G < N (True) II. I ≥ L (False)

Quantitative Aptitude

41. (c); Average no. of females in HR dept 45. (e); Average of A, B, C =


220+200+300
=
720
= 240
75 80 60 60 3 3
80× +50× +100× +60×
100 100 100 100
= 46. (a); Total females in company C = 150
4
=
60+40+60+36
=
196
= 49 females in HR department in company C
4 4 60
= 100× = 60
60 100
42. (b); Females in company C (HR) = 100× 100 = 60 therefore females other than in HR
25 department = 150-60= 90
Males in company A (HR) = 80 × 100 = 20
Difference = 60 – 20 = 40 47. (b);
40
∴ % = 20 × 100 = 200% more
125
43. (c); Total employee in E = 200 × 100 = 250
48. (b); 18, ?, 9, 18, 72, 576
∴ employee of HR dept in E = 100 18 × 0.5 = 9
∴ other employee = 150 9×1=9
100
∴% of other employee = 150 × 200 = 75% 9 × 2 = 18
18 × 4 = 72
44. (a); Males in HR dept in C and D 72 × 8 = 576
40 40
= 100 × 100 + 60 × 100 = 40 + 24 = 64 49. (d); 12 × 0.5 + 0.5 = 6.5
Females in HR dept of B and C = 50 ×
80
+ 6.5 × 1 + 1 = 7.5
100
60 7.5 × 1.5 + 1.5 = 12.75
100 × 100 = 100 12.75 × 2 + 2 = 27.5
∴ Difference = 100 – 64 = 36 27.5 × 2.5 + 2.5 = 71.25

15 Adda247 | No. 1 APP for Banking & SSC Preparation


Website: bankersadda.com | sscadda.com | store.adda247.com | Email: contact@bankersadda.com
50. (b); 5 × 2 + 5 = 15 57. (e); 4x² – 24x + 30 = 0
15 × 3 + 5 = 50 4x² – 16x – 8x + 32 = 0
50 × 4 + 5 = 205 4x (x – 4) –8 (x–4) = 0
x = 4, 2
205 × 5 + 5 = 1030
y² – 8y + 15 = 0
1030 × 6 + 5 = 6185
y² – 5y – 3y + 15 = 0
51. (d); y(y – 5)–3 (y – 5) = 0
∴ y = 5, 3
∴ No relation exists
58. (c); x² – 21x + 108 = 0
x² – 9x – 12x + 108 = 0
52. (b); (Ds –Du ) 3 = 18 km
x(x – 9) – 12 (x – 9) = 0
Different in 1 hr. = 6km x = 9, 12
Ds and Du y² – 17y + 72 = 0
∴ Speed of boat in still water = 20 km/hr. ∴ y² – 8y – 9y + 72 = 0
Ds = 23 km/hr., Du = 17 km/hr. y (y – 8) – 9 (y – 8) = 0
Distance travelled = 4 × 23 = 92 km ∴ y = 8, 9
∴x≥y
53. (c);
59. (b); x² – 11x + 30 = 0
x² – 6x – 5x + 30 = 0
∴ x(x – 6) – 5(x – 6) = 0
x = 6, 5
y² – 15y + 56 = 0
6 y² – 7y – 8y + 56 = 0
∴ B’s share = 22000 × = 12000 y (y – 7) – 8 (y – 7) = 0
11
∴ y = 7, 8
54. (b); P Q
∴x<y
–4 5 4
+12 P + Q = 68 60. (c); x³ = 512
3
Age increased in 16 year = 32 years x = √512 = 8
Sum of Age of P and Q before 4 years = 36 y² = 64
∴ 5x + 4x = 36 y = √64 = ± 8
∴x≥y
X=4
P’s age 2 years ago = 5x + 2 = 22 years 61. (b); Let CP = 100 x
∴ marked price = 150x
55. (b); ∴ selling price after giving discount = 132x
∴ 32x = 256
x=8
∴ CP = Rs 800
62. (b); Item sold by A and C = 550 + 570 = 1120
∴ tank filled in 1 min = 2 units Item sold by B and D = 750 + 650 = 1400
60
Total time = = 30 minutes ∴ diff. =1400 – 1120 = 280
2
300+350+250+380+210 1490
56. (d); x² = 81 63. (a); Average = 5
= 5
= 298
x=±9 64. (c); Item sold by B and C on Monday
Y² – 18y + 81 = 0 = 350 + 250 = 600
(y – 9)² = 0 Item sold by B and C on Tuesday
∴ y = 9, 9 = 400 + 320 = 720
100
∴x≤y ∴ % increase = 120 × 600 = 20%

16 Adda247 | No. 1 APP for Banking & SSC Preparation


Website: bankersadda.com | sscadda.com | store.adda247.com | Email: contact@bankersadda.com
65. (d); Items sold on Monday by B, D and E Total students in school A = 960
= 350 + 380 + 210 = 940 And total students in school B = 840
Item sold on Tuesday by B and E = 400 + 420 Total students in school B in commerce stream
400 1
= 820 = 21 × 100 × 840 = 160
∴ diff = 940 – 820 = 120 Total students in school A in art stream
1
= × 960 = 480
66. (a); Item sold by C and E on Monday 2

= 250 + 210 = 460 Now, total students in school A in science stream


3
Item sold by A and B together on Tuesday = 960− 16 × 960 − 480 = 300
= 400 + 250 = 650 And total students in school B in art stream
2
∴ ? = 460 ×
100 = 840 − 7 × 840 − 160 = 440
650
≃ 71% (approx) Streams A B
650 Art 480 440
67. (b); Avg. by A and B on Monday = 2
= 325 Commerce 180 160
Avg. of B and C on Tuesday =
720
= 360 Science 300 240
2
Diff. = 360 – 325 = 35 70. (d); Required percentage =
480 −240
× 100 = 100%
240
68. (b); 160
71. (a); Required ratio = = 8 : 15
300

72. (e); Total student art & commerce stream in C


125
= 720 – 160 × = 520
100

1
Required difference
Dist travelled by A in 2 hr = 30 km = (480 + 180) – 520 = 140
Remaining distance to be covered = 680 – 30 300+240 540
73. (b); Required average = = = 270
= 650 km 2 2
74. (c); Total boys in art stream of school A & B
Relative speed = 60 + 70 = 130
650
together
∴ time taken = = 5 hr 5 7
= 480 × + 440 × = 300 + 280 = 580
130
8 11
∴ time = 8 : 30 + 5 hr = 1 : 30 pm Total girls in art stream of school A & B
69. (a); By going with the options together
3 4
6000×3×15 = 480 × 8 + 440 × 11 = 180 + 160 = 340
Interest received at SI = = 2700 Rs
100
Required difference = 580 −340 = 240
∴ T + 5 = 20%
6000×44 75. (d); Let the investment of Q = 100x
Interest received after 2 yrs at CI = 100 Investment of P = 160x
= 2640 Investment of R = 120x
∴ Difference = 2700 – 2640 = 60 Rs Ratio of profit:
T=15% P Q R
160x × 2 100x × 4 120x × 3
Direction (70−𝟕𝟒): 8 : 10 : 9
Let total students in A = x ATQ,
And, total students in B = y 19 unit = Rs. 8550
Total students in school A in commerce stream 8 unit = 450 × 8 = Rs. 3600

=x×
75
×
1
=
3x
76. (b); Let the selling price be 250x
4 100 16
then, profit = 150x
Total students in school B in science stream
200 1 2y
CP=250x − 150x = 100x
=y × 7
× 100 = 7
175
Now, new C.P. = 100x × 100 = 175x
3x 2y
Given, 16 + 7
= 420 … (i) New S.P. = 250x
And x + y = 1800 … (ii) New profit = 250x – 175x = 75x
150x−75x
So, from (i) and (ii), Required % = 250x × 100 = 30%

17 Adda247 | No. 1 APP for Banking & SSC Preparation


Website: bankersadda.com | sscadda.com | store.adda247.com | Email: contact@bankersadda.com
77. (c); Circumference of any circle = 2π × radius 79. (b); Let the cost price be Rs 3x
1386 Then the marked price= Rs 5x
Radius of 1st circle = √ = 21 cm And let the discount given be Rs 4y
π
176 Then loss incurred= Rs y
Radius of 2nd circle = 2π
= 28 cm ATQ
Side of square =
5
× 2 × (21 + 28) = 35 cm 3x − y = 5x − 4y
14 3y = 2x
Perimeter of square = 4 × 35 = 140 cm 15
Marked price=Rs 2 y
78. (b); Ways to select 4 balls out of 16 balls = 16C 4 4y
Required discount %=15 × 100 = 53 3 %
1
y
Ways to select one red balls = 5C 1 2
5
80. (e); Speed of 1st train = 72 × 18 = 20 m/s
Ways to select two black balls = 6C 2
∴ Dist travelled by 1st train = 20 × 18 = 360 m
Ways to select one blue balls = 5C 1 ∴ length of train (1st) = 360 – 160 = 200 m
5
∴ Required probability Speed of 2nd train = 90 × 18 = 25 m/s
5C 1 ×6C 2 ×5C 1 75
= = ∴ Distance travelled = 25 × 15 = 375 m
16C 4 364
∴ length of 2nd train = 375 − 160 = 215 m

18 Adda247 | No. 1 APP for Banking & SSC Preparation


Website: bankersadda.com | sscadda.com | store.adda247.com | Email: contact@bankersadda.com
IBPS RRB PO Prelims 2018| Memory Based Paper |
For Practice

REASONING ABILITY

Direction (1-5): Study the following information 6. If point X is 6m south of point A then which point is
carefully and answer the given questions: at shortest distance from point X?
(a) E (b) A (c) F
Eleven boxes A, B, C, D, E, F, G, H, I, J, K are kept one above (d) B (e) G
the other. Box G is kept at fifth position from the top. Two
boxes are kept between G and H. Box D is kept just above 7. What is the distance of point C from point H?
box H. There are as many boxes above box D as below (a) 9m (b) 5m (c) 4m
box B. Five boxes are kept between box F and box K, (d) 6m (e) 7m
which is kept at one of the positions below box G. Box A 8. Point B is in which direction with respect to point F?
is kept at one of the positions above box F. Only one box (a) South (b) South-east (c) North
is kept between Box G and Box C. Box I is kept above box (d) North-east (e) North-west
E but not just above. Box E is not kept immediately above
Direction (9-13): Study the following information
or immediately below box C.
carefully and answer the given questions
1. What is the position of box I? Eight persons A, B, C, D, E, F, G, H are sitting around a
(a) 8th from the bottom circular table such that five of them are facing towards
(b) 7th from the top the center and the rest are facing away from the center.
(c) 3rd from the top Three persons are sitting between F and H, who is facing
(d) 6th from the bottom center. C is 2nd to the right of F and faces opposite
(e) none of these direction to F. A sits 3rd to the left of C.G is one of the
neighbor of E. Two persons sit between G and B, who is
2. How many boxes are kept between box E and Box H? not neighbor of H.G does not face C. G and A face same
(a) seven (b)six (c) five direction but opposite to F.
(d) four (e) eight
9. What is the position of E with respect to A?
3. Which among the following statement is true (a) immediate right (b) 5th to the left
regarding box J? (c) 2nd to the right (d) 2nd to the left
(a) it is 7th from the bottom (e) none of these
(b) Box K is placed above box J
10. How many persons are sitting between C and H, wen
(c) only two boxes are kept between box B and box counted from the left of C?
J (a) one (b) two (c) three
(d) It is kept just below box H (d) four (e) none
(e) All are true 11. Four of the five are alike in a certain way, which
4. Which of the following represents the boxes kept among the following does not belongs to that group?
between boxes A and I? (a) C (b) B (c) F
(a)C, B (b) A, K (c) F, G (d) D (e) E
(d) J, D (e) none of these 12. Which of the following represents the immediate
5. Which of the following box is kept just above box B? neighbor of G?
(a) C (b) K (c) F (a) C (b) B (c) F
(d) D (e) A
(d) D (e) none of these
13. Which of the following is not true regarding F?
Direction (6-8): Study the following information
(a) it faces towards the center
carefully and answer the given questions
(b) E is immediate left to F
Point B is 14m east of point A. Point C is 9m north of point (c) Two persons sit between F and D, when counted
B. Point D is 12m east of point C. Point E is 15m south of from the right to D
point D. Point F is 30m west of point E. Point G is 10m (d) All are true
north of point F. Point H is 18 m east of point G. (e) no one sits between F and B

19 Adda247 | No. 1 APP for Banking & SSC Preparation


Website: bankersadda.com | sscadda.com | store.adda247.com | Email: contact@bankersadda.com
Direction (14-18): Study the following information released immediately after 100-minute duration movie.
carefully and answer the given questions. Movie C released immediately after the one which is of
130-minute duration. More than two movies released in
Certain number of persons are sitting in a row facing
between C and D. The movie which is of 90-minute
north. M sits 4th to the right of S. Five persons sit
duration released before E. One of movie was of 20
between M and X. T sits at one of the positions left to S.
minutes more duration than E.
The number of persons sitting between M and U are
same as between S and T. Q is 2nd from one of the 21. How many movies were released after E?
extreme ends. Four persons sit between S and U. No one (a) One (b) Two (c) None
sits to the right of N, who is immediate right to P. X is 3rd (d) Three (e) More than three
left to P. Not more than two persons sit between Q and U.
22. Which of the following movie was of 150-minute
14. How many persons are sitting in the row? duration?
(a) 17 (b) 20 (c) 24 (a) E
(d) 26 (e) 27 (b) A
15. How many persons are sitting between S and T? (c) There is no such movie
(a) seven (b)six (c) five (d) C
(d) four (e) eight (e) D

16. What is the position of U from the left end? 23. What is the total duration of movie D and E together?
(a) 6th (b) 5th (c) 4th (a) 135 (b) 225 (c) 165
(d) 2nd (e) 3rd (d) 175 (e) 190

17. How many persons are sitting between Q and M? 24. Which of the following statement is true regarding
(a) seven (b) eleven (c) ten B?
(d) nine (e) eight (a) The movie released after B is of 120-minute
duration
18. Which of the following represents the person sitting (b) Two movies released in between A and B
at extreme end? (c) Movie B is of 100-minute duration
(a) M (b) U (c) X (d) Total duration of movie B and A is 225 minutes
(d) P (e) T (e) Movie A released after B.
19. If the second, forth, seventh and eighth letter of the 25. Which of the following statement is true?
word “FRACTION” are combined to form a (a) The movie released before A is of 130-minute
meaningful word, then what will be the 3rd letter duration
from the left in the so formed word. If more than one (b) Three movies released in between A and E
meaningful word is formed then the answer is X, if no (c) No movie released in between A and E
such word is formed then answer is Z? (d) Total duration of movie C and A is 230 minutes
(a) O (b) X (c) R (e) Movie C released immediately after E.
(d) Z (e) C
Direction (26-28): Study the following information
20. How many pair of digits have same number of digits carefully and answer the given questions:
between them in the number “573814269” as in the
numeric series? F is the husband of G. K is the mother-in-law of G. H is the
(a) five (b) four (c) six Father of F. M is the mother of H, P is the mother of K and
(d) three (e) more than six B.

Direction (21-25): Study the following information 26. If Y is the father of H then how is Y related to M?
carefully and answer the given questions: (a) Mother (b) Father (c) Sister
(d) Brother (e) Husband
Movies of different duration released on different days
starting from Monday to Friday (starting from Monday). 27. How is P related to F?
Movie A was released On Tuesday. No movie released (a) Grandfather (b) Aunt (c) Mother
between A and the one which is of 75-minute duration. (d) Grandmother (e) Wife
Only one movie is released between the one which is of 28. How is B related to H?
75-minute duration and the one which is of 100-minute (a) Sister (b) Brother (c)Husband
duration. No movie released between the one which is of (d) Can’t be determine (e) Wife
100 minute and B. Only one movie released after B. B
20 Adda247 | No. 1 APP for Banking & SSC Preparation
Website: bankersadda.com | sscadda.com | store.adda247.com | Email: contact@bankersadda.com
Direction (29-31): Study the following information (a) Neither I nor II follows
carefully and answer the given questions: There are six (b) Only I follows
persons M, N, O, P, Q, R of different heights. N is shorter (c) Either I or II follow
than M but taller than Q. Only two person are taller than (d) Both I and II follow
M. R is taller than Q and O. Q is not the shortest. The one (e) Only II follows
who is second shortest is 154m. P is not the shortest 35. Statements: Some Door are Fan.
person. No Door is Rose.
29. If M is 19m taller than Q then what is the height of M? No Fan is Shelf.
(a) 190m (b) 181m (c) 175m Conclusions: I. All Door are Shelf is a possibility.
(d) 130m (e) 173m II. All Shelf can be Doors.
(a) Either I or II follows
30. If P is 181m than which of the following is true? (b) Only II follows
I. Only one person is taller than P. (c) Neither I nor II follow
II. The difference between the heights of P and Q is (d) Both I and II follow
27m (e) Only I follows
III. O is the shortest person.
(a) Only I (b) Only II and I Direction (36-40): Study the following information
(c) All are true (d) Only III and II carefully and answer the given questions:
(e) Only III and I Fourteen persons are sitting in two parallel rows such
that seven persons are sitting in each row. A, B, C, D, E, F,
31. How many persons are shorter than N? G are sitting in row-1 facing north while P, Q, R, S, T, U, V
(a) One (b) Two (c) None are sitting in row-2 facing south. G sits third to the left of
(d) Three (e) More than three A and neither of them sits at an extreme end of the row.
Directions (32-35): Question consists of Some The one faces A sits immediate right to T. Only one
statements followed by two conclusions. Consider the person sits between T and Q. The one who faces Q sits
given statements to be true even if they seem to be at third to the right of E. S sits to the immediate left of V. S
variance with commonly known facts. Read all the neither faces G nor E. D is an immediate neighbour of the
one who faces S. The one who faces C sits fifth to the left
conclusions and then decide which of the given
of P. B sits third to the left of F. U sits at one of position to
conclusions logically follow from the given statements
the right of R.
using all statements together.
36. Four of the following are alike in a certain way so
32. Statements: All Grills are Arrow.
form a group which of the following does not belong
Some Hat are Grills.
to that group?
Some Cell are Arrow.
(a) U (b) B (c) T
Conclusions: I. Some Cell are definitely not (d) C (e) P
Grills.
II. Some Hat can never be Arrow. 37. How many persons sits between F and C?
(a) Only I follows (a) One (b) Two (c) None
(b) Only II follows (d) Three (e) More than three
(c) Neither I nor II follow 38. Which of the following is not true regarding U?
(d) Both I and II follow (a) No one sits to the right of U
(e) Either I or II follow (b) U sits third to the right of Q,
33. Statements: All Grills are Arrow. (c) P is an immediate neighbour of U.
Some Hat are Grills. (d) E is an immediate neighbour of the one who
Some Cell are Arrow. faces U,
Conclusions: I. Some Hat are Arrow. (e) Only two persons sit between U and S
II. Some Grills are Cell. 39. What is the position of C with respect to A?
(a) Only II follows (a) Second to the left
(b) Only I follows (b) Third to the right
(c) Either I nor II follow (c) Immediate right
(d) Both I and II follow (d) Immediate left
(e) Neither I or II follow (e) Second to the right
34. Statements: Some Door are Fan.
40. What is the position of B with respect to D?
No Door is Rose.
(a) Third to the left
No Fan is Shelf.
(b) Second to the left
Conclusions: I. Some Fan can never be Rose.
(c) Forth to the left
II. Some Rose are Shelf is a
(d) Third to the right
possibility. (e) Fifth to the right
21 Adda247 | No. 1 APP for Banking & SSC Preparation
Website: bankersadda.com | sscadda.com | store.adda247.com | Email: contact@bankersadda.com
QUANTITATIVE APTITUDE
Directions (41-45): Find the wrong number in the 43. 250, 260, 291, 314, 340, 370, 405
following number series ? (a) 370 (b) 314 (c) 260
(d) 405 (e) 250
41. 1, 3, 7, 15, 31, 64, 127
(a) 1 (b) 3 (c) 15 44. 750, 535, 411, 348, 322, 314, 315
(d) 64 (e) 127 (a) 315 (b) 750 (c) 411
(d) 348 (e) 314
42. 1, 15, 119, 475, 949, 947, 473
45. 2, 7, 27, 107, 427, 1708, 6827
(a) 947 (b) 475 (c) 15
(a) 107 (b) 1708 (c) 2
(d) 473 (e) 1 (d) 6827 (e) 7
Directions (46-50): Study the line-graph carefully & answer the question given below.
Line-graph given below shows the total no. of products for (kid + adult) in two different stores P & Q in five different years.
Store P Store Q
80

70
No. of product

60

50

40

30

20
2000 2001 2002 2003 2004

46. What is the difference between total no. of products 50. Total no. of products in store P in year 2003 and in
in store P in year 2003 & 2004 together and total no. store Q in year 2004 together is what percent
of products in year 2000? more/less than total no. of products in store Q in
(a) None of these (b) 10 (c) 20 year 2000?
(d) 15 (e) 5 (a) 150% (b) 40% (c) 125%
47. If total products in both the stores in year 2006 is (d) 100% (e) 50%
increased by 20% as compared to year 2004. Then
find total no. of products in year 2006? Directions (51-55): Solve the given quadratic equations
(a) 102 (b) None of these and mark the correct option based on your answer—
(c) 96 (d) 108 (a) x ≥ y (b) x ≤ y (c) x > y
(e) 92 (d) x = y or no relation can be established between x and
y.
48. What is the ratio of total products in store Q in year
(e) x < y
2002 & 2003 together to total products in store Q in
year 2000? 51. (i) x 2 − 20x + 96 = 0 (ii) y 2 = 64
(a) 23 : 12 (b) 23 : 11 (c) 28 : 11
(d) None of these (e) 27 : 13 52. (i) 4x 2 − 21x + 20 = 0 (ii) 3y2 − 19y + 30 = 0

49. What is the average no. of products in all the years 53. (i) x2 − 11x + 24 = 0 (ii) y2 − 12y + 27 = 0
together in store P?
54. (i) x2 + 12x + 35 =0 (ii) 5y2 + 33y + 40 =0
(a) 48 (b) 43 (c) 57
(d) None of these (e) 53 55. (i) 4x2 + 9x + 5 =0 (ii) 3y2 + 5y + 2 =0

22 Adda247 | No. 1 APP for Banking & SSC Preparation


Website: bankersadda.com | sscadda.com | store.adda247.com | Email: contact@bankersadda.com
Directions (56-60): Study the following paragraph 56. How many students appeared in at least two exams?
carefully & answer the question given below. (a) 240 (b) 260 (c) 300
(d) 360 (e) 500
There are 1000 students in a college. Out of 1000
students some appeared in exams ‘X’, ‘Y’ and ‘Z’ while 57. How many students appeared in two exams only?
some not. Number of student not appeared in any exam (a) 280 (b) 220 (c) 340
is equal to number of students appeared in exam ‘Z’ only. (d) 300 (e) 260
Number of students appeared in exam ‘Y’ is 360. Ratio of
58. How many students appeared in at most two exams?
number of students appeared in exam ‘X’ and ‘Y’ only to
(a) 240 (b) 260 (c) 300
number of students appeared in exam ‘Y’ and ‘Z’ only is 2
(d) 500 (e) 960
: 3. Number of student appeared in exam ‘X’ and ‘Z’ both
is half of number of students appeared in only exam ‘Z’. 59. How many students not appeared in exam Y?
Number of students appeared in exam ‘X’ only is 50% (a) 440 (b) 360 (c) 540
more than number of students appeared in ‘Y’ only. (d) 640 (e) None of these
Number of students appeared in all the three exam is 4% 60. How many students appeared in exam X or in exam
of the total number of students in the college. Number of Z?
students appeared in ‘Y’ exam only is same as number of (a) 240 (b) 360 (c) 500
students appeared in ‘Y’ and ‘Z’ only.
(d) 680 (e)760
Direction (61-65): Bar chart given below shows Number of tigers in different National Parks i.e. A to D of a country in
two different years. Study the data carefully and answer the following questions
1998 2018
100
Number of tigers →

80

60

40

20

0
A B C D
National Parks→
61. Number of tigers in National Park B and C together 64. Number of tigers in National Park ‘E’ in 2018 is 40%
in 2018 is how much less more/less than Number of more than number of tigers in National Park ‘D’ in
1998 while number of tigers in National park ‘E’ in
tigers in National Park A and D together in 1998?
1998 is 25% less than number of tigers in National
(a) 40 (b) 44 (c) 52 Park ‘C’ in 2018. Find total number of tigers in
(d) 60 (e) 72 National park ‘E’ in 1998 and 2018 together?
(a) 148 (b) 84 (c) 172
62. Number of tigers in National Park ‘D’ in both years (d) 160 (e) 136
together is what percent of the Number of tigers in
65. Average number of tigers in all National park in 2018
National Park ‘C’ in both years together? is how much less/more than average number of
(a) 60% (b) 160% (c) 140% tigers in all National park in 1998?
(d) 120% (e) 180% (a) 14 (b) 16 (c) 18
(d) 20 (e) 22
63. Find the ratio between number of tigers in National 66. The difference between downstream speed and
Park ‘A’ in 2018 to number of tigers in National Park upstream speed of boat is 6 km/hr and boat travels
‘B’ in 1998? 72 km from P to Q (downstream) in 4 hours. Then
(a) 9 : 10 (b) 10 : 9 (c) 16 : 13 find the speed of boat in still water?
(a) 15 km/hr (b) 18 km/hr (c)20km/hr
(d) 13 : 16 (e) 3 : 4 (d) 16 km/hr (e) 24 km/hr

23 Adda247 | No. 1 APP for Banking & SSC Preparation


Website: bankersadda.com | sscadda.com | store.adda247.com | Email: contact@bankersadda.com
67. In a vessel, there are two types of liquids A and B in 73. The sum of four times of an amount ‘x’ and (x – 9.75) is
the ratio of 5 : 9. 28 lit of the mixture is taken out and Rs. 442. Find the approximate value of x.
2 lit of type B liquid is poured into it, the new (a) Rs. 85 (b) Rs. 90 (c) Rs. 100
ratio(A:B) thus formed is 1 : 2. Find the initial (d) Rs. 1100 (e) Rs. 75
quantity of mixture in the vessel? 74. A and B entered into a partnership by investing some
(a) 84 lit (b) 42 lit (c) 50 lit amounts. The investment of A is twice of the
(d) 56 lit (e) 70 lit investment of B. Another person C joined them after
68. The average weight of 5 students in a class is 25.8 kg. 4 months. At the end of a year, the profit share of A
and C is equal. Then find the profit share of B is what
When a new student joined them, the average weight
percent of the profit share of C.
is increased by 3.9 kg. Then find the approximate 1
(a) 50% (b) 33 % (c) 40%
weight of the new student. 3
(a) 55 kg (b) 49 kg (c) 42 kg (d) 60% (e) 75%
(d) 44 kg (e) 58 kg 75. The ratio of age of Ishu 8 years hence and that of
69. A person has purchased two adjacent plots, one is in Ahana 6 years hence is 5 : 6. The age of Ishu 10 years
hence is equal to the age of Ahana 6 years hence.
rectangular shape and other is in square shape and
Then, find the present age of Ishu.
combined them to make a single new plot. The
(a) 1.5 yr (b) 2 yr (c) 3 yr
breadth of the rectangular plot is equal to the side of
(d) 4 yr (e) 5 yr
the square plot and the cost of fencing the new plot
is Rs. 390 (Rs. 5/m). Find the side of square if the 76. What is the difference between 20% of P and 20% of
length of the rectangular plot is 15 m. (P + 5000).
(a) 10 m (b) 8 m (c) 12 m (a) 1500 (b) 1200 (c) 1000
(d) 9 m (e) 6 m (d) 2000 (e) 1600
77. The ratio of the diameter of base and height of a
70. A shopkeeper marked his article 50% above the cost
cylinder is 2 : 3. Find the radius of the cylinder if the
price and gives a discount of 20% on it. If he had
approximate volume of cylinder is 3234.01 cm³?
marked his article 75% above the cost price and 21 7
(a) cm (b) cm (c) 21 cm
gives a discount of 20% on it then find the earlier 2 2
profit is what percent of the profit earned latter? (d) 7 cm (e) 14 cm
1
(a) 50% (b) 60% (c) 333% 78. A train of some length passes the platform of length
(d) 40% (e) 75% 524 m in 55 seconds. Find the length of train if the
speed of train is 72 km/hr.
71. A person invested two equal amounts in two (a) 476 m (b) None of these
different schemes. In first scheme, amount is (c) 428 m (d) 526 m
invested at 8% p.a. on SI for T years and SI received (e) 576 m
is Rs 2000 while in second scheme, amount is
79. Efficiency of B is two times more than efficiency of A.
invested at 10% p.a. for 2 years at CI and the
Both started working alternatively, starting with B
compound interest received is Rs. 1050. Find the and completed the work in total 37 days. If C alone
value of T. complete the same work in 50 days then find in how
(a) 4 yr (b) 8 yr (c) 6 yr many days A and C together will complete the work?
(d) 5 yr (e) 3 yr
72. Satish saves 20% of his monthly salary. And of the (a) 24 days (b) 30 days (c) 36 days
1 1 (d) 48 days (e) 18 days
remaining salary 4
th and 2
th he gives to his mother
and sister respectively and the remaining salary he 80. 7 men and 6 women together can complete a piece of
work in 8 days and work done by a women in one
submits as his EMI for the payment of his car. If his
day is half the work done by a man in one day. If 8
annual EMI was Rs. 60,000, then find his monthly
men and 4 women started working and after 3 days
salary?
4 men left the work and 4 new women joined then,
(a) Rs. 40,000 (b) Rs. 35,000 in how many more days will the work be completed
(c) Rs. 32,000 (d) Rs. 30,000 (a) 7 days (b) 6 days (c) 5.25 days
(e) Rs. 25,000 (d) 6.25 days (e) 8.14 days
24 Adda247 | No. 1 APP for Banking & SSC Preparation
Website: bankersadda.com | sscadda.com | store.adda247.com | Email: contact@bankersadda.com
Solutions

REASONING ABILITY
Direction (1-5): 19. (c); 2nd, 4th,7th and 8th letters are R, C, O, N
BOX The meaningful word formed is CORN
D
H 20. (e);
A
F
G
I
C
Direction (21-25):
J
E Days Movies Duration
K Monday D 75
B Tuesday A 90
1. (d); 2. (b); 3. (c); Wednesday E 100
4. (c); 5. (b); Thursday B 130
Direction (6-8): Friday C 120

21. (b); 22. (c); 23. (d);

24. (a); 25. (c);

Direction (26-28):

6. (c); 7. (b); 8. (d);


Direction (9-13):

26. (e); 27. (d); 28. (d);

Direction (29-31): R/P > R/P > M > N > Q > O

29. (e); 30. (d); 31. (b);

32. (c);
9. (d); 10. (a); 11. (a);
12. (e); 13. (c);

Direction (14-18):

14. (c); 15. (e); 16. (b);


17. (b); 18. (e);

25 Adda247 | No. 1 APP for Banking & SSC Preparation


Website: bankersadda.com | sscadda.com | store.adda247.com | Email: contact@bankersadda.com
33. (b); 35. (b);

Direction (36-40):

34. (d);

36. (e); 37. (b); 38. (b);

39. (c); 40. (c);

QUANTITATIVE APTITUDE
41. (d); 48. (c); Required ratio
80+60 140
= 55
= 55
= 28 : 11

49. (b); Required Average


25+40+65+55+30 215
42. (a); = =
5
= 43
5

50. (d); Required percentage


(55 + 55) − 55
= 55
× 100
43. (c); =
55
× 100 = 100%
55

51. (a); (i) x 2 − 20x + 96 = 0


x 2 − 12x − 8x + 96 = 0
x(x − 12) − 8(x − 12) = 0
44. (e); (x − 12)(x − 8) = 0
x = 12,8
(ii) y 2 = 64
y = ±8
∴ x≥y
45. (b);
52. (d); (i) 4x 2 − 21x + 20 = 0
4x 2 − 16x − 5x + 20 = 0
4x(x − 4) − 5(x − 4) = 0
Alternate, (4x − 5)(x − 4) = 0
5
x = 4,4
(ii) 3y 2 − 19y + 30 = 0
3y 2 – 9y − 10y + 30 = 0
3y(y − 3) − 10(y − 3) = 0
46. (e); Required difference
(3y − 10)(y − 3) = 0
= (55 + 30) – (55 + 25) = 5 10
y= 3
,3
47. (a); Total no. of products in year 2006 ∴ No relation can be established
120
= (55 + 30) × 100 = 102.0 between x and y

26 Adda247 | No. 1 APP for Banking & SSC Preparation


Website: bankersadda.com | sscadda.com | store.adda247.com | Email: contact@bankersadda.com
53. (d); (i) x 2 − 11x + 24 = 0 Number of students appeared in Y exam only
x 2 − 8x − 3x + 24 = 0 = No. of students appeared in Y and Z only = 3x
x(x − 8) − 3(x − 8) = 0 Number of students appeared in exam X and Y only
2
(x − 3)(x − 8) = 0 = 3 × 3x = 2x
x = 3 ,8
(ii) y 2 − 12y + 27 = 0
y 2 – 9y − 3y + 27 = 0
y(y − 9) − 3(y − 9) = 0
(y − 9)(y − 3) = 0
y = 9 ,3
∴ No relation can be established
between x and y

54. (b); (i) x 2 + 12x + 35 = 0


x 2 + 7x + 5x + 35 = 0
Now, 2x + 3x + 3x + 40 = 360
x(x + 7) + 5(x + 7) = 0
⇒ x = 40
(x + 7)(x + 5) = 0 a
and, 12.5x + a + + a = 1000
x = −7 , −5 2
5a
(ii) 5y 2 + 33y + 40 = 0 = 500
2
5y 2 + 25y + 8y + 40 = 0 ⇒ a = 200
5y(y + 5) + 8(y + 5) = 0
(y + 5)(5y + 8) = 0
8
y = − , −5
5
∴ y≥x
55. (b); (i) 4x 2 + 9x + 5 = 0
4x 2 + 4x + 5x + 5 = 0
4x(x + 1) + 5(x + 1) = 0
(4x + 5)(x + 1) = 0
5
x = −1 , − 4
(ii) 3y 2 + 5y + 2 = 0 56. (c); Students appeared in atleast two exams
3y 2 + 3y + 2y + 2 = 0 = 80 + 60 + 40 + 120 = 300
3y(y + 1) + 2(y + 1) = 0
57. (e); Students appeared in two exams only = 80 + 60
(3y + 2)(y + 1) = 0
+ 120 = 260
2
y = − 3 , −1
58. (e); Students appeared in atmost two exams
∴ y≥x
= 180 + 120 + 200 + 60 + 80 + 120 + 200= 960
Solutions (56-60):
59. (d); Student not appeared in exam Y
Total students = 1000 = 1000 – 360 = 640
Let, students appear in exam Z only = a
Total students appeared in exam Y = 360 60. (d); Students appeared in exam X or in exam Z
Ratio of number of students appeared in exam X and Y = 180 + 60 + 40 + 80 + 200 + 120 = 680
only to students appeared in exam Y and Z only 61. (d); Number of tigers in National Park B and C
=2:3 together in 2018 = 52 + 32 = 84
Students appeared in exam X and Z both Number of tigers in National Park A and D
= a/2 together in 1998
Number of students appeared in all three exams = 64+80 = 144
4
= 100 × 1000 = 40 Required difference = 144-84 = 60

27 Adda247 | No. 1 APP for Banking & SSC Preparation


Website: bankersadda.com | sscadda.com | store.adda247.com | Email: contact@bankersadda.com
62. (b); Number of tigers in National Park D in 1998 Let the breadth of rectangular plot be y m and
and 2018 together = 80 + 48 = 128 length = 15 m
Number of tigers in National Park C in 1998 ATQ,
and 2018 together = 48 + 32 = 80 30 + y + 3y = 390/5
Required % =
128
× 100 = 160% ⇒ 30 + 4y = 78
80
⇒ 4y = 48 ⇒ y = 12 m
36 9
63. (a); Required Ratio = =
40 10 70. (a); Let the CP be Rs. 100x
64. (e); Number of tigers in National Park E in 2018 Then, MP = Rs. 150x
80
140
= 100 × 80 = 112 SP = 150x × 100 = Rs. 120x
Number of tigers in National Park E in 1998 Profits = Rs. 20x
75 New MP = Rs. 175x
= 100 × 32 = 24 80
New SP = 175x × = Rs. 140x
Number of tigers in National Park E in 1998 100

and 2018 together New Profit = Rs. 40x


20x
= 112 + 24 = 136 Required % = 40x × 100 = 50%

65. (b); Total number of tigers in 2018 71. (d); Let the amount be Rs. x
= 36 + 52 + 32 + 48 = 168 CI at 10% in 2 years = 10 + 10 +
10 × 10
100
Total number of tigers in 1998
= 21%.
= 64 + 40 + 48 + 80 = 232
232 168 ATQ,
Required difference = − x × 21
4 4 = 1050 ⇒ x = Rs. 5000
64 100
= = 16 And,
4
5000 × 8 × T
66. (a); Let the speed of boat in still water be x km/hr = 2000
and that of stream be y km/hr 100
⇒ T = 5 years.
ATQ,
(x + y) – (x – y) = 6 72. (e); Let the monthly salary be Rs. 100 x.
⇒ 2y = 6 ⇒ y = 3 km/hr EMI per month
72 1 1
Downstream stream = (x + y) = = 18 km/hr = 100x – (20x + 80x × + 80x × ) = Rs. 20x
4 4 2
⇒ x = 15 km/hr ATQ,
20x × 12 = 60,000
67. (d); Let the initial quantity of mixture in vessel be
⇒ x = 250
x lit
Monthly Salary = Rs. 25,000
ATQ,
5
x× −10
14 1 73. (b); ATQ,
9 =2
x× −18+2
14 4x + x – 9.75 = 442
5x−140
⇒ 9x−224 = 2
1 5x = 451.75
x = Rs. 90
⇒ 10x – 280 = 9x – 224
⇒ x = 56 lit 74. (a); Let the investment of B be Rs. x
∴ investment of A = Rs 2x
68. (b); Weight of new student
Ratio of profit,
= 6 × (25.8 + 3.9) – 5 × 25.8 ≈ 49 kg
A : B : C
69. (c); 12 × 2x : 12 × x : 8 × y
ATQ,
24x = 8y
y = 3x
12 × x
∴ Required percentage = 8 × 3x × 100
= 50%

28 Adda247 | No. 1 APP for Banking & SSC Preparation


Website: bankersadda.com | sscadda.com | store.adda247.com | Email: contact@bankersadda.com
75. (b); Let present age of Ishu & Ahana be x year & y 5
78. (e); Speed of train in m/s. = 72 × 18 = 20 m/s
year respectively Let length of train be x m
∴ ATQ, ATQ,
x+8 5 524 + x
= = 20
y+6 6 55
6x + 48 = 5y + 30 x = 1100 – 524 = 576m
6x – 5y = – 18 … (i)
79. (b); Lets efficiency of A is x unit/day and B’s
x + 10 = y + 6
efficiency is 3x unit/day
x–y=–4 … (ii)
So, B work for 19 days and A work for 18 days
∴ x = 2 years ATQ—
∴ present age of Ishu is 2 years. Total work = 19 × 3x + 18 × x = 75x
20 20 75x
76. (c); quired difference = 100 (P + 5000) − 100 × P Efficiency of C = 50

= 1000 = 1.5x unit/day


75x
(A + C) together = = 30 days
77. (d); Let diameter of base be 2x cm & height of (x+1.5x)

cylinder be 3x cm 80. (d); One day work of women = half of work done by
2x
∴ radius = = x cm men in one day
2
We know, Let efficiency of one women = w unit/day
Man’s efficiency = 2w unit/day
Volume of cylinder = πr 2 h (r→ radius, h →
Total work = (7 × 2w + 6 × w) × 8 =160w unit
height)
8 men and 4 women start work for 3 days
ATQ,
Total work done = (8 × 2w + 4 × w) × 3
πr 2 h = 3234
= 60w
22
× x 2 × 3x = 3234 4 women replace 4 man
7
= (4 × 2w + 8 × w) =16w
x = 7cm 100w
Radius = 7cm Days required = = 6.25 days
16w

29 Adda247 | No. 1 APP for Banking & SSC Preparation


Website: bankersadda.com | sscadda.com | store.adda247.com | Email: contact@bankersadda.com
IBPS RRB PO Prelims 2017| Memory Based Paper |
For Practice
REASONING ABILITY
1. What should come in place of question mark (?) in 7. Which of the following boxes is kept at the top?
the following series based on the above (a) B (b) A (c) D
arrangement? (d) E (e) None of these
ZN XD UG QK ? 8. Choose the odd one out?
(a) LK (b) LO (a) B (b) G (c) A
(c) LP (d) KP (d) D (e) E
(e) Other than the given options
9. Which of the following boxes is kept between F and
2. How many such pair of numbers are there in the A?
given number “46579739” (Both backward and (a) B (b) G
forward) same as far as according to numeric series? (c) C (d) H
(a) One (b) Two (e) None as box F is immediately above box A
(c) Three (d) More than three
(e) None of these. 10. How many boxes are there between C and A
(a) Less than 2 (b) 4 (c) 5
3. If it is possible to make only one meaningful word (d) 6 (e) None of these
with the 1st ,2nd , 4th and 7th letters of the word
‘ECUADOR’ which would be the second letter of the Directions (11-15): In these questions, relationships
word from the right? If more than one such word can between different elements are shown in the statements.
These statements are followed by two conclusions. Give
be formed give ‘Y’ as the answer. If no such word can
answer
be formed, give ‘Z’ as your answer.
(a) if only conclusion I follows
(a) Y (b) E (c) I
(b) if only conclusion II follows
(d) Z (e) M
(c) if either conclusion I or conclusion II follows
4. If 1 is subtracted from each odd number and 2 is (d) if neither conclusion I nor conclusion II follows
added to each even in the number 9436527, then (e) if both conclusions I and II follow
how many digits will appear twice in the new 11. Statement: R ≥ S ≥ T > U > X; T < V < W
number thus formed? Conclusions: I. R > X II. X < W
(a) Only 8 (b) Only 8 and 6 (c) 8, 6 and 4
(d) 2, 4 and 6 (e) None of these 12. Statement: E = F < G < H; G ≥ I
Conclusions: I. H > I II. E > I
5. How many letter will be remain the same position in
the word ‘MONSTER’ when they arranged in the 13. Statement: A > B > F > C; D > E > C
ascending order from left to right? Conclusions: I. C < A II. B > D
(a) One (b) Two (c) Three 14. Statement: K ≤ L ≤ M = N; P ≥ O ≥ N
(d) More than Three (e) None Conclusions: I. K < P II. K = P
Directions (6-10): Read the following information 15. Statement: D < E < F < G; K > F
carefully and answer the following questions. Conclusions: I. K ≤ G II. K > D
Eight boxes A, B, C, D, E, F, G and H are place one above
the other in any particular order. Box no. 1 is at the Directions (16-20): Read the following information
carefully and answer the following questions.
bottom and box no. 8 is at the top. Three boxes are placed
Seven persons A, B, C, D, E, F and G were born on different
between A and B. Box H is placed immediately below A.
months viz. January, February, March, April, June, August
There are two boxes between H and G. There are as many
and October of the same year, but not necessarily in the
boxes between C and D as between H and B. Box C is kept
same order.
above D. Box E is kept immediately below box D. Three
Only three persons were born before E and D is not one
boxes are there between E and F.
of them. F was not born immediately after E. B was born
6. How many boxes are there above box D? after F. A was born immediately before the month in
(a) 4 (b) 3 (c) 6 which G was born. Only two persons were born between
(d) 2 (e) None of these G and F.

30 Adda247 | No. 1 APP for Banking & SSC Preparation


Website: bankersadda.com | sscadda.com | store.adda247.com | Email: contact@bankersadda.com
16. How many persons were born between C and E? order) and all of them are facing North. Therefore in the
(a) Three (b) Two (c) Four given seating arrangement each member seated in a row
(d) Five (e) None of these faces another member of the other row. Q sits fourth to
the left of A. The one facing A sits third to the left of
17. Who amongst the following is the oldest?
S. Only one person sits between S and E. E does not sit at
(a) A (b) C (c) E
any of the extreme ends of the line The one facing U sits
(d) B (e) F
second to the right of B. U does not sit at any of the
18. Who amongst the following was born between the extreme ends of the line. Only two people sit between B
months in which A and D were born? and Y. The one facing B sits second to the left of Z. F is not
(a) E (b) G (c) C an immediate neighbour of U. P is not immediate
(d) B (e) Both E and G neighbour of Q.
19. How many persons were born after D? 26. Which of the following groups of people represents
(a) One (b) Three (c) Four the people sitting at extreme ends of both the rows?
(d) Two (e) None of these (a) Q, Y, Z, R (b) F, Y, F, B (c) S, Y, Z, R
(d) Q, F, Z, B (e) Q, Y, Z, S
20. Who amongst the following is the person who was
born in the month which has less than 30 days? 27. Who amongst the following faces, F?
(a) F (b) B (c) G (a) Q (b) P (c) A
(d) C (e) A (d) X (e) B

Directions (21-25): Study the following information 28. Which of the following is true with respect to the
given information?
carefully and answer the given questions:
(a) B faces one of the immediate neighbours of Z.
In a certain code language (b) F sits exactly between R and E.
‘card win team time’ is written as ‘la ta ja sa’ (c) None of the given options is true
‘fight game play card’ is written as ‘ja pa ra da’ (d) A is an immediate neighbour of B
‘in win team fight’ is written as ‘da ta fa la’. (e) A faces U.
21. What is the code for ‘time’? 29. Which of the following is true regarding X?
(a) sa (b) da (c) ja (a) B sits second to the right of X.
(d) la (e) None of these (b) F is an immediate neighbor of the person who
22. ‘card fight in’ can be coded as? faces X
(c) Both P and Y are immediate neighbours of X
(a) sa ja ra (b) fa ja da
(d) Only one person sits between X and A
(c) da ra ta (d) Can’t be determined
(e) None of the given options is true
(e) None of these
30. Who amongst the following sits second to the right
23. What is the code for ‘game’? of the person who faces P?
(a) ra (a) F (b) U (c) R
(b) pa (d) E (e) S
(c) Either ra or pa
(d) da Directions (31-35): Study the following information
(e) None of these carefully and answer the questions given below:
Eight friends M, N, O, P, Q, R, S and T are sitting around a
24. Which of the following is the code for ‘in’?
circular table with equal distance between them but not
(a) ta (b) da (c) la necessarily in the same order. Some of them are facing
(d) fa (e) None of these the centre with some face outside (i.e. opposite to
25. If ‘game in risk’ is coded as ‘Pa fa xa’ than what will centre).
be the code for ‘risk card fight’? O sits second to the right of R, R faces the centre. Only
(a) Ja sa da (b) ja da ra (c) sa da fa two people sit between O and N (either form O’s right or
(d) xa ja da (e) None of these O’s left). S sits second to the right of O. T sits to the
immediate right of N. S and N face opposite direction (i.e.
Directions (26-30): Study the following information to if N faces the centre then S faces outside and vice versa).
answer the given questions Immediate neighbor of S face the same direction (i.e. If
Twelve people are sitting in a two parallel rows one neighbor faces the centre then the other also faces
containing six people each in such a way that there is an the centre and vice-versa) Only three people sit between
equal distance between adjacent persons. In row 1 – A, B, P and Q. Neither P nor M is an immediate neighbor of R.
P, Q, X and Y are seated (but not necessarily in the same Q sits second to the right of M. Both T and Q face a
order) and all of them are facing south. In row 2 – E, F, R, direction opposite to that of O (i.e. if O faces the centre
Z, S and U are seated (but not necessarily in the same then both T and Q faces outside and vice-versa).

31 Adda247 | No. 1 APP for Banking & SSC Preparation


Website: bankersadda.com | sscadda.com | store.adda247.com | Email: contact@bankersadda.com
31. Who sits exactly between M and P? 36. Statements: All bags are purses.
(a) N (b) S (c) R No purse is black.
(d) Q (e) None of these
All blacks are covers.
32. How many people in the given arrangement face the Conclusions: I. All bags are covers
centre? II. Some covers are purses.
(a) One (b) Three (c) Five
(d) Four (e) None of these 37. Statements: Some cats are rats.
33. Who sits second to the right of T? Some rats are fishes.
(a) O (b) Q All fishes are birds.
(c) S (d) R Conclusions: I. Some fishes are rats.
(e) Other than the given options
II. All cats being birds is a possibility
34. Four of the following five are alike in a certain way
based on the given seating arrangement and so form 38. Statements: Some flowers are roses.
a group. Which is the one that does not belong to that No rose is red.
group? All red are leaves.
(a) P (b) O (c) T
(d) M (e) Q Conclusions: I. Some flowers are definitely not
red.
35. What is P’s position with respect to R?
II. Some leaves are definitely not
(a) Second to the left (b) Third to the right
(c) Third to the left (d) Sixth to the right roses.
(e) Second to the right
39. Statements: All cards are sheets.
Directions (36–40): In each question below are given All files are cards.
some statements followed by two conclusions numbered
Some sheets are papers.
I and II. You have to take the given statements to be true
even if they seem to be at variance with commonly Conclusions: I. All files being papers is a
known facts. Read all the conclusions and then decide possibility.
which of the given conclusions logically follows from the II. All files are not sheets.
given statements, disregarding commonly known facts.
Give answer 40. Statements: Some flowers are roses.
(a) If only conclusion I follows. No rose is red.
(b) If only conclusion II follows.
All red are leaves.
(c) If either conclusion I or II follows.
(d) If neither conclusion I nor II follows. Conclusions: I. Some flowers are not leaves.
(e) If both conclusions I and II follow. II. No leave is a red.

QUANTITATIVE APTITUDE

Directions (41-45): What should come in place of the 44. 6, 4, 5, 11, 39, ?
question mark (?) in following number series problems? (a)159 (b) 169 (c) 189
(d)198 (e) None of these
41. 190, 94, 46, 22, ? , 4 45. 89, 88, 85, 78, 63, ?
(a) 12 (b) 14 (c) 10 (a) 30 (b) 34 (c) 36
(d) 8 (e) None of these (d) 32 (e) None of these
46. There are 3 consecutive odd numbers and 3
42. 5, 28, 47, 64, 77, ? consecutive even numbers. The smallest even
(a) 84 (b) 86 (c) 89 number is 9 more than largest odd number. If the
(d) 88 (e) None of these square of average of all the 3 given odd number is
507 less than the square of the average of all the 3
43. 7, 4, 5, 12, 52, ? given even number, what is the smallest odd
(a) 424 (b) 428 (c) 318 number.
(a) 11 (b) 13 (c) 17
(d) 440 (e) None of these (d) 19 (e) 9

32 Adda247 | No. 1 APP for Banking & SSC Preparation


Website: bankersadda.com | sscadda.com | store.adda247.com | Email: contact@bankersadda.com
47. A can complete a task in 15 days B is 50% more 49. A started a business with a initial investment of Rs.
efficient than A. Both A and B started working 1200. ‘X’ month after the start of business, B joined A
together on the task and after few days B left task and with on initial investment of Rs. 1500. If total profit
1
A finished the remaining 3 of the given work. For how was 1950 at the end of year and B’s share of profit
many days A and B worked together. was 750. Find ‘X’
(a) 3 (b) 5 (c) 4 (a) 5 month (b) 6 month (c) 7 month
(d) 6 (e) 2 (d) 8 month (e) 9 month
48. A boat can travel 9.6 km downstream in 36 min. If
50. Ratio between curved surface area and total surface
speed of the water current is 10% of the speed of the
boat in downstream. How much time will boat take area of a circular cylinder is 3 : 5. If curved surface
to travel 19.2 km upstream. area is 1848 cm3 then what is the height of cylinder.
(a) 2 hours (b) 3 hours (c) 1.25 hours (a) 28 (b) 14 (c) 17
(d) 1.5 hours (e) 1 hour (d) 21 (e) 7

Directions (51-55): Given below is the pie chart which shows the percentage distribution of a book ‘XYZ’ publishes in 5
different stores.

Total books = 550

E, 22% A, 18%

B, 12%

D, 32% C, 16%

51. If number of female who bought the books in store E 55. What is the difference between average of book sold
are 21 more than number of males who bought by store A and E together and average books sold by
books from same store then find the number of
store C and D together?
females who bought book in store E.
(a) 33 (b) 11 (c) 22
(a) 75 (b) 78 (c) 71
(d) 68 (e) 73 (d) 44 (e) 20

52. Find the central angle for the book D. Directions (56-60): In each of these questions, two
(a) 117.5° (b) 115.2° (c) 112.8° equations (I) and (II) are given. You have to solve both
(d) 108.5° (e) 118.8°
the equations and give answer
53. If total books of another publisher ‘MNP’ is 20% (a) if x>y (b) if x≥y
more than books of ‘XYZ’ publisher then what will be (c) if x<y (d) if x ≤y
total books sold by store A and B for publisher ‘MNP’.
(e) if x = y or no relationship can be established.
Percentage-distribution for different stores for MNP
remains same as for ‘XYZ’ 56. I. x 2 + 9x + 20 = 0 II. y 2 = 16
(a) 200 (b) 178 (c) 181
(d) 186 (e) 198 57. I. x 2 − 7x + 12 = 0 II. 3y 2 − 11y + 10=0
54. What is the ratio of total books sold by store A and C 58. I. x 2 − 8x + 15 = 0 II. y 2 − 12y + 36 = 0
together to the total books sold by store D and E
together 59. I. 2x 2 + 9x + 7 = 0 II. y 2 + 4y + 4 = 0
(a) 17 : 27 (b) 18 : 29 (c) 21 : 28
(d) 22 : 23 (e) 24 : 29 60. I. 2x 2 + 15x + 28 = 0 II. 2y 2 + 13y + 21=0

33 Adda247 | No. 1 APP for Banking & SSC Preparation


Website: bankersadda.com | sscadda.com | store.adda247.com | Email: contact@bankersadda.com
61. Train A completely crosses train B which is 205 m (a) 20 (b) 17 (c) 23
long in 16 second. If they are travelling in opposite (d) 26 (e) 29
direction and sum of speed of both are 25 m/s. then
find the difference (in meter) between lengths of 64. A bag contains 6 Red, 5 Green and 4 Yellow coloured
both trains. balls. 2 balls are drawn at random after one another
(a) 5 (b) 6 (c) 8 without replacement then what is the probability
(d) 10 (e) 12 that atleat one ball is Green.
2 4 3
62. A trader mixes 14 kg rice of variety A which costs Rs. (a) 3 (b) 5 (c) 8
60/kg with 18 kg of quantity of type B rice. He sells (d)
4
(e)
2
100 7 7
the mixture at Rs. 65/Kg and earns a profit of %.
3
Then what was the cost price of type B rice. 65. Cost price of B is 200 more than cost price of A. B is
(a) 30 (b) 20 (c) 40 sold at 10% profit and A is sold at 40% loss and
(d) 50 (e) 45 selling price of A and B are in the ratio 4 : 11. If A is
sold at 20% loss then what will be selling price of A.
63. Present age of A is 3 years less than present age of B.
Ratio of B’s age 5 year ago and A’s age 4 year hence (a) 320 (b) 400 (c) 240
is 3 : 4 then find present age (in years) of A. (d) 160 (e) 360

Directions (66-70): Read the following table carefully and answer the following questions—
No. of students and % of students passed out of those who appeared are given for two subjects from year 2001 to 2005 in
a college XYZ.

66. Find the average number of students who were 70. Find the average number of students appeared in
failed in Economics in year 2002 and year 2003 Economics from year 2001 to 2004 together?
together? (a) 3090 (b) 3015 (c) 3060
(a) 1435 (b) 1565 (c) 1720 (d) 3075 (e) 3850
(d) 1590 (e) None of these Direction (71-75): What approximate value should
come in place of question mark (?) in the following
67. Number of students failed in Statistics in the year
questions? (Note: You are not expected to calculate the
2003 is what % of the number of students failed in exact value)
Economics in the same year?
(a) 145.75% (b) 150% (c) 156.25% 71. ? % of (5284.89 ÷ 7.08) = 986.01 – 533. 06
(d) 158.25% (e) None of these (a) 42 (b) 39 (c) 74
(d) 65 (e) 60
68. Find the ratio between the total number of students
72. (1041.84 + ?) ÷ 3.02 = 1816.25 ÷ 4.01
appeared in Economics from 2002 to 2004 together (a) 442 (b) 337 (c) 385
and the total number of students appeared in (d) 268 (e) 320
Statistics from year 2003 to 2005 together?
(a) 13 : 14 (b) 14 : 13 (c) 15 : 16 73. 69.3% of 445.12 ÷ 14.06 = 623.08 ÷ ?
(d) 16 : 15 (e) None of these (a) 28 (b) 19 (c) 21
(d) 33 (e) 37
69. Find the difference between the total number of 74. ?2+ 114.09 – 24.06 × 5.14 = 163.19
students passed in Statistics from year 2002 and (a) 7 (b) 13 (c) 11
total number of students failed in Economics from (d) 15 (e) 19
year 2005.
(a) 690 (b) 385 (c) 485 75. 768.16 ÷ 11.87 × √257 – 58.05 = ?
(d) 550 (e) 610 (a) 1033 (b) 1175 (c) 966
(d) 880 (e) 975
34 Adda247 | No. 1 APP for Banking & SSC Preparation
Website: bankersadda.com | sscadda.com | store.adda247.com | Email: contact@bankersadda.com
Directions (76-80): Study the following line graph carefully and answer the following questions.
Number of males and number of females are given. They are visiting a place from Monday to Friday.
Male Female
230

180

130

80
MON TUE WED THUS FRI
76. Find the ratio of the total number of males visited the (a) 30 (b) 60 (c) 40
place on Tuesday and Thursday together to the total (d) 50 (e) None of these
number of females visited the place on Monday and
Friday together? 79. If on Saturday the number of males and number of
(a) 29 : 30 (b) 30 : 29 (c) 25 : 26 females increased by 25% and 20% respectively as
(d) 26 : 25 (e) None of these compared to that on Friday then find the total
77. Total number of males and females together visited number of males and females together visited the
the place on Tuesday are what percent more/less place on Saturday?
than the total number of male and females together (a) 196 (b) 306 (c) 316
visited the place on Thursday ? (d) 206 (e) 216
12 3 3
(a) 26 % (b) 25 % (c) 26 %
13 13 13 80. Total number of males and females visited the place
7
(d) 25 % (e) None of these on Monday and Tuesday together is how much more
13
than the total number of males and females visited
78. Find the difference between the total number of
females visited the place from Monday to the place on Thursday and Friday together?
Wednesday and the total number of males visited the (a) 175 (b) 125 (c) 150
place from Wednesday to Friday? (d) 160 (e)130

Solutions

REASONING ABILITY
1. (c); LP Directions (6-10):
2. (d); Number Box
8 B
7 C
6 G
3. (a); Race, Care 5 F
4. (c); 4 A
3 H
2 D
1 E
5. (a);
6. (c); 7. (a); 8. (e);
9. (e); 10. (e);

35 Adda247 | No. 1 APP for Banking & SSC Preparation


Website: bankersadda.com | sscadda.com | store.adda247.com | Email: contact@bankersadda.com
Directions (11-15): 29. (b); 30. (e);
11. (e); Both conclusion I and II follow. Direction (31-35):
12. (a); Only conclusion I follows.
13. (a); Only conclusion I follows.
14. (c); Either conclusion I or II follows.
15. (b);Only conclusion II follows.
Directions (16-20):
Month Person
January C
February A
March G 31. (b); 32. (b); 33. (c);
April E
34. (b); 35. (c);
June D
August F Directions (36–40):
October B 36. (d);
16. (b); 17. (b); 18. (e);
19. (d); 20. (e);
Directions (21-25):
Word Code 37. (e);
Card ja
Time sa
Win/team la/ta
Fight da
Game/Play pa/ra 38. (e);
In fa
21. (a); 22. (b); 23. (c);
24. (d); 25. (d); 39. (a);
Direction (26-30):

40. (d);

26. (a); 27. (e); 28. (e);

QUANTITATIVE APTITUDE

41. (c); Series is ÷2–1, ÷2–1 43. (a); (7+1) × 0.5 = 4


(22÷2)–1=10
(4+1) × 1 = 5
42. (d);
(5+1) × 2 = 12
(12+1) × 4 =52
Adding prime No. (52 +1) × 8 = 424
77 + 11 = 88

36 Adda247 | No. 1 APP for Banking & SSC Preparation


Website: bankersadda.com | sscadda.com | store.adda247.com | Email: contact@bankersadda.com
44. (c); (6×1)–2 = 4 51. (c); Let male who purchased book from Store E = 𝑥
(4×2)–3 = 5 Then
22
(5×3) –4 = 11 𝑥 + 𝑥 + 21 = 100 × 550
(11×4) –5 = 39 𝑥 = 50
(39×5) –6 = 189 Required number of females = 50 + 21 = 71
45. (d); 52. (b);
18
=
𝑥
5 32
18×32
𝑥= 5
= 18 × 6.4 = 115.2
120
53. (e); Total books of store XYZ= 100 × 550
63 – 31 = 32 = 660
46. (a); Let a consecutive odd numbers Total books sold by store A and B
= 𝑥 – 2, 𝑥 and 𝑥 + 2 = (18% + 12%) of 660 = 198
and consecutive even numbers 54. (a); Required ratio = (18% + 16%) : (32% + 22%)
= 𝑦 – 2, 𝑦, 𝑦 + 2
= 34 : 54 = 17 : 27
So, 𝑦 – 2 = 9 + 𝑥 + 2
𝑦 – 𝑥 = 13 … (i) 55. (c); Required difference
and 1
= [(32% + 16%) − (18% + 22%)]550
(𝑥)2 + 507 = (𝑦)2 2
1
𝑦 2 − 𝑥 2 = 507 = × 8% 𝑜𝑓 550 = 4% 𝑜𝑓 550 = 22
2
(𝑥 + 𝑦)(𝑦 − 𝑥) = 507
(𝑥 + 𝑦) =
507
⇒ 𝑥 + 𝑦 = 39 … (𝑖) 56. (d);I 𝑥 2 + 5𝑥 + 4𝑥 + 20 = 0
13
Solving (i) and (ii) 𝑦 = 26 and 𝑥 = 13 𝑥(𝑥 + 5) + 4(𝑥 + 5) = 0
so smallest odd numbers = 𝑥 – 2 = 13 – 2 = 11 (𝑥 + 4)(𝑥 + 5) = 0
𝑥 = −4, −5
47. (c); A complete work in 15 days.
II. 𝑦 2 = 16
B will complete work in 10 days.
𝑦 = ±4
They together will complete whole work
15×10 ∴𝑥≤𝑦
= 25 = 6 𝑑𝑎𝑦𝑠
A and B together worked for = 6 × 2⁄3= 4 days 57. (a); I. 𝑥 2 − 7𝑥 + 12 = 0
9.6 𝑥 2 − 4𝑥 − 3𝑥 + 12 = 0
48. (d);𝑆𝑝𝑒𝑒𝑑 𝑜𝑓 𝑑𝑜𝑤𝑛𝑠𝑡𝑟𝑒𝑎𝑚 = 𝑘𝑚⁄𝑚𝑖𝑛
36 𝑥(𝑥 − 4) − 3(𝑥 − 4) = 0
= 16 𝑘𝑚/ℎ𝑟 (𝑥 − 3)(𝑥 − 4) = 0
Speed of current = 1.6 km/hr
𝑥 = 3, 4
Let speed of man in still water = 𝑥
So, 𝑥 = 16 – 1.6 = 14.4 km/hr II. 3𝑦 2 − 11𝑦 + 10 = 0
19.2
𝑅𝑒𝑞𝑢𝑖𝑟𝑒𝑑 𝑡𝑖𝑚𝑒 𝑖𝑛 𝑢𝑝𝑠𝑡𝑟𝑒𝑎𝑚 = 14.4−1.6 3𝑦 2 − 6𝑦 − 5𝑦 + 10 = 0
3𝑦(𝑦 − 2) − 5(𝑦 − 2) = 0
= 1.5 ℎ𝑜𝑢𝑟𝑠
(3𝑦 − 5)(𝑦 − 2) = 0
49. (b);Ratio of profit of A and B = 1200 : 750 5
= 24 : 15 = 8 : 5 𝑦 = 2, 3
So, ∴𝑥>𝑦
1200×12 8
=
1500×𝑦 5 58. (c); I. 𝑥 2 − 8𝑥 + 15 = 0
𝑦 = 6 months 𝑥 2 − 3𝑥 − 5𝑥 + 15 = 0
x = 6 month 𝑥(𝑥 − 3) − 5(𝑥 − 3) = 0
2𝜋𝑟ℎ 3 (𝑥 − 3)(𝑥 − 5) = 0
50. (d);2𝜋𝑟(𝑟+ℎ) = 5
𝑥 = 3,5
5h = 3r + 3h II. 𝑦 2 − 12𝑦 + 36 = 0
2h = 3r
𝑦 2 − 6𝑦 − 6𝑦 + 36 = 0
and
𝑦(𝑦 − 6) − 6(𝑦 − 6) = 0
2πrh = 1848
22 2 (𝑦 − 6)(𝑦 − 6) = 0
2 × 7 × 3 ℎ × ℎ = 1848
𝑦=6
ℎ = 21 ∴𝑥<𝑦

37 Adda247 | No. 1 APP for Banking & SSC Preparation


Website: bankersadda.com | sscadda.com | store.adda247.com | Email: contact@bankersadda.com
59. (e); I. 2𝑥 2 + 9𝑥 + 7 = 0 65. (a); Let C.P. of A = 𝑥
2𝑥 2 + 7𝑥 + 2𝑥 + 7 = 0 So C.P. of B = 200 + 𝑥
𝑥(2𝑥 + 7) + 1(2𝑥 + 7) = 0 According to question
110
(𝑥 + 1)(2𝑥 + 7) = 0 100
(𝑥+200) 11 𝑥+200 1
7 60 = ⇒ =
𝑥 = −1, − 100
𝑥 4 6𝑥 4
2
II. 𝑦 + 4𝑦 + 4 = 0
2 𝑥 = 400
𝑦 2 + 2𝑦 + 2𝑦 + 4 = 0 If it is sold at 20% loss then selling price
80
𝑦(𝑦 + 2) + 2(𝑦 + 2) = 0 = × 400 = 320
100
(𝑦 + 2)(𝑦 + 2) = 0
𝑦 = −2, −2 66. (b);No. of students failed in Economics in year 2002
(100−45)
∴ No relation. = × 3800 = 2090
100
60. (d);I. 2
2𝑥 + 15𝑥 + 28 = 0 No. of students failed in Economics in year 2003
(100−60)
2𝑥 2 + 8𝑥 + 7𝑥 + 28 = 0 = × 2600 = 1040
100
2𝑥(𝑥 + 4) + 7(𝑥 + 4) = 0 2090+1040
Required average = = 1565
(2𝑥 + 7)(𝑥 + 4) = 0 2
55×38+40×26
𝑥 = (− ) , −4
7 Short trick = = 1565
2
2
II. 2𝑦 2 + 13𝑦 + 21 = 0 67. (c); No. of students failed in Statistics in year 2003
2𝑦 2 + 7𝑦 + 6𝑦 + 21 = 0 =
100−35
× 2500 = 1625
𝑦(2𝑦 + 7) + 3(2𝑦 + 7) = 0 100

(𝑦 + 3)(2𝑦 + 7) = 0 No. of students failed in Economics in year 2003


100−60
−7
𝑦 = −3, 2 = × 2600 = 1040
100
1625
𝑥≤𝑦 Required % = × 100 = 156.25%
1040
65×25
61. (d);In 16 second distance covered by both Short trick = 40×26 × 100 = 156.25%
= 16 × 25 = 400 m
68. (d);Total no. of students appeared in Economics
So length of A = 400 – 205 = 195
from 2002 to 2004
Required difference = 10 m
= 3800 + 2600 + 4800 = 11200
62. (c); Let cost price of mixture = 𝑦 Total no. of students appeared in Statistics from
4
𝑆𝑜, 𝑦 = 65 2003 to 2005
3
𝑦 = 48.75 = 2500 + 3200 + 4800 = 10500
From mixture and allegation Required ratio = 11,200 : 10,500 = 16 : 15
69. (b);Total no. of students passed in Statistics in year
55
2002 = 100 × 2700 = 1485
Total no. of students failed in Economics in year
50
2005 = 100 × 2200 = 1100
7 48.75−𝑥 Required difference = 1485 – 1100 = 385
= 60−48.75
9 Short trick = 55 × 27 – 50 × 22 = 385
78.75 = 438.75 − 9𝑥
360 = 9𝑥 70. (e); Average no. of students appeared in Economics
𝑥 = 40 Rs./kg from year 2001 to 2004 together
4200+3800+2600+4800 15400
= = 4 = 3850
63. (a); Let B’s age = 𝑥 4
So A’s age = 𝑥 − 3 ?
𝑥−5 3 71. (e); 100 × 750 = 450 ⇒ ? ≈ 60
𝑥+1
=4
(1042+?)
𝑥 = 23 72. (e); 3.02
= 454 ⇒ ? = 320
A’s age = 23−3 = 20 𝑦𝑒𝑎𝑟𝑠
310 625
73. (a); = ⇒ ? ≈ 28
64. (d);Probability that no ball is green 14 ?
10𝐶 × 9𝐶 90 3
1 1
= 15×14 = 7 74. (b);?2 = 170 ⇒ ? ≈ 13
15×14
3 4
Required probability = 1 − 7 = 7 75. (c); ≈ 64 × 16 – 58 ≈ 966

38 Adda247 | No. 1 APP for Banking & SSC Preparation


Website: bankersadda.com | sscadda.com | store.adda247.com | Email: contact@bankersadda.com
76. (a); Total no. of males visited on Tuesday and 79. (b);On Saturday —
Thursday = 140 + 150 = 290 Total no. of males visited the place
Total no. of females visited on Monday and =
125
× 120 = 150
Friday = 170 + 130 = 300 100

Required ratio = 290: 300 = 29: 30 Total no. of females visited the place
120
= × 130 = 156
77. (a); Total no. of males and females together on 100
Tuesday = 140 + 190 = 330 Required males and females
Total no. of males and females together on = 150 + 156 = 306
Thursday = 150 + 110 = 260
330−260 12 80. (c); Total males and females visited the place on
Required % = 260 × 100 = 26 13 %
Monday and Tuesday together
78. (d);Total no. of females visited from Monday to = 160 + 140 + 170 + 190 = 660
Wednesday = 170 + 190 + 140 = 500 Total males and females visited the place on
Total no. of males visited from Wednesday to Thursday and Friday together
Friday = 180 + 150 + 120 = 450 = 150 + 120 + 110 + 130 = 510
Required difference = 500 – 450 = 50 Required no. of persons = 660 – 510 = 150

39 Adda247 | No. 1 APP for Banking & SSC Preparation


Website: bankersadda.com | sscadda.com | store.adda247.com | Email: contact@bankersadda.com
IBPS RRB Clerk Prelims 2019 | Memory Based Paper |
For Practice

REASONING ABILITY
Directions (1-4): In each of the question, relationships 8. How many floors are there above the floor on which
between some elements are shown in the statements. G lives?
These statements are followed by conclusions numbered (a) One (b) Two (c) Three
I and II. Read the statements and give the answer. (d) More than Four (e) Four
(a) If only conclusion I follows.
(b) If only conclusion II follows. 9. Who lives immediately below A?
(c) If either conclusion I or II follows. (a) D (b) E (c) F
(d) If neither conclusion I nor II follows. (d) C (e) None of these
(e) If both conclusions I and II follow.
Directions (10-14): Study the following sequence and
1. Statements: P < R ≤ M = L > O ≤ V > Y answer the given questions.
Conclusions: I. L > P II. O > R
A@3%4 ENM$8&6LDS♠ 986QY Z17%RO
2. Statements: A ≥ B > D = F < E ≤ C
G⧫2IB2U&
Conclusions: I. B > E II. D < C
3. Statements: A = E ≥ D ≥ C < F ≤ B 10. Which of the following element is twelfth to the left
Conclusions: I. C < A II. A = C of the twentieth element from the left end of the
given arrangement?
4. Statements: F ≥ N = O > P ≤ K > T (a) 6 (b) & (c) M
Conclusions: I. K < F II. N < K
(d) $ (e) None of these
Direction (5-9): Study the following information
11. If all the symbols are dropped from the series, which
carefully and answer the question given below-
element will be fourth to the right of the one which
Seven people viz. A, B, C, D, E, F and G lives in a building is twelfth from the right end?
on seven different floors such as ground floor is (a) 9 (b) O (c) R
numbered 1, the floor just above is numbered 2 and so (d) 7 (e) None of these
on till top floor numbered as seven but not necessarily in
the same order. 12. How many such numbers are there in the given
There are less than three floors above A. Only one person series which are immediately preceded by a symbol
lives between C and A. G lives immediately below D. D and followed by a letter?
lives on an even number floor. B lives immediately above (a) None (b) One (c) Two
A. F lives above E. F does not lives on the 5th floor. F does (d) Three (e) Four
not lives on an even number floor.
13. Four of the following five are alike in a certain way
5. Four of the following five belongs to a group find the
and forms a group find the one that does not belongs
one that does not belongs to that group?
to that group?
(a) CD (b) EC (c) FB
(a) 3E% (b) R⧫2 (c) M&$
(d) AB (e) GC
(d) D9S (e) Y7Z
6. Who among the following lives on the top floor?
(a) E (b) B (c) F 14. What should come in place of question mark (?) in
(d) D (e) None of these the following series based on the above
arrangement?
7. Number of persons lives above F is same as the
34% N$M 6DL 8Q6 ?
number of persons below __ ?
(a) B (b) D (c) C (a) %OR (b) 7Z% (c) O%R
(d) G (e) None of these (d) R%O (e) R%7

40 Adda247 | No. 1 APP for Banking & SSC Preparation


Website: bankersadda.com | sscadda.com | store.adda247.com | Email: contact@bankersadda.com
Direction (15-19): Study the following information 21. Statements: All bamboos are sticks
carefully and answer the question given below- No bamboos is a fish.
Conclusions: I. Some sticks are fish.
Seven people viz. P, Q, R, S, T, U and V are sitting around
II. No sticks are fish.
a circular table having equal distance between them. All
(a) Both I and II follow
of them are facing inside.
(b) Either I or II follows
P sits immediate right of Q. Only one person sits between
(c) Only II follows.
P and S (either from left or right). U sits third to the right
(d) Only I follows.
of S. T is an immediate neighbor of U. R sits second to the
(e) Neither I nor II follows
left of V.
22. Statements: Only a few wells are mats.
15. If all the persons are arranged according to the
All pillows are mats.
alphabetical order in anticlockwise direction
Conclusions: I. At least some pillows are wells.
starting from P, then how many persons position will
II. All wells can never be pillow.
remain unchanged (except P)?
(a) Both I and II follow
(a) Three (b) One (c) Two
(b) Either I or II follows
(d) None (e) None of these (c) Only II follows.
16. How many persons sits between Q and U, if counted (d) Only I follow.
from the left of Q? (e) Neither I nor II follows
(a) One (b) Two (c) Three Direction (23-27): Study the following information
(d) None (e) None of these carefully and answer the question given below-
17. Who sits second to the right of T? There are ten persons are sitting in two parallel row such
(a) P (b) Q (c) R that five persons are sitting in each row. A, B, C, D and E
(d) S (e) None of these are sitting in row 1 and faces north and M, N, O, P and R
18. Four of the following five belongs to a group find the are sitting in row 2 and faces south such that persons
one that does not belongs to that group? sitting in row 1 faces the persons sitting row 2.
(a) VQ (b) PV (c) RT B sits immediate right of A. Neither A nor B sits at the
(d) SU (e) TQ extreme ends. Two person sits between P and N. B faces
19. Who among the following sits second to the left of the the one who sits on the immediate left of P. M sits on the
one who sits 4th to the right of V? immediate right of R. C sits at the end of the row. D sits
(a) U (b) T (c) R on the left of E. D does not face R.
(d) S (e) None of these 23. Four of the following five belongs to a group find the
Directions (20-22): In each of the questions below are one that does not belongs to that group?
given some statements followed by two conclusions. You (a) O (b) C (c) D
have to take the given statements to be true even if they (d) P (e) N
seem to be at variance with commonly known facts. Read 24. Who among the following sits second to the left of the
all the conclusions and then decide which of the given one who faces B?
conclusions logically follows from the given statements, (a) R (b) N (c) O
disregarding commonly known facts. Give answer (d) M (e) None of these
20. Statements: Only a few lamps are bottles. 25. How many persons sits on the left of N?
No bottle is ship. (a) One (b) Two (c) No one
Conclusions I. Some ships are definitely not (d) Three (e) None of these
lamps.
II. All lamps can never be ships. 26. How many persons sits between D and C?
(a) Both I and II follow (a) One (b) Two (c) Three
(b) Either I or II follows (d) No One (e) Can’t be determined
(c) Only II follows. 27. Who among the following faces A?
(d) Only I follow. (a) M (b) N (c) O
(e) Neither I nor II follows (d) R (e) None of these

41 Adda247 | No. 1 APP for Banking & SSC Preparation


Website: bankersadda.com | sscadda.com | store.adda247.com | Email: contact@bankersadda.com
28. IF ‘He will Say’ is coded as ’1 3 9’ and ‘Say To Him’ is Direction (34-36): Study the following information
coded as ‘3 5 2’ and ‘He May Do’ is coded as ‘8 7 9’ carefully and answer the question given below-
then what will be the code of ‘will’? Uncertain number of persons are sitting in a linear row
(a) 3 (b) 1 (c) 9 facing north. B sits fifth to the left of E. Two persons sits
(d) 8 (e) Can’t be determined between B and D. D sits second position from one of the
extreme end. Five persons sits between S and E. S is not
29. How many pairs of letters are there in the word an immediate neighbor of B. As many as persons sits
“MINUTE” each of which have as many letters between E and S as between S and C. As many as persons
between them in the word as they have between sits between D and B as between B and F. C sits third
them in the English alphabetical series? position from the extreme end.
34. How many persons are sitting in the row?
(a) Three (b) One (c) Two (a) 21 (b) 23 (c) 24
(d) More than three (e) None (d) 26 (e) Can’t be determined
Direction (30-33): Study the following information 35. If G sits 2nd to the right of S, then what is the position
carefully and answer the question given below- of G from right end?
(a) 7 (b) 9 (c) 8
There are six persons i.e. A, B, C, D, E and F who all are of (d) 6 (e) None of these
different weight. No two persons have same weight. Only
36. What is the position of F with respect to E?
two persons are lighter than A. B is heavier than A but (a) Second to the right
lighter than C and D. F is heavier than E but lighter than (b) Second to the left
D. D is not the heaviest. The weight of 2nd heaviest (c) Third to the left
person is 115 kg and the weight of lightest is 56 kg. (d) Fifth to the Right
(e) None of these
30. How many persons are heavier than F?
Direction (37-39): Study the following information
(a) One (b) Two (c) Three
carefully and answer the question given below-
(d) Four (e) None of these Point C is 12m west of point A. Point B is 18m north of
31. If the sum of weight of E and A is 131 and the sum of point A. Point E is 9m south of point D. Point F is 14m
west of point E. Point D is 28m east of point B. F is 13m
weight of D and B is 213, then what is the sum of
south of point G.
weight of A and B?
(a) 172 (b) 173 (c) 174 37. Four of the following five belongs to a group find the
one that does not belongs to that group?
(d) 175 (e) None of these
(a) CB (b) AD (c) AE
32. Which among the following person is the 2nd (d) BG (e) FB
heaviest? 38. In which direction point A with respect to point G?
(a) A (b) B (c) C (a) North-west (b) South-east (c) South-west
(d) D (e) None of these (d) North (e) North-east
39. If Point S is 4m south of point G then what is the
33. Which of the following statement is true?
distance between point B and point S?
I. Only two persons are heavier than B. (a) 28m (b) 9m (c) 8m
II. Sum of weight of D and E is 171 Kg. (d) 14m (e) None of these
III. Weight of E is 58 Kg.
40. Find the odd one out?
(a) Only II (b) Both I and II (a) PSRQ (b) MONL (c) ADCB
(c) Both III and II (d) All are True (e) Only III (d) VYXW (e) ILKJ

Quantitative Aptitude

41. 1, 2, 5, 16, 65, 328, 1957 43. 84, 96, 83, 95, 80, 94, 81
(a) 5 (b) 328 (c) 16 (a) 95 (b) 81 (c) 83
(d)1957 (e) 65 (d) 80 (e) 84
42. 4, 11, 25, 46, 74, 129, 151 44. 3, 5, 8, 17, 33, 58, 94
(a) 129 (b) 11 (c) 151 (a) 8 (b) 94 (c) 58
(d) 4 (e) 46 (d) 3 (e) 5

42 Adda247 | No. 1 APP for Banking & SSC Preparation


Website: bankersadda.com | sscadda.com | store.adda247.com | Email: contact@bankersadda.com
45. A boat covers 36 km in upstream in 2 hours and 66 48. In 64 liter of pure milk, 20 liter of water is mixed
1
km in downstream in 3 hours. Find the speed of and then th of the mixture is taken out. When x liter
4
boat in still water? of water is added again then ratio of water to that of
(a) 21km/h (b) 19 km/h (c) 20.5 km/h the milk becomes 1:2. Find value of x?
(d) 20 km/h (e) 19.5 km/h (a) 10 liter (b) 8 liter (c) 12 liter
(d) 6 liter (e) 9 liter
46. Two inlet taps A and B can fill a tank in 36 minutes
and 60 minutes respectively. Find the time taken by 49. Total cost of x pens and (x-2) pencils is Rs 424. If
1
both the taps together to fill 6 th of the tank? one pencil and one pen costs Rs 4 and Rs 20
3 1 respectively then find x?
(a) 3 minutes (b) 3 4 minutes (c)32 minutes
1 1
(a) 16 (b) 18 (c) 15
(d) 3 3 minutes (e) 2 3 minutes (d) 20 (e) 21
47. If circumference of first circle is 132 cm and 50. A is 6 years younger than B and ratio of present age
circumference of second circle is 110 cm then find of B to C is 12:5. If ratio of present age of A to C is
the difference between area of both the circle? 2:1 then find present age of B?
(a) 423.5 cm2 (b) 412.5 cm2 (c) 420 cm2 (a) 20 years (b) 30 years (c) 24 years
(d) 422.4 cm 2 (e) 419.8 cm2 (d) 18 years (e) None of these

Directions (51-55): Given bar graph shows the data of two types of school buses X and Y for three schools A, B and C.
Study the chart carefully and answer the following questions.

Bus Type X Bus Type Y


60

50
Number of buses

40

30

20

10

0
A B C
Schools

51. What is the average number of X type buses from 54. What is the difference of average number of all buses
from school A and average number of all buses from
school B and school C together?
school C?
(a) 40 (b) 70 (c) 30 (a) 16 (b) 4 (c) 8
(d) 59 (e) 54 (d) 24 (e) 12
55. Which school has maximum number of buses?
52. X type buses from school A are how much more than (a) School B
that of X type buses from school B? (b) School C
(c) School A & School C
(a) 555/19% (b) 25% (c) 55/9% (d) School A & School B
(d) 455/6% (e) 331/3% (e) School A
Directions (56-60): Given below are two equations in
53. What is the average number of all the buses from each question, which you have to solve and give answer
school B? (a) if𝑥 > 𝑦
(b) if𝑥 ≥ 𝑦
(a) 43 (b) 39 (c) 31 (c) if𝑦 > 𝑥
(d) 54 (e) 59 (d) if𝑦 ≥ 𝑥
(e) if𝑥 = 𝑦 or no relation can be established

43 Adda247 | No. 1 APP for Banking & SSC Preparation


Website: bankersadda.com | sscadda.com | store.adda247.com | Email: contact@bankersadda.com
56. I.2𝑥 2 − 5𝑥 + 2 = 0 II.2𝑦 2 − 9𝑦 + 7 = 0 68. If ratio of salary of A to that of B is 1:3 and each
57. I.3𝑥 + 7𝑥 + 4 = 0
2
II. 𝑦 + 9𝑦 + 20 = 0
2 spends 15% of his salary on house rent. Find the
house rent paid by A if remaining amount with A and
58. I.𝑥 − 7𝑥 + 10 = 0
2
I.𝑦 2 − 14𝑦 + 45 = 0 B together is Rs 42500.
59. I. x² – 3x = 4 II. y² + 6y + 8 = 0 (a) Rs 1800 (b) Rs 1845 (c) Rs 1785
60. I. x² – 3x = 10 II. y² + 7y + 10 = 0 (d) Rs 1760 (e) Rs 1875

Directions (61-65): Following are the details of three 69. A started a business by investing Rs. 50,000. After 6
shopkeepers and numbers of items sold by them on three months B joined him by investing Rs. 75,000. After
different days another 6 months C joined with Rs. 1,25,000. What is
the ratio of profit shared after 2 years among A, B and
Shopkeepers Monday Tuesday Wednesday C?
A 160 240 210
(a) 4 : 5 : 6 (b) 8 : 9 : 10 (c) 8 : 9 : 12
B 200 180 320
(d) 4 : 5 : 8 (e) None of these
C 150 330 280
61. Find the ratio of items sold by A and B on Monday to 70. At what rate will a sum of Rs. 1000 amounts to Rs.
items sold by B and C on Wednesday? 1102.50 in 2 years at compound interest?
(a) 5 : 3 (b) 3 : 5 (c) 3 : 4 (a) 6.5% (b) 6% (c) 5%
(d) 4 : 7 (e) 5 : 8 (d) 5.5% (e) None of these

62. Find the average number of items sold by all 3 Directions (71-80): What should come in place of
shopkeepers on Wednesday? question mark (?) in the following questions?
(a) 280 (b) 290 (c) 270 5
71. ?² = 40% of 11 of 352
(d) 250 (e) 260
(a) 12 (b) 16 (c) 6
63. Items sold by A and B together on Tuesday is what (d) 4 (e) 8
percentage of items sold by B and C on Wednesday?
(a) 70% (b) 75% (c) 60% 72. ?2 = (√1444 + √676) ÷ 4
(d) 65% (e) 80% (a) 6 (b) 16 (c) 8
(d) 2 (e) 4
64. Find the difference of number of items sold by B on
Monday and Tuesday together and items sold by A 73. (
?−0.5
)=
120
on Tuesday and Wednesday? 0.2 2

(a) 80 (b) 60 (c) 50 (a) 30 (b) 12.5 (c) 25


(d) 70 (e) 100 (d)17.5 (e) 22.5

65. Find the ratio of items sold by B on all 3 days together 74. 60% 𝑜𝑓 ? −√324 = 222
to the items sold by C on all 3 days? (a) 600 (b) 250 (c) 200
(a) 35 : 38 (b) 38 : 35 (c) 30 : 34 (d) 400 (e) 350
(d) 30 : 38 (e) 35 : 41 23 ×32
75. = √64
66. Marked price of an article is Rs 250 more than cost (90÷?)

price of that article and it is sold at a discount of 15% (a) 15 (b) 12 (c) 10
on marked price. Find the cost price of the article if (d) 11 (e) 16
the profit percent earned is 27.5%? 160
76. √4 ×? =
(a) Rs 600 (b) Rs 550 (c) Rs 500 10
(d) Rs 750 (e) Rs 900 (a) 64 (b) 60 (c) 68
(d) 56 (e) 72
67. In year 2016, ratio of boys to girls in a school is
36:19. And in year 2017, number of boys is increased 77. √5929 + √8464 = (? )2
by 1440 and number of girls is increased by 15%. If (a) 17 (b) 21 (c) 15
in 2017, there were total increase in the number of (d) 13 (e) 11
students is 1725 then find the increased number of 1 1 50
boys in the school. 78. 7 2 − 2 2 = ?
(a) 7240 (b) 5440 (c) 6040 (a) 8 (b) 5 (c) 15
(d) 4440 (e) 5040 (d)12 (e) 10

44 Adda247 | No. 1 APP for Banking & SSC Preparation


Website: bankersadda.com | sscadda.com | store.adda247.com | Email: contact@bankersadda.com
1
79. [(2 × 4) + 4] × 8 =?× 10 80. 80% of (1.5 × 4+? ) = 24
(a) 4.8 (b) 3.6 (c) 2.4 (a) 30 (b) 36 (c) 24
(d) 3.2 (e) 4.2 (d) 28 (e) 42

Solutions

REASONING ABILITY
Directions (1-4): 21. (b);
1. (a); 2. (b) 3. (c)
4. (d);
Direction (5-9): 22. (c)
Floors Person
7 F
6 B
5 A
4 E Direction (23-27):
3 C
2 D
1 G
5. (e); 6. (c); 7. (d);
8. (d); 9. (b); 23. (d); 24. (b); 25. (c);
26. (c); 27. (e); 28. (b);
Directions (10-14):
29. (c);
10. (c); 11. (c); 12. (d);
13. (b); 14. (a) Direction (30-33):
C > D (115kg) > B > A > F > E (56kg)
Direction (15-19):
30. (d); 31. (b); 32. (d);
33. (b);

Direction (34-36):

34. (c); 35. (a) 36. (b)


Direction (37-39):

15. (c); 16. (d); 17. (b);


18. (e); 19. (a);
Directions (20-22):
20. (c);

37. (e); 38. (c) 39. (d)


40 (b);

45 Adda247 | No. 1 APP for Banking & SSC Preparation


Website: bankersadda.com | sscadda.com | store.adda247.com | Email: contact@bankersadda.com
Quantitative Aptitude

41. (b); The wrong no. is 328 49. (b); ATQ


1×1+1=2 20𝑥 + 4 × (𝑥 − 2) = 424
2×2+1=5 24𝑥 = 432 ⇒ 𝑥 = 18
5 × 3 + 1 = 16
16 × 4 + 1 = 65 50. (e); Let present age of B and C be 12x years and
65 × 5 + 1 = 326 5x years respectively.
326 × 6 + 1 = 1957 Then present age of A=10𝑥 years
So, there should be 326 instead of 328 ATQ
42. (a); The wrong no is 129 12𝑥 − 10𝑥 = 6 ⇒ 𝑥 = 3
Present age of B=36 years
51. (a); Average number of X type buses from school
So, there should be 109 instead of 129 36+44
B and school C together = 2
= 40
43. (d); The wrong no. is 80
52. (e); X type buses of school A = 48
X type buses of school B = 36
48−36 1
So, there should be 82 instead of 80 Required value = X 100 = 33 %
36 3
44. (e); The wrong no. is 5 53. (c); Average number of all the buses from school
36+26
B= = 31
2

54. (b); Average number of all the buses from school


So, there should be 4 instead of 5. A=
48+38
= 43
2
45. (d); Upstream speed of boat=18 km/hr Average number of all the buses from school
Downstream speed of boat=22 km/hr 44+34
18+22 C= 2
= 39
Speed of boat in still water= = 20 𝑘𝑚/ℎ
2 Required difference = 43 – 39 = 4
46. (b); Let the capacity of the tank be 180 units (LCM
55. (e); Total buses from school A = 48 + 38 = 86
of 36 and 60)
Total buses from school B = 36 + 26 = 62
Efficiency of tap A=5 units/ minute
Efficiency of tap B=3 units/minute Total buses from school C = 44 + 34 = 78
1
th of the tank= 30 units Clearly, School A has maximum number of
6
30 3 buses.
Required time= = 3 minutes
5+3 4
56. (e)
132×7
47. (a); Radius of first circle= = 21 𝑐𝑚
2×22
22
Area of first circle= 7
× 21 × 21 = 1386 cm2
110×7
Radius of second circle= 2×22 = 17.5 cm
22
Area of second circle= × 17.5 × 17.5
7 57. (a)
= 962.5 cm2
Required difference=423.5 cm2
48. (e); Ratio of milk to that of water in the initial
mixture=16:5
1
th of the mixture=21 liter 58. (d)
4
16
64−21× 2
21
5 =
20−21× +𝑥 1
21
𝑥 = 9 liter

46 Adda247 | No. 1 APP for Banking & SSC Preparation


Website: bankersadda.com | sscadda.com | store.adda247.com | Email: contact@bankersadda.com
59. (a); I. x² – 3x – 4=0 67. (e); Let the number of students in the exam be
𝑥 2 − 4𝑥 + 𝑥 − 4 = 0 55x
(x – 4) (x + 1) = 0 Then number of boys= 36x
x = 4, –1 Number of girls=19x
II. y² + 6y + 8 = 0 ATQ
y² + 2y + 4y + 8 = 0 55𝑥 + 1725 = (36𝑥 + 1440) + 19𝑥 × 1.15
(y + 2) (y + 4) = 0 𝑥 = 100
y = –2, –4 Increased number of boys=3600+1440=5040
⇒x>y
68. (e); Let the salary of A and B be Rs 100x and Rs
60. (b); I. x² – 3x = 10 300x respectively
𝑥 2 − 3𝑥 − 10 = 0 ATQ
x² – 5x + 2x – 10 = 0 85𝑥 + 255𝑥 = 42500 ⇒ 𝑥 = 125
(x – 5) (x + 2) = 0 House rent paid by A=Rs 1875
x = –2, 5
69. (b);
II. y² + 7y + 10 = 0
y² + 5y + 2y + 10 = 0
(y + 5) (y + 2) = 0
y = – 2, –5 ⇒ x ≥ y
Required ratio = 8:9:10
61. (b); Items sold by A and B on Monday
2
= 200 + 160 = 360 70. (c); ATQ,
1102.50
= (1 +
𝑟
)
1000 100
Item sold by B and C on Wednesday 𝑟 2 105 2
= 320 + 280 = 600 or, (1 + ) =( )
100 100
360 6 3
∴ Required ratio = = = 𝑟 2 5 2
600 10 5 or, (1 + 100) = (1 + 100)
62. (c); Average of items sold by A, B, C on Thus, on comparing, 𝑟 = 5%
Wednesday 5
210+320+280 810 71. (e); ?2 = 40% of 11 × 352
= = = 270 2 5
3 3
?2 = 5 × 11 × 352 = 64 ⇒ ? = 8
63. (a); items sold by A and B on Tuesday = 240 + 180
= 420 72. (e); ?2=
(√1444+√676)
=
38+26
=
64
=16 ⇒? = 4
Items sold by B and C on Wednesday = 320 + 4 4 4

280 = 600 73. (b); (? −0.5) = 60 × 0.2


420×100
∴ Required percentage = = 70% ? = 12 + 0.5 = 12.5
600
60
64. (d); items sold by B on Monday and Tuesday 74. (d); ×? −18 = 222
100
= 200 + 180 = 380 60
×? = 240
Items sold by A on Tuesday and Wednesday 100
240×100
= 240 + 210 = 450 ?= 60
⇒ ? = 400
∴ Required difference = 450 – 380 =70 8×9×?
75. (c); =8
90
65 (a); Item sold by B on all 3 days = 200 +180 + 320 90×8
= 700 ?= 8×9
= 10 ⇒ ? = 10
Items sold by C on all 3 days = 150 + 330 +
76. (a); √4 ×? = 16
280 = 760
700 35 4 × ? = 256 ⇒ ? = 64
Required ratio = 760 = 38
77. (d); 77 + 92 = ?2
66. (c); Let the marked price be Rs 100x 169 = ?² ⇒ ? = 13
Then selling price= Rs 85x 50
200 78. (e); 5 = ⇒? = 10
Cost price=𝑅𝑠 3 𝑥 ?
9
ATQ 79. (b); × 8 =?× 10 ⇒? = 3.6
200 2
100𝑥 − 3 𝑥 = 250 80
𝑥 = 7.5 80. (c); × (6+? ) = 24
100
Cost price=Rs 500 6+? = 30 ⇒ ? = 24

47 Adda247 | No. 1 APP for Banking & SSC Preparation


Website: bankersadda.com | sscadda.com | store.adda247.com | Email: contact@bankersadda.com
IBPS RRB Clerk Prelims 2018 | Memory Based Paper |
For Practice

REASONING ABILITY
Directions (1-5): Read the following information (a) Only I follows
carefully and answer the questions given below. (b) Only II follows
(c) Either I or II follows
Six persons J, P, Q, R, V, Z are sitting in a row. Some of
(d) Neither I nor II follows
them are facing north while some of them are facing
(e) Both I and II follow
south. J sits second from one of the extreme end of the
row. P sits third to the right of J. R is not an immediate 7. Statements: Some logic are answers.
neighbor of P and Z. Both the immediate neighbors of V All keys are answers.
faces opposite direction. Both the Immediate neighbors Conclusions: I. All keys are logic.
of Z faces same direction. V sits second to the left of P. Q II. No keys are logic.
sits to the right of R. R faces north. Q faces same direction (a) Only I follows
as Z. (b) Only II follows
(c) Either I or II follows
1. Four of the following five are alike in a certain way,
(d) Neither I nor II follows
and so form a group. Which of the following does not
(e) Both I and II follow
belong to the group?
(a) R, V (b) V, P (c) J, P 8. Statement: All numbers are digits.
(d) V, Q (e) J, R Some numbers are points.
Some points are marks.
2. What is the position of Q with respect to Z?
Conclusions: I. Some points are digits.
(a) Second to the left (b) Third to the right
II. All marks being numbers is a
(c) Third to the left (d) Fifth to the right
possibility.
(e) Second to the right
(a) Only I follows
3. Who amongst the following sits exactly between Z (b) Only II follows
and J? (c) Either I or II follows
(a) R (b) P (c) Q (d) Neither I nor II follows
(d) Both V and Q (e) V (e) Both I and II follow
4. How many persons in the given arrangement are Directions (9-13): Read the following information
facing North? carefully and answer the questions given below.
(a) More than four (b) Four Seven boxes M, N, O, P, Q, R, S are arranged one above
(c) One (d) Three (e) Two another. Only two boxes are placed above box P. Only one
box is placed between box S and P. As many as boxes are
5. Who is sitting 4th to the right of Q?
placed between box S and Q as between box Q and M.
(a) R (b) Z (c) P
(d) J (e) None of these Three boxes are placed between box N and O. N is placed
above O.
Directions (6-8): Some statements are given followed
9. How many total numbers of boxes are placed in
by two conclusions. You have to consider the statements
between box S and Q?
to be true even if they seem to be at variance from
(a) Two (b) One (c) Three
commonly known facts. You have to decide which of the
(d) More than three (e) None
following conclusions follow from the given statements:
10. Which of the following is true regarding Box N?
6. Statements: No symbol is letter.
All expression are letter. (a) Three boxes are placed between box Q and N
(b) Box N is placed below Q
Some symbols are word.
(c) Box N is placed at top
Conclusions: I. No word is letter.
(d) Only one box is placed above box N
II. Some symbols being expression is
possibility. (e) No box is placed between box N and R

48 Adda247 | No. 1 APP for Banking & SSC Preparation


Website: bankersadda.com | sscadda.com | store.adda247.com | Email: contact@bankersadda.com
11. Which box is placed at top? 21. How many persons born were before R?
(a) S (b) N (c) Q (a) One (b) Three (c) Four
(d) R (e) M (d) Two (e) None of these
12. Which box is placed immediately above box Q? 22. Who among the following is the oldest?
(a) M (b) P (c) S (a) C (b) A (c) Q
(d) N (e) R (d) T (e) Y

13. How many boxes are placed in between R and M? 23. Which of the following is not true regarding Y?
(a) Two (b) One (c) Three (a) Four persons born between C and Y
(d) More than three (e) None (b) R was born before Y
(c) Q is born immediately after Y
Directions (14-18): Answer these questions based on (d) Only Q was born between Y and R
the following information. (e) No one was born after Y
In a certain code:
24. A family consists of five members A, P, R, T, H. P is
“arrange things in order” is coded as - “po gb ik mn”
wife of A. R is the daughter of A. R has only one
“order for new things” is coded as - “po gb fc bv”
brother T. H is daughter-in-law of P. How is H related
“new places to order” is coded as - “gb cq bv ra”
to R?
“places in unknown country” is coded as – “ de ra lf ik” (a) mother (b) sister-in-law
14. What will be the code for “order”? (c) daughter (d) daughter-in-law
(a) gb (b) fc (c) cq (e) none of these
(d) ik (e) can’t be determined 25. If the digits of the number “46752983” are arranged
15. What may be the code for “things to vanish”? in increasing order form left to right within the
(a) po cq hx (b) po vm ik number, then how many digits will remain on the
(c) cq fc ik (d) either (a) or (b) same position after the applied operation?
(e) None of these (a) Two (b) One (c) Three
(d) Four (e) None of these
16. What will be the code for “ arrange”?
26. How many meaningful words can be made by using
(a) gb (b) mn (c) cq
letters ‘A’, ‘E’, ‘L’ and ‘T’, keeping L as the first letter
(d) ik (e) can’t be determined
of the word?
17. What may be the code for “ in country”? (a) One (b) Two (c) Three
(a) lf ik (d) Four (e) None of these
(b) de ik Directions (27-31): Read the following information
(c) po gb carefully and answer the questions given below.
(d) either (a) or (b) Point E is 15m east of point B. Point G is 20m north of
(e) None of these point E. Point K is 10m east of point G. Point M is 30m
18. “bv” is the code for? south of point K. Point P is 20m west of point M. Point L
(a) things (b) new (c) arrange is 10m north of point P.
(d) places (e) None of these 27. If Point V is 10m east of point S and Point S is 10m
north of point L, then what will be the distance
Directions (19-23): Read the following information
between point E and V?
carefully and answer the questions given below.
(a) 10m (b) 15m (c) 20m
Six persons A, C, Q, R, T, Y were born in six different (d) 5m (e) 25m
months January, April, May, August, September,
December of a year. Three persons were born in between 28. What is the total distance between point B and L?
A and Y. A was born before Y. No one was born in (a) 10m (b) 15m (c) 20m
between C and A. Two persons were born in between C (d) 5m (e) 30m
and R. T was born before Q. 29. If Point Z is 10m north of point M, then point what is
19. Who among the following was born in May? the distance between point E and Z?
(a) C (b) A (c) Q (a) 10m (b) 15m (c) 20m
(d) T (e) Y (d) 25m (e) 30m
20. How many persons were born between A and Q? 30. Point K is in which direction from point P?
(a) One (b) Three (c) Four (a) South (b) South-east (c) North
(d) Two (e) None of these (d) North-east (e) North-west

49 Adda247 | No. 1 APP for Banking & SSC Preparation


Website: bankersadda.com | sscadda.com | store.adda247.com | Email: contact@bankersadda.com
31. Four of the following five are alike in a certain way, 36. Which of the following element is 5th to the right of
and so form a group. Which of the following does not 10th from the right end?
belong to the group? (a) 9 (b) 8 (c) 2
(a) P, L (b) P, M (c) G, E (d) 1 (e) 4
(d) L, E (e) G, B
Directions (37-40): Read the following information
Directions (32-36): These questions are based on the carefully and answer the questions given below.
following arrangement. Study it carefully and answer the There are six wallets A, B, C, P, Q and R, each containing
questions below it. different amount of money in it. Wallet B has more
135345 9287236527381218498124735 money than wallet Q but less than wallet P. Only wallet R
2489824 has more money than wallet C. Wallet Q does not has the
least amount of money. The wallet containing 3rd highest
32. Which element is exactly midway between the
amount of money has Rs. 3000, which is Rs.1000 more
seventh element from the left end and sixteenth from
than the wallet which has 2nd lowest amount of money.
the right end?
(a) 8 (b) 2 (c) 5 37. Which of the following wallet has the least amount of
(d) 6 (e) 7 money?
(a) A (b) B (c) C
33. How many perfect squares are there to the right of
(d) Q (e) P
the fourteenth element from the right end?
(a) Two (b) One (c) Three 38. What may be the amount of money in wallet C?
(d) Four (e) more than four (a) Rs. 2500 (b) Rs. 2000 (c) Rs. 3500
(d) Rs. 2250 (e) Rs. 2100
34. How many perfect cubes are there in the above
arrangement, each of which is immediately preceded 39. What may be the amount of money in wallet B, if it
by an odd number and immediately followed by an has Rs. 250 less than the wallet P?
even number? (a) Rs. 2500 (b) Rs. 2750 (c) Rs. 3500
(a) None (b) Three (c) Two (d) Rs. 3250 (e) Rs. 2200
(d) One (e) More than three
40. Which of the following is true regarding wallet P?
35. How many such odd digits are there in the given (a) Only wallet A has less money than wallet P
arrangement, each of which is immediately followed (b) Wallet B has more money than wallet P
and preceded by an odd number? (c) Wallet P has 3rd highest amount of money
(a) None (b) One (c) Two (d) Wallet Q has more amount of money than P
(d) Three (e) More than three (e) none of these

QUANTITATIVE APTITUDE

41. The upstream speed of a boat is 18 km/hr which is 44. A train having speed of 72 km/hr crosses a pole in 18
500% more than the speed of stream. Find how sec and a platform in 33 sec. Find the length of
much distance boat will cover in 3 hours while platform?
travelling in downstream. (a) 320 m (b) 300 m (c) 330 m
(a) 66 km (b) 63 km (c) 72 km (d) 360 m (e) 350 m
(d) 75 km (e) 78 km 45. The circumference of a circle is 66 cm. Find the
42. If 𝐴 – 𝐵 = 252 and 𝐴 + 𝐵 = 42 then find the value
2 2 approximate area of square if the radius of circle is
of ‘B’? two times of the side of a square.
(a) 18 (b) 16 (c) 14 (a) 18 cm² (b) 32 cm² (c) 25 cm²
(d) 20 (e) 22 (d) 36 cm² (e) 28 cm²

43. A alone can do a work in 40 days. The ratio of time Directions (46-50): What approximate value should
taken by A and B to do the same work is 5 : 3. Then, come in place of question mark (?) in the following
find in how many days both will complete the work questions?
together ? 46. √1443.98 ÷ 18.98 + 328.1 =?× 22.01
(a) 18 days (b) 12 days (c) 20 days (a) 10 (b) 12 (c) 18
(d) 15 days (e) 10 days (d) 15 (e) 22

50 Adda247 | No. 1 APP for Banking & SSC Preparation


Website: bankersadda.com | sscadda.com | store.adda247.com | Email: contact@bankersadda.com
47. 29.98% of 880.001 = ? + 110.9 58. Number of employees in Housing department is how
(a) 144 (b) 153 (c) 158 much more than number of employees in Security
(d) 160 (e) 163 department?
(a) 10 (b) 20 (c) 30
48. (?)² + 255.93 = 49.932% of 800.112 (d) 40 (e) 50
(a) 12 (b) 8 (c) 15 59. In Security department, 40% are female employees
(d) 18 (e) 6 then find total male employees working in Security
3 department?
49. √1728.01+? = 256.01 (a) 16 (b) 40 (c) 32
(a) 230 (b) 235 (c) 238 (d) 8 (e) 24
(d) 241 (e) 244
60. Ratio between total number of male and female
50. 74.91% of ? = (17.932)² employees in HR department is 2 : 3. Find total
(a) 420 (b) 425 (c) 408 number of female employees working in HR
(d) 432 (e) 444 department?
(a) 32 (b) 48 (c) 64
Directions (51-55):-Find the wrong number in the (d) 40 (e) 56
given number series questions. Directions (61-70): What value should come in place of
51. 100, 118, 136, 149, 160, 167, 172 question mark (?) in the following questions?
(a) 172 (b) 160 (c) 100 61. ?2 = 42 + 82 − 31
(d) 118 (e) 136 (a) 6 (b) 7 (c) 8
(d) 9 (e) 10
52. 1.5, 2.5, 6, 24, 100, 505, 3036
(a) 1.5 (b) 6 (c) 100 62. 13 × 6 +? × 4 = 18 × 7
(d) 3036 (e) 2.5 (a) 6 (b) 8 (c) 10
(d) 12 (e) 14
53. 160, 80, 80, 120, 240, 600, 900
63. 40% of ? = 25% of 320 + 75% of 160
(a) 240 (b) 120 (c) 160
(a) 500 (b) 400 (c) 300
(d) 900 (e) 600 (d) 200 (e) 100
54. 5040, 2520, 840, 210, 42, 8, 1 64. 112 + 62 = ? +37
(a) 8 (b) 5040 (c) 840 (a) 130 (b) 110 (c) 120
(d) 1 (e) 42 (d) 140 (e) 150
55. 15, 17, 26, 151, 200, 929, 1050 65.
360
= 12 × 6 − 33
?
(a) 17 (b) 1050 (c) 15
(a) 9 (b) 5 (c) 6
(d) 929 (e) 26 (d) 7 (e) 8
Direction (56-60): There are total five departments in a
66. √225 + √441 = ?2
company. There are total 90 employees in Finance (a) 3 (b) 4 (c) 5
department which is 25% of total employees in the (d) 6 (e) 8
company. 2/9 of the total employees of the company are
working in HR department. Employees working in Sales 67. 16 × 8−? = 26
department is 25% more than that in HR department. (a) 64 (b) 32 (c) 128
Ratio between employees working in Security and (d) 192 (e) 96
Housing department is 4 : 5. 68. 16 × 54 ÷ 36 + 6 =?
144
56. Find number of employees working in HR (a) 7 (b) 30 (c) 20
department is what percent more than number of (d) 24 (e) 16
employees working in Security department?
(a) 250% (b) 200% (c) 150% 69. ? = √6 × 3 × 5 + 50% 𝑜𝑓 620
(d) 100% (e) 50% (a) 14 (b) 16 (c) 18
(d) 10 (e) 20
57. Find the average number of employees working in 18×8−?×2
Sales, Finance and Housing department? 70. 62 = 3
(a) 60 (b) 70 (c) 80 (a) 36 (b) 27 (c) 18
(d) 90 (e) 100 (d) 9 (e) 54

51 Adda247 | No. 1 APP for Banking & SSC Preparation


Website: bankersadda.com | sscadda.com | store.adda247.com | Email: contact@bankersadda.com
Direction (71-75): - Table given below shows marks (a) 64.25% (b) 61.25% (c) 67.25%
obtained by four students in four different subjects in an (d) 70.25% (e) 73.25%
exam. Study the data carefully and answer the following
76. An article was sold at a discount of 20% at Rs. 1020.
questions
If the article was sold at discount of Rs. 199 in place
Subjects
English Hindi Science Maths
of 20% discount then find the selling price.
Students (a) Rs. 1066 (b) Rs. 1076 (c) Rs. 1086
Paul 65 60 80 65 (d) Rs. 1096 (e) Rs. 1094
Aditya 75 75 60 75
Neeraj 85 55 95 85 77. The total age of A, B and C four years hence will be
Sandy 60 60 65 60 98 years. Find the age of C four years hence if the
present age of A and B is 32 years and 23 years
71. Marks scored by Sandy in English and Maths
respectively.
together is what percent of the Marks scored by (a) 31 yr. (b) 32 yr. (c) 35 yr.
Aditya and Neeraj in English together? (d) 37 yr. (e) 33 yr.
(a) 25% (b) 50% (c) 75%
(d) 100% (e) 125% 78. A invests Rs. 12,000 for X months while B invests Rs.
16,000 for 9 months in a scheme. The profit share of
72. Find the ratio of total marks scored by all four B is Rs. 12,000 out of total profit Rs. 21,000. Then
students together in Hindi to total marks scored by find the value of X ?
all four students together in Science? (a) 6 months (b) 9 months (c) 8 months
(a) 5 : 6 (b) 57 : 50 (c) 1 : 1 (d) 7 months (e) 10 months
(d) 20 : 19 (e) 6 : 5
79. A mixture of milk and water contains 60% milk and
73. Total marks scored by Paul are how much more/less remaining water. How much water should be added
than total marks scored by Neeraj? (in percentage) in mixture to reverse the proportion
(a) 70 (b) 60 (c) 40 of milk and water?
(d) 50 (e) 80 (a) 25% (b) 37.5% (c) 62.5%
(d) 75% (e) 50%
74. Find the average of the marks scored by Aditya in
English, Hindi and Science together? 80. The simple interest on a certain sum for 2 years at
(a) 65 (b) 85 (c) 80 8% per annum is Rs. 225 less than the compound
(d) 75 (e) 70 interest on the same sum for 2 years at 10% per
annum. The sum is:
75. If maximum marks for each subject are 100 then find (a) Rs. 3200 (b) Rs. 4200 (c) Rs. 4000
what percentage of total marks is obtained by Sandy? (d) Rs. 3600 (e) Rs. 4500

Solutions

REASONING ABILITY

Direction (1-5): 7. (d);

1. (d) 2. (a) 3. (e)


4. (b) 5. (d) 8. (e);

6. (d);

52 Adda247 | No. 1 APP for Banking & SSC Preparation


Website: bankersadda.com | sscadda.com | store.adda247.com | Email: contact@bankersadda.com
Direction (9-13): 22. (a) 23. (c)
Boxes
24. (b)
S
N
P
Q
25. (a)
R
O
M
9. (a) 10. (d) 11. (a) 26. (b);LATE, LEAT
12. (b) 13. (b) Direction (27-31):
Directions (14-18):

Word Code
Places ra
Order gb
New bv
To cq
Things po
For fc
In ik
Arrange mn
Unknown/country de/lf 27. (a) 28. (d) 29. (a)

14. (a) 15. (a) 16. (b) 30. (d) 31. (e) 32. (b);

17. (d) 18. (b) 33. (e);


Direction (19-23): 34. (c); 184,982
Months Persons
35. (d); 735,135,353
January C
April A 36. (b);
May T Directions (37-40):
August R
September Q R > C > P (Rs. 3000) > B > Q (Rs.2000) > A
December Y 37. (a); 38. (c); 39. (b);
19. (d) 20. (d) 21. (b) 40. (c);

QUANTITATIVE APTITUDE
41. (c); Let the speed of stream be 𝑥 km/hr And A + B = 42 …(ii)
Then, Solve (i) and (ii), we get ⇒ B = 18
600
Speed of upstream = 𝑥 × = 18 ⇒ 𝑥 = 3km/hr 43. (d);Let the time taken by A and B be 5𝑥 days and 3𝑥
100
Speed of boat in still water = 18 + 3 = 21 km/hr days respectively.
Distance covered in 3 hours in downstream = ⇒ 5𝑥 = 40 days ⇒ 𝑥 = 8 days
(21 + 3) × 3 = 72 km B’s time = 3 × 8 = 24 days
Time taken by both together to complete the
42. (a); (A + B)(A – B) = 252 40×24 𝑎×𝑏
⇒ 42 × (A– B) = 252 [A + B = 42 given] work = 40+24 [𝑢𝑠𝑒 𝑎+𝑏 for two persons]
⇒ (A– B) = 6 …(i) =15 days.

53 Adda247 | No. 1 APP for Banking & SSC Preparation


Website: bankersadda.com | sscadda.com | store.adda247.com | Email: contact@bankersadda.com
44. (b);Speed of train = 72 km/hr Solution (56-60): Let total employees in company be
5
= 72 × 18 = 20 m/s 100x
ATQ,
Length of train = 18 × 20 = 360 m 25
Length of (train + platform) × 100𝑥 = 90
100
= 20 × 33 = 660 m ⇒ Total employees in company = 100𝑥 = 360
∴ length of platform = 660 m – 360 m = 300 m Employees working in HR department
2
45. (e); ATQ, = 9 × 360 = 80
2πr = 66 cm Employees working in Sales department
22 125
⇒ 2 × × 𝑟 = 66 cm = 100 × 80 = 100
7
66×7 21
⇒𝑟= = cm Remaining employees = 360 − 90 − 80 − 100 = 90
44 2
21 21 Employees working in Security department
Side of a square = = cm 4
2×2 4 = × 90 = 40
21 2 9
∴ Area of square = (side)² = ( 4 ) Employees working in Housing department
441 5
= ≈ 28 𝑐𝑚2 = 9 × 90 = 50
16

46. (d);√1444 ÷ 19 + 328 = ? × 22


⇒ 2 + 328 = ? × 22 80−40 40
330
⇒ ? = 22 = 15 . 56. (d);Required % = × 100 = ×100 = 100%
40 40
100+90+50 240
47. (b);30% of 880 = ? + 111 57. (c); Required average = = = 80
3 3
30×880
⇒ 100 =? +111 58. (a); Required difference = 50 − 40 = 10
⇒ ? = 264 – 111 = 153.
50×800 59. (e); Total number of male employees working in
48. (a); (? )2 + 256 = 100 60
Security department = 100 × 40 = 24
(? )2 + 256 = 400
⇒ (? )2 = 144 60. (b);Total number of female employees working in
⇒? = 12 3
HR department = 5 × 80 = 48
49. (e); 12 + ? = 256
61. (b); ?2 = 42 + 82 − 31
⇒ ? = 244
?2 = 16 + 64 − 31 = 80 − 31 = 49
75×?
50. (d); 100 = (18)2 ?= 7
75×? 324×100
⇒ = 324 ⇒ ? = = 432. 62. (d); 13 × 6 +? × 4 = 18 × 7
100 75
78 +? × 4 = 126
51. (d); ?=
126−78
= 12
4

63. (a); 40% 𝑜𝑓 ? = 25% 𝑜𝑓 320 + 75% 𝑜𝑓 160


2 25 75
×? = × 320 + × 160
52. (b); 5 100 100
2
5
×? = 80 + 120
5
? = 200 × 2 = 500
53. (d); 64. (c); 112 + 62 =? +37
121 + 36 − 37 =?
? = 120
54. (a); 65. (e);
360
= 12 × 6 − 33
?
360
= 72 − 27
?
360
?= =8
55. (c); 45

66. (d);√225 + √441 = ?2


15 + 21 =?2
?2 = 36
?= 6

54 Adda247 | No. 1 APP for Banking & SSC Preparation


Website: bankersadda.com | sscadda.com | store.adda247.com | Email: contact@bankersadda.com
67. (a); 16 × 8−? = 26 77. (c); Sum of present age of A, B and C
128 − 64 =? ⇒ ? = 64 = 98 – 4 × 3
68. (b);16 × 54 ÷ 36 + 6 =? = 98 – 12 = 86 yr.
54
? = 16 × 36 + 6 = 30 Present age of C = 86 – (32 + 23) = 31 yr.
Age of C four years hence = 31 + 4 = 35 yr.
69. (e); ? = √6 × 3 × 5 + 50% of 620 78. (b);Profit share ratio of
? = √90 + 310 = √400 = 20 A B
18×8−?×2 12000 × x 16000 × 9
70. (c); 62 =
3 x : 12
36 × 3 = 144−?× 2 ATQ,
?× 2 = 144 − 108 x 9000
36 = 12000
? = 2 = 18 71. (c); Marks scored by Sandy 12
⇒ x = 9 months.
in English and Maths together = 60 + 60 = 120
Marks scored by Aditya and Neeraj in English 79. (e); Let, total quantity = 100ℓ
together = 75 + 85 = 160 Quantity of milk = 60 ℓ
120
Required % = × 100 = 75% And quantity of water = 40 ℓ
160
ATQ,
60+75+55+60 250 5
72. (a); Required ratio = 80+60+95+65 = 300 = 6 40
=
60
100 100+x
73. (d);Total marks scored by Paul 2 (100 + x) = 5 × 60
= 65 + 60 + 80 + 65 = 270 200 + 2x = 300
Total marks scored by Neeraj 2x = 100
= 85 + 55 + 95 + 85 = 320 x = 50 ℓ
Required difference = 320 − 270 = 50 Water added in % =
50
× 100 = 50%
100
75+75+60
74. (e); Required average = = 70
3 80. (e); Let the sum be Rs. P.
60+60+65+60 11 11 P×2×8
75. (b);Required % = 400
× 100 = 61.25% P[
10
×
10
− 1] –
100
= 225
21 16P
76. (b);MP of article =
1020
× 100 = Rs. 1275 ⇒ P [100] – 100 = 225
80 225×100
Selling price = 1275 – 199 = Rs. 1076 ⇒P= ⇒ P=Rs. 4500
5

55 Adda247 | No. 1 APP for Banking & SSC Preparation


Website: bankersadda.com | sscadda.com | store.adda247.com | Email: contact@bankersadda.com
IBPS RRB Clerk Prelims 2017 | Memory Based Paper |
For Practice

REASONING ABILITY
Directions (1-5): In each of the question, relationships 9. Who among the following lives on Top floor?
between some elements are shown in the statements(s). (a) N (b) J (c) K
These statements are followed by conclusions numbered (d) M (e) None of these
I and II. Read the statements and give the answer. 10. Which of the following combination is false?
(a) If only conclusion I follows. (a) J-7 (b) L-3 (c) G-2
(b) If only conclusion II follows. (d) H-4 (e) N-1
(c) If either conclusion I or II follows.
(d) If neither conclusion I nor II follows. 11. In a row of children facing North, Rajan is twelfth
from the right end and is fifth to the right of Satyarthi
(e) If both conclusions I and II follow.
who is tenth from the left end. How many total
1. Statements: A < B >N = M, B ≤ V, M > R number of children are there in the row?
Conclusions: I. B > R II. V > A (a) 29 (b) 28 (c) 26
(d) 27 (e) None of these
2. Statements: D <E >F = G > H = I ≤ J
Conclusions: I. F > I II. J ≥ E 12. Raj leaves his home and goes straight 20 meters,
then turns right and goes 10 meters. He turns left and
3. Statements: M < N < O > P, N<E goes 30 meters and finally turns right and starts
Conclusions: I. E < M II. E>O walking. If he is now moving in the north direction,
4. Statements: C ≥ D< E = F ≥ G, C < W then in which direction did he start his walking?
Conclusions: I. E =G II. G< E (a) East (b) West (c) North
(d) South (e) None of these
5. Statements: R < T < S < P > Q, R> X
Directions (13-17): In each of the questions given
Conclusions: I. S < Q II. X < S
below, a group of digits/letter is given followed by four
Direction (6-10): Study the following information combinations of symbols numbered (a), (b), (c) and (d).
carefully and answer the question given below- You have to find out which of the four combinations
Eight people viz. G, H, I, J, K, L, M and N lives in a Building correctly represents the group of digits/letters based on
on different floors from top to bottom (such as ground the symbol codes and the conditions given below. If none
floor numbered as 1 and top is numbered as 8) but not of the four combinations represents the group of digits
necessarily in the same order. correctly, give (e) ie ‘None of these’ as the answer.
There is a gap of three floors between J and L and both of
them lives on odd number of floor. N lives just above H,
who lives on even numbered floor. I lives on floor
Condition for coding the group elements:
number 6.Only one person lives between L and M. J lives (i) If the first letter is Vowel and the last digit is divisible
above I. Three persons live between K and H. by 2, then both are to be coded as +.
6. Who among the following lives on ground floor? (ii) If the first as well as the last digit is odd, then both
(a) N (b) J (c) K are to be coded by the code of the first digit.
(d) M (e) None of these (iii) If the first letter is consonant and the last digit is odd
number, then the code of first and last elements are
7. Who among the following lives immediately below to be interchanged.
L?
13. WX6ZF1
(a) K (b) I (c) G
(a) ^®$@£∞ (b) ^@$∞<! (c) ^®£@$∞
(d) H (e) None of these
(d) ∞®@>!< (e) None of these
8. How many persons lives between I and H? 14. FE1XI6
(a) One (b) Three (c) Fives (a) ∞^@<!£ (b) $<^^£@ (c) $>^®µ£
(d) Two (e) None of these (d) $<^@^£ (e) None of these

56 Adda247 | No. 1 APP for Banking & SSC Preparation


Website: bankersadda.com | sscadda.com | store.adda247.com | Email: contact@bankersadda.com
15. 5L2IA1 25. If all the odd numbers are dropped from the series,
(a) ∆!<µ&∆ (b) ∆!&^<µ (c) ∆!<µ^& which number will be eighth to the left of eleventh
(d) µ&∆!<^ (e) None of these number from the left end?
(a) 2 (b) 8 (c) 6
16. E2ZA6 (d) 4 (e) None of these
(a) &>!^@ (b) @<@&! (c) @&<@&
(d) +<@&+ (e) None of these 26. If 1 is subtracted from all odd numbers and 2 is
subtracted from all even numbers in the given
17. IZ2W2 number series, then which number will be sixteenth
(a) @≠^$& (b) +@<∞+ (c) <∞µ@≠ from the right end?
(d) @≠>!^ (e) None of these (a) 0 (b) 2 (c) 3
Directions (18-22): Read the following information (d) 8 (e) 6
carefully and answer the questions given below. 27. If the position of the 1st and the 16th numbers, the 2nd
A, B, C, D, E, F, G and H are eight members standing in a and the 17th numbers, and so on up to the 15th and
row (not necessarily in the same order) facing north. the 30th numbers, are interchanged, which number
C and B have as many members between them as G and will be 7th to the right of 19th number from the right
C have between them. D, who is 4th from the extreme left end?
end, is 2nd to the left of E. G is 3rd place away from one of (a) 5 (b) 9 (c) 8
the extreme end. Neither B nor C sits any extreme end. F (d) 4 (e) None of these
sits immediate right of A.
28. How many total even numbers which is immediately
18. How many persons sit between G and B? preceded by a ‘whole cube’ or ‘immediately preceded
(a) One (b) Three (c) Two by a whole square’ in the above sequence?
(d) Four (e) None of these (a) Four (b) Five (c) Three
(d) Six (e) None of these
19. Who among the following persons sits at extreme
ends? Directions (29–33): In each question below are given
(a) A, G (b) B, C (c) F, H some statements followed by two conclusions numbered
(d) H, A (e) None of these I and II. You have to take the given statements to be true
even if they seem to be at variance with commonly
20. Who sits second to the right of E? known facts. Read all the conclusions and then decide
(a) B (b) H (c) G which of the given conclusions logically follows/follow
(d) C (e) None of these from the given statements, disregarding commonly
21. Who sits third to the left of G? known facts. Give answer
(a) A (b) None (c) F (a)If only conclusion I follows.
(d) E (e) B (b)If only conclusion II follows.
(c) If either conclusion I or II follows.
22. Who sits immediate left of C? (d)If neither conclusion I nor II follows.
(a) A (b) H (c) C (e)If both conclusions I and II follow.
(d) D (e) None of these
29. Statements: All shirts are skirts.
23. Find the odd one out? No skirt is top. All tops are kurta.
(a) ACB (b) DFE (c) GIH Conclusions: I. All shirts are kurta
(d) JLK (e) MNO II. Some kurta are skirts.
Directions (24-28): Study the following number 30. Statements: Some chocolate are chips.
sequence and answer the questions following it. Some chips are jelly.
9324579581506429826359821543 All jelly are whoppers.
21 Conclusions: I. Some jelly are chips.
II. All chocolate being whoppers is
24. How many odd numbers are there in the numeric a possibility
series which are immediately preceded by a number,
which is a whole square? 31. Statements: Some frooti are Maaza.
(a) One No Maaza is slice.
(b) Two All slice are fanta.
(c) Three Conclusions: I. Some frooti are definitely not
slice.
(d) More than three
II. Some fanta are definitely not
(e) None of these
Maaza.
57 Adda247 | No. 1 APP for Banking & SSC Preparation
Website: bankersadda.com | sscadda.com | store.adda247.com | Email: contact@bankersadda.com
32. Statements: All carbon are oxygen. 36. If the third alphabet in each of the words is changed
All Nitrogen are carbon. to the previous alphabet in the English alphabetical
Some oxygen are Sulphur. order, how many words thus formed will be without
Conclusions I. All Nitrogen being Sulphur is a any vowels?
possibility. (a) None (b) One (c) Two
II. All Nitrogen are not oxygen. (d) Three (e) More than three
33. Statements: All September are October. 37. If the position of the first and the third alphabet of
No October is November. each of the words are interchanged, which of the
No November is December. following will form a meaningful word in the new
Conclusions: I. Some September are not arrangement?
Novembers (a) NOW (b) SAD (c) RAT
II. No October is December. (d) WAF (e) Both (a) and (c)
Directions (34-38): Following questions are based on 38. If in each of the given words, each of the consonants
the five words given below, Study the following words is changed to its previous letter and each vowel is
and answer the following questions. changed to its next letter in the English alphabetical
NOW SAD WAF RAT CAT series, then how many words thus formed will at
least one vowel appear?
(The new words formed after performing the mentioned (a) None (b) One (c) Two
operations may not necessarily be a meaningful English (d) Three (e) None of these
word.)
39. If in the number 9737132710, positions of the first
34. If the given words are arranged in the order as they and the second digits are interchanged, positions of
appear in a dictionary from left to right, which of the the third and fourth digits are interchanged and so
following will be the fourth from the left end? on till the positions of 9th and 10th digits are
(a) WAF (b) NOW (c) SAD interchanged, then which digit will be 6th from the
(d) CAT (e) RAT left end?
35. How many letters are there in the English (a) 7 (b) 1 (c) 3
alphabetical series between the second letter of the (d) 9 (e) None of these
word which is second from the right end and the 40. How many pairs of letters are there in the word”
third letter of the word which is second from the left WORSHIP” which have as many letters between
end? them in the word as in alphabetical series?
(a) Two (b) Three (c) Four (a) None (b) One (c) Two
(d) Five (e) None of these (d) Three (e) Four

QUANTITATIVE APTITUDE
41. The retail price of a water geyser is Rs. 1265. If the 44. In a mixture of 45 litres, the ratio of milk and water
manufacturer gains 10%, the wholesale dealer gains is 3 : 2. How much water must be added to make the
15% and the retailer gains 25%, then the cost of the ratio 9 : 11?
product is: (a) 10 litres (b) 15 litres (c) 17 litres
(a) Rs. 800 (b) Rs. 900 (c) Rs. 700 (d) 20 litres (e) None of these
(d) Rs. 600 (e) None of these
45. A person can row with the stream at 8 Km per hour
42. A pipe can fill a cistern in 6 hrs. Due to a leak in its and against the stream at 6 Km an hour. The speed of
bottom, it is filled in 7 hrs. When the cistern is full, in the current is:
how much time will it be emptied by the leak? (a) 1 Km/h (b) 2 Km/h (c) 4 Km/h
(a) 42 hrs (b) 40 hrs (c) 43 hrs
(d) 5 Km/h (e) None of these
(d) 45 hrs (e) None of these
43. Ram travels a certain distance at 3 km/h and reaches 46. A father’s age is three times the sum of the ages of his
15 minutes late. If he travels at 4 km/h, he reaches two children, but 20 years hence his age will be equal
15 minutes earlier. The distance he has to travel is: to the sum of their ages. Then, the father’s age is:
(a) 4.5 km (b) 6 km (c) 7.2 km (a) 30 years (b) 40 years (c) 35 years
(d) 12 km (e) None of these (d) 45 years (e) None of these

58 Adda247 | No. 1 APP for Banking & SSC Preparation


Website: bankersadda.com | sscadda.com | store.adda247.com | Email: contact@bankersadda.com
47. A sum was put at simple interest at a certain rate for 49. A card is drawn at random from a well-shuffled pack
3 years. Had it been put at 1% higher rate, it would of 52 cards. What is the probability of getting a two
have fetched Rs. 5100 more. The sum is: of hearts or a two diamonds?
3 2 1
(a) Rs. 170000 (b) Rs. 150000 (c) Rs. 125000 (a) 26 (b) 17 (c) 26
(d) Rs. 120000 (e) None of these 4
(d) 13 (e) None of these
48. From among 36 teachers in a school, one principal
50. A sum is invested for 3 years at compound interest at
and one vice-principal are to be appointed. In how
5%, 10% and 20% respectively. In three years, if the
many ways can this be done?
sum amounts to Rs. 16,632, then find the sum.
(a) 1260 (b) 1250 (c) 1240
(a) Rs. 11000 (b) Rs. 12000 (c) Rs. 13000
(d) 1800 (e) None of these
(d) Rs. 14000 (e) None of these
Directions (51-55): Table shows the mobile phones sold on different days by different sellers. Read the table carefully
and answer the questions.

51. Find the difference of mobile phones sold by P and R Directions (Q.56-65): What should come in place of
together on Monday to the mobile phones sold by S question mark (?) in following simplification problems?
and T on Wednesday ?
56. 45% of 600+? % of 480 = 390
(a) 60 (b) 50 (c) 80 (a) 20 (b) 25 (c) 30
(d) 20 (e) None of these (d) 40 (e) None of these
52. Find the ratio of mobile phone sold by Q on Tuesday 2 1 2
57. 4 + 7 − 5 = ?
and Saturday together to the mobile phone sold by 3
2
6 9
2 11
R on Thursday and Sunday together? (a) 6 (b) 6 (c) 6
3 9 18
7
(a) 7 : 19 (b)19 : 5 (c)19 : 6 (d) 6 18 (e) None of these
(d) 2 : 5 (e) None of these
58. 65% of 240+ ? % of 150 = 210
53. Mobile phones sold by P and S together on (a) 45 (b) 46 (c) 32
Wednesday is what percent of mobile phone sold by (d) 36 (e) None of these
T on Sunday ? 2 2
(a) 400% (b) 200% (c) 100% 59. of 1 of 75% of 540 = ?
3 5
(d) 50% (e) None of these (a) 378 (b) 756 (c) 252
(d) 332 (e) None of these
54. What is the average of mobile phone sold by Q on
Wednesday, T on Sunday and S on Monday ? 60. 555.05 + 55.50 + 5.55 + 5 + 0.55 = ?
(a) 24 (b) 36 (c) 30 (a) 621.65 (b) 655.75 (c) 634.85
(d) 28 (e) None of these (d) 647.35 (e) None of these

55. The mobiles sold by P on Thursday are of two types 61. 1425 + 8560 + 1680 ÷ 200 = ?
i.e. Windows phone and Android phone in ratio 3 : 4. (a) 58.325 (b) 9973.4 (c) 56.425
Find the number of Windows phones sold by P on (d) 9939.4 (e) None of these
Thursday? 62. ? % of 800 = 293 – 22% of 750
(a) 14 (b) 24 (c) 16 (a) 14 (b) 18 (c) 12
(d) 12 (e) None of these (d) 16 (e) 20

59 Adda247 | No. 1 APP for Banking & SSC Preparation


Website: bankersadda.com | sscadda.com | store.adda247.com | Email: contact@bankersadda.com
63. 25.6% of 250 + √? = 119 Directions (Q.71-75): What should come in place of
(a) 4225 (b) 3025 (c) 2025 question mark (?) in following simplification problems?
(d) 5625 (e) None of these
71. 50% of 250 + √? = 165
5 5 1 11
64. 4 6 − 5 9 = ? −2 3 + 18 (a) 1700 (b) 1600 (c) 1800
3 1 7 (d) 2000 (e) None of these
(a) 4 (b) 2 18 (c) 1 9
(d) 1
11
(e) None of these 72. 140% of 56 + 56% of 140 = ?
18
(a) 78.4 (b) 158.6 (c) 156.6
65. [30% of {(80% of 850) ÷ 34}] = ? (d) 87.4 (e) None of these
(a) 5 (b) 4 (c) 6 1 5 5 1
(d) 8 (e) 9 73. 1 4 + 1 9 × 1 8 ÷ 6 2 = ?
1 1 1 (a) 17 (b) 27 (c) 42
66. The sides of a triangle are in the ratio of ∶ ∶ . If the
2 3 4 (d) 18 (e) None of these
perimeter is 52 cm, then the length of the smallest
side is: 74. 999.09 + 99.90 + 9.99 + 9 + 0.99 =?
(a) 9 cm (b) 10 cm (c) 11 cm (a) 1118.97 (b) 1128.97 (c) 1218.97
(d) 12 cm (e) None of these (d) 1139.97 (e) None of these

67. If A’s salary is 25% higher than B’s salary, then how 75. 20% of [{(220% of 40) − 10}]% of 500 = ?
much per cent is B’s salary lower than A’s? (a) 58 (b) 68 (c) 98
(a) 15% (b) 20% (c) 25% (d) 78 (e) None of these
1
(d) 33 % (e) None of these Directions (Q.76-80): What should come in place of
3
1 question mark (?) in following number series ?
68. Ravi sells an article at a gain of 12 %. If he had sold
2
it at Rs. 22.50 more, he would have gained 25% The 76. 5, 8, 12, 18, 27, ?
cost price of the article is: (a) 39 (b) 40 (c) 41
(a) Rs. 162 (b) Rs. 140 (c) Rs. 196 (d) 42 (e) 43
(d) Rs. 180 (e) None of these 77. 2, 10, 30, 68, 130, ?
69. A certain job was assigned to a group of men to do it (a) 210 (b) 215 (c) 222
in 20 days. But 12 men did not turn up for the job and (d) 228 (e) 235
the remaining men did the job in 32 days. The
78. 142, 133, 115, 88, ?
original number of men in the group was:
(a) 50 (b) 53 (c) 55
(a) 32 (b) 34 (c) 36
(d) 40 (e) None of these (d) 51 (e) 52

70. A vessel contains liquid P and Q in the ratio 5 : 3. If 79. 3, 8, 18, 38, 78, ?
16 litres of the mixture are removed and the same (a) 158 (b) 154 (c) 150
quantity of liquid Q is added, the ratio become 3 : 5. (d) 162 (e) 166
What quantity does the vessel hold? 80. 6, 3, 3, 6, 24, ?
(a) 35 litres (b) 45 litres (c) 40 litres (a) 184 (b) 186 (c) 188
(d) 50 litres (e) None of these (d) 190 (e) 192

Solutions

REASONING ABILITY
1. (e); I. B > R (True) II. V > A (True) 3. (d);I. E < M (False) II. E>O (False)

2. (a); I. F > I (True) II. J ≥ E (False) 4. (c); I. E =G (False) II. G< E (False)

60 Adda247 | No. 1 APP for Banking & SSC Preparation


Website: bankersadda.com | sscadda.com | store.adda247.com | Email: contact@bankersadda.com
5. (b);I. S < Q (False) II. X < S (True) 24. (d);More than three
Direction (6-10): 25. (b);8
Floors Persons
26. (a); 0
8 K
7 J 27. (d);4
6 I 28. (b);Five
5 N
29. (d);
4 H
3 L
2 G
1 M
6. (d); 7. (c); 8. (a); 30. (e);
9. (c); 10. (e);
11. (c); Sathyarthi’s position from left end = 10th
Sathyarthi’s position from right end = 17th
Total number of children in the row 31. (e);
= 10 + 17 – 1 = 26
12. (b);Raj started walking towards west.
13. (c); By using condition (iii) the code of WX6ZF1 will
be ^®£@$∞. 32. (a);
14. (c); The code of FE1XI6 will be $>^®µ£.
15. (a); By using condition (ii) the code of 5L2IA1 will be
∆!<µ&∆.
16. (d);By using condition (i) the code of E2ZA6 will be 33. (a);
+<@&+.
17. (b);By using condition (i) the code of IZ2W2 will be
+@<∞+.
Direction (18-22): 34. (c); SAD
35. (a); TWO
36. (a); None
18. (b); 19. (d); 20. (b);
37. (e); WON, TAR
21. (b); 22. (d);
38. (b);One
23. (e);
39. (b);1
So, the odd one out will be MNO. 40. (d);Three- RS, HI, and PS

QUANTITATIVE APTITUDE
100 100 100
41. (a); Cost price = 110×115×125 ×1265=Rs. 800 43. (b);Let D be the required distance
𝐷 𝐷 15+15
1 So, 3 − 4 =
42. (a); In one hour, 6 of the cistern can be filled 60

1 Or, D = 6 km
In one hour, only 7 of the cistern can be filled due
to leak in its bottom 44. (b);Let, M = 3K, W = 2K
1 1 1
∴ In one hour 6 − 7 = 42 of the cistern is empty ∴ 3K + 2K = 45 ⇒ K = 9
∴ The whole cistern will be emptied in 42 hrs ⇒ Milk = 27 litres and water = 18 litres

61 Adda247 | No. 1 APP for Banking & SSC Preparation


Website: bankersadda.com | sscadda.com | store.adda247.com | Email: contact@bankersadda.com
Now suppose x litres of water is added to the 45
56. (b);100 𝑜𝑓 600 + 100 𝑜𝑓 480 = 390
?

mixture such that


27 9 ⇒ 270 + 4.8 ×? = 390
= 11 ⇒ 162 + 9𝑥 = 297 390−270
18+𝑥 ∴? = 4.8
= 25
⇒ 9𝑥 = 135 ⇒ 𝑥 = 15
14 43 47 84+129−94 119 11
45. (a); Let the speed of the current be x Km/h and speed 57. (c); ? = 3
+ 6
− 9
= 18
= 18
= 6 18
of the person in still water be y km/h. 65 ?
58. (d); 𝑜𝑓 240 + 𝑜𝑓 150 = 210
∴y+x=8 100 100
y – x = 6 ⇒ y = 7, x = 1 ⇒ 156 + 1.5 ×? = 210
210−156
∴ Speed of the current = 1 Km/h. ∴ ?= = 36
1.5
46. (a); Let the father’s age be x years and age of his 2 7 75
59. (a); ? = 𝑜𝑓 𝑜𝑓 𝑜𝑓 540 = 7 × 54 = 378
children be a and b years 3 5 100
𝑥
∴ (𝑎 + 𝑏) = 60. (a); ? = 555.05 + 55.50 + 5.55 + 5 + 0.55
3
And (𝑎 + 𝑏) + 20 + 20 = 𝑥 + 20 = 621.65
𝑥
⇒ 3 + 20 = 𝑥 ⇒ 𝑥 = 30 years 61. (e); ? = 1425 + 8560 + 1680 ÷ 200
1680
5100 = 1425 + 8560 +
47. (a); Simple interest for 1 year = = 𝑅𝑠 1700 200
3
= 9985 + 8.4 = 9993.4
1% of sum = 1700 800×? 750×22
∴ sum =
1700×100
= 𝑅𝑠 170000 62. (d); 100
= 293 − 100
1
⇒ 8 × ? = 293 − 165 = 128
48. (a); One principal can be appointed in 36 days ⇒ ?=
128
= 16
One vice-principal appointed in remaining 35 8

ways 25.6
63. (b);250 × 100 + √? = 119
∴ Total no. of ways = 36 × 35 = 1260.
⇒ 64 + √? = 119
49. (b);∴ Required probability ⇒ √? = 119 − 64 = 55
13 𝐶 + 13 𝐶
2 2
= 52 𝐶 ⇒ ? = 55 × 55 = 3025
2
78+78 156 2 5 5 1 11
= = = 64. (e); 4 + − 5 − =? −2 − +
1326 1326 17 6 9 3 18
Alternately, 5 5 1 11
⇒ ?= 4 − 5 + 2 + ( − + − )
Required probability 6 9 3 18
13 12 13 12 15−10+6−11
= 52 × 51 + 52 × 51 ⇒ 1+( 18
)= 1+0 = 1
13 12 2
= 2 × 52 × 51 =17 65. (c); ? = [
30
× {(
80
× 850) ÷ 34}]
100 100
50. (b);Let, P be the sum. 30
= [100 × {680 ÷ 34}]
5 10 20
∴ 16632 = 𝑃 (1 + 100) (1 + 100) (1 + 100) 30
= [100 × 20] = 6
21 11 6
Or, 16632 = 𝑃 × 20 × 10 × 5
1 1 1
Or, P = Rs.12,000 66. (d); Sides of a triangle are in ratio 2 ∶ 3 ∶ 4, i.e.,
6 : 4 : 3.
51. (b);Required difference = (40 + 80) – (12 + 58)
Let the sides be 6K, 4K and 3K, respectively.
= 120 – 70 = 50
∴ 13K = 52 ⇒ K = 4
52. (c); Required ratio=
92+98
=
190
= 19 ∶ 6. ∴ Sides of the triangle are 24 cm, 16 cm and 12
13+47 60
cm, respectively.
53. (a); Required percentage
48+12 60 67. (b);A = B + 25% of B
= 15 × 100 = 15 × 100 = 400% 𝐵 5𝐵
⇒𝐴=𝐵+4= 4
27+15+60 102
54. (e); Average= 3
= 3
= 34. 4
⇒ 𝐵 = 𝐴 = 𝐴 − 𝐴 = 𝐴 − 20% 𝑜𝑓 𝐴
1
5 5

55. (d);Windows phones sold by P on Thursday 1


3
68. (d); 12 % = 𝑅𝑠 22.50
2
= 7 × 28 =12
⇒ C.P. = Rs 180.

62 Adda247 | No. 1 APP for Banking & SSC Preparation


Website: bankersadda.com | sscadda.com | store.adda247.com | Email: contact@bankersadda.com
69. (a); Suppose x = original number of men in the group 74. (a); 999.09 + 99.90 + 9.99 + 9 + 0.99
∴ (x- 12) men did the job in 32 days = 1118.97
∴ 20x = 32(x – 12) 20 220
75. (d); × [{( × 40) − 10}] % of 500 = ?
i.e., x = 32 100 100
1
70. (c); Let, the quantity of liquid P and Q be 5x and 3x × [{88 − 10}]% of 500 = ?
5
1 78
litres respectively. × 100 × 500 = ?
5
5
Quantity of P removed = 5+3 × 16 = 10 litres ? = 78
3 76. (b);
Quantity of Q removed = 5+3 × 16 = 6 litres
5𝑥−10 3
Now, =
3𝑥−6+16 5
⇒ 25𝑥 − 50 = 9𝑥 + 30
⇒ 16𝑥 = 80 ⇒ 𝑥 = 5 77. (c);
∴ Quantity that vessel hold = 8 ×5 = 40 litres
50
71. (b);100 𝑜𝑓 250 + √? = 165
⇒ 125 + √? = 165
⇒ √? = 40 78. (e);
∴ ? = (40)2 = 1600
140 56
72. (e); 100 𝑜𝑓 56 + 100 𝑜𝑓 140
= 78.4 + 78.4 = 156.8 79. (a);
1 5 5 1 5 14 13 13
73. (e); ? = 1 4 + 1 9 × 1 8 ÷ 6 2 = 4 + 9
× 8
÷ 2
5 14 13 2
=4+ 9
× 8
× 13 80. (e);
5 7 45+14 59 23
= + = = =1
4 18 36 36 36

63 Adda247 | No. 1 APP for Banking & SSC Preparation


Website: bankersadda.com | sscadda.com | store.adda247.com | Email: contact@bankersadda.com
Most Asked Topics in IBPS RRB PO Prelims Exam
2020 - Based on Previous Year’s Exam Analysis
As the time left for IBPS RRB 2020 exam is limited, this is the perfect time to tighten your seat belt and
perform brilliantly in exam. In this space we will be discussing about the Most Asked Topics of IBPS RRB
PO- Based on previous year’s analysis.
The IBPS RRB PO Prelims Exam in the years 2018 and 2019 was held over various shifts and here we are
taking the reference of Shift-2 held on 4th August 2019 and Shift-1 held on 11th August 2018. Overall the
difficulty level of exam held in both the years was moderate.

Subject Good Attempts 2019 Good Attempts 2018


Reasoning Ability 28-33 27-32
Quantitative Aptitude 24-27 20-25
Total 56-61 51-56

Most Asked Topics of Quantitative Aptitude in IBPS RRB PO Prelims Exams


If we see the questions asked in the quantitative aptitude section the trend remains more-or less the same
where maximum weightage goes to the topic- Data Interpretation and Arithmetic Word Problems. In 2018
exam, although there were no direct questions on approximation but around 4-5 questions of arithmetic
were based on approximation.

Topics No. of Questions (2019) No. of Questions (2018)


Data Interpretation 15- Moderate 15- Moderate – Calculative
Quadratic Equation 5- Easy 5- Moderate
Number Series 5- (Missing No. Series)- Moderate 5- (Wrong No. Series)- Moderate
Arithmetic Word Problems 15- Moderate 15- Moderate – Calculative
Total 40- Moderate 40- Moderate

In the Shift-4 held on 4 August 2019, there were near about 18 questions on Data Interpretation and 11
from Arithmetic Word Problems. The ration or the weightage may differ slightly from one shift to another
but certainly these are the most important topics for IBPS RRB PO Prelims Exam.

Most Asked Topics of Reasoning Ability in IBPS RRB PO Prelims Exams


Puzzles and Seating Arrangement always dominates the Reasoning Ability section in bank exams and so is
the case with IBPS RRB PO Prelims exams as well. The level of the questions asked in the Reasoning Ability
section in 2019 and 2018 was Easy to Moderate and Moderate respectively. The following are details on
weightage of various topics in the reasoning section of previously held IBPS RRB PO Prelims exams.
64 Adda247 | No. 1 APP for Banking & SSC Preparation
Website: bankersadda.com | sscadda.com | store.adda247.com | Email: contact@bankersadda.com
Topics No. of Questions (2019) No. of Questions (2018)
Puzzles and Seating Arrangement 20- Moderate 27- Moderate-Difficult
Syllogism (based on ‘A few’ concept) 05- Easy-Moderate 5-Moderate
Direction Sense 02- Easy 3- Easy
Blood Relation 03- Easy 3- Moderate
Alphabetical Series 02- Easy-Moderate 1- Easy-Moderate
Odd One Out 01- Easy —
Order & Ranking 05- Easy-Moderate —
Miscellaneous 02- Easy-Moderate —
Numeric Series — 1- Easy-Moderate
Total 40- Easy-Moderate 40- Easy-Moderate

The following are the types of Puzzles and Seating Arrangements most frequently asked in IBPS RRB PO
Prelims Exam- Based on previous years’ exam analysis:
 Linear Uncertain Number of people
 Circular Seating Arrangement
 Linear Seating Arrangement with certain no. on people
 Parallel Rows Seating Arrangement
 Random Arrangement Puzzle
 Day Based Puzzle
 Box Based Puzzle
 Order-Ranking or Comparison based puzzle
 Month Based Puzzle
 Square Seating Arrangement

65 Adda247 | No. 1 APP for Banking & SSC Preparation


Website: bankersadda.com | sscadda.com | store.adda247.com | Email: contact@bankersadda.com
Most Asked Topics in IBPS RRB Clerk 2020 Exam-
Based On Previous Year’s Paper
It’s high time to wrap up your preparation and get to revise all the topics you have studied so far. Revising
and practicing questions from frequently asked topics may be helpful in both scoring good marks and
improve speed of solving questions in the exam hall. Read out the article and have a glimpse of
important topics from the previous year papers that could be expected in IBPS RRB Clerk 2020.

IBPS RRB Clerk 2018 & 2019


The IBPS RRB Office Assistant Exam in the years 2018 and 2019 was held in multiple shifts but here we
will be taking the reference of 18th August 2019 shift-I and Shift-4 held on 19th August 2018. We
can say that both the year exam level was easy-moderate.

Subject Good Attempts 2019 Good Attempts 2018


Reasoning Ability 37-39 33-35
Quantitative Aptitude 31-33 32-36
Total 71-76 65-71

Most Asked Topics of Quantitative Aptitude in IBPS RRB Office Assistant


Quantitative- This is one section that can help you fetch very good score as this section carries 40 marks
out of 80 and there are some topics that are always asked in this section and carry a great weightage. In
both the year we can see that Data Interpretation and Simplification carried a significant weightage
in this section.

Topics No. of Questions (2019) No. of Questions (2018)


Data Interpretation 13- Moderate 10- Moderate
5- (Missing No. Series)- 5- (Wrong No. Series)-
Number Series
Moderate Moderate
Arithmetic Word Problems 07- Moderate 10- Moderate – Calculative
Simplification/Approximation 10-Easy 15- Easy-Moderate
Total 40- Easy-Moderate 40- Easy- Moderate

If we consider the shift-I of 18th August 2019, there were questions from missing number series and were
moderate in nature. Arithmetic is another topic that carries a good weightage and we can see that
almost 7 questions of moderate level were asked. Similarly in 2018, There were again 5 questions from
wrong number series and around 10 questions from Arithmetic part.

66 Adda247 | No. 1 APP for Banking & SSC Preparation


Website: bankersadda.com | sscadda.com | store.adda247.com | Email: contact@bankersadda.com
“Most Asked Topics of Reasoning Ability in IBPS RRB Office Assistant”
Reasoning ability is another section in IBPS RRB Office Assistant Prelims exam and it is also of 40 marks.
Puzzles and Seating Arrangement is one such topic that can help you score great marks in this exam.
In both the years the level of exam can be considered as Easy-Moderate.

Topics No. of Questions (2019) No. of Questions (2018)


Puzzles and Seating Arrangement 20- Moderate 10- Moderate
Syllogism 05-Easy 5-Moderate
Inequality 05- Easy —–
Direction Sense 02- Easy 5- Moderate
Coding Decoding —– 5- Easy-Moderate
Blood Relation 03- Easy ——
Alphabetical Series 05- Easy ——
Numeric Series — 5- Easy-Moderate
Total 40- Easy-Moderate 40- Easy-Moderate
Given below are the types of Puzzles and Seating Arrangements most frequently asked in IBPS RRB PO
Prelims Exam- Based on previous years’ exam analysis:
 Box based puzzle
 Random arrangement based puzzled, salary of 6 persons were given
 Square Based Puzzle (8 people, 4 corner – outside & 4 middle – inside)
 Floor Based Puzzle (8 Floor, 8 People)
 Parallel Seating Arrangement (5 each side, North-South)
 Uncertain Number of People

Frequently Asked Questions:


Q. Can a student skip any of the topic mentioned above?
As we have provided only selected topics, so skipping any of these is not a good idea.

Q. For IBPS RRB is there any interview round after qualifying both Prelims and Mains exam?
No, the selection is done on the basis of marks in prelims and mains

Q. If a student prepares with the above-mentioned topics will


he be able to score good marks?
Yes, if without skipping any topic and clearing the basics one could
score good marks in IBPS RRB exam.

67 Adda247 | No. 1 APP for Banking & SSC Preparation


Website: bankersadda.com | sscadda.com | store.adda247.com | Email: contact@bankersadda.com
What Should Be Expected Good Attempts for IBPS
RRB PO And Clerk Prelims Exams?
IBPS RRB PO & Clerk Good Attempts- As the time is almost here for IBPS RRB PO and Clerk Preliminary
Exams 2020, students are perplexed as what should be the expected good attempts for IBPS RRB PO and
Clerk Prelims Exams and in this space we will try to cover this question in detail considering many factors.
One of the important thing to note is that number of good attempts varies directly with the level of exam
and also the average performance of the students. Normalization is another factor that plays an important
role and hence one can get an idea about the number of good attempts based on the previous year Cut-
off and Vacancies and in this we will be mentioning the same and will give you a comprehensive
analysis. Let's take a look.

IBPS RRB 2018-19: Previous Year's Vacancies


Year IBPS RRB PO IBPS RRB Clerk
2019 3352 5249
2018 3312 3688
Number of good attempts as well as cut-off varies greatly based on the number of vacancies. If the number
of vacancies are higher in one state then the good attempts will be low and vice- versa

IBPS RRB 2018-19: Previous Year's Good Attempts


Year IBPS RRB PO IBPS RRB Clerk
2019 56-62 70-74
2018 51-56 67-69
Given above is the previous year's Good attempts for IBPS RRB PO & IBPS RRB Clerk for the past two years.
This was the generalize good attempts for all the shifts though you can add +5 or -5 as per your state. IBPS
RRB PO & IBPS RRB Clerk good attempts varies state wise as well. Good attempts in 2019 were higher
than 2018 good attempts because the competition is tough and increasing and hence this year also you can
expect the good attempts to be somewhere around the above mentioned good attempts.

IBPS RRB PO: Previous Year's Cut-Off


Given below is the IBPS RRB PO previous year's cut off for your better understanding that how it plays an
important role in next year's good attempts:
State Cut-off 2018 Cut-Off 2019
Uttar Pradesh 50.00 58.75
Haryana 57.00 64.50
Maharashtra 47.50 56
Karnataka 44.25 46.25
Gujarat 48.25 43.50
Telangana 45.25 54
Bihar 45.00 58
Andhra Pradesh 52.50 65

68 Adda247 | No. 1 APP for Banking & SSC Preparation


Website: bankersadda.com | sscadda.com | store.adda247.com | Email: contact@bankersadda.com
State Cut-off 2018 Cut-Off 2019
Uttarakhand 54.00 55.75
Odisha 50.50 55.75
Himachal Pradesh 59.75 59.75
Tamil Nadu 43.25 55.25
Rajasthan 50.50 58.50
West Bengal 48.50 55.25
Punjab 54.75 63.50
Chhattisgarh 53.50 55.50
Jammu & Kashmir 47.25 55.25
As you can see that the cut-off for the 2019 year was more than the cut-off in 2018, One of the reason for
this could be the level of exam. The good attempts in 2018 was less that's may be because the level of exam
was of higher than the level of exam in 2019. Another factor could be the number of vacancies. The cut-off
is higher in those states where the vacancies have been reduced in comparison to the last year.

IBPS RRB Clerk: Previous Year Cut Off


Given below is the last two years cut-off for IBPS RRB Clerk Prelims. This will help you in better
understanding of the number of good attempts one must aim for this year's exam:
State Cut-off 2018 Cut Off 2019
Uttar Pradesh 70.75 74
Haryana 76.25 76
Madhya Pradesh 70.50 68.25
Himachal Pradesh 77.50 71
Punjab 74.75 77.50
Rajasthan 73.00 75.25
Bihar 70.25 74.25
Odisha 71.25 73.75
Gujarat 69.75 63.50
Andhra Pradesh 72.50 71.50
West Bengal 75.25 74.75
Tripura 48.75 68.25
Maharashtra 69.75 69.25
Kerala 73.50 75
Telangana 67.75 68.50
Karnataka 66.25 65.25
Assam 67.50 64.50
Jharkhand 69.75 58.50
Tamil Nadu 61.75 68

69 Adda247 | No. 1 APP for Banking & SSC Preparation


Website: bankersadda.com | sscadda.com | store.adda247.com | Email: contact@bankersadda.com
As we can see that in case of IBPS RRB Clerk as well, the cut-off has
only risen. The factors are same namely: level of exam, vacancies in
that particular state etc. It's hardly observed that that if the
vacancies were same in a state over the past two, the cut off has
still not dropped thus showing the tough competition.

IBPS RRB 2020: Expected Good Attempts


Now, that we have considered almost all the factors that plays
an important role in determining the IBPS RRB good attempts.
We can say that number of good attempts will be more this year in
comparison to the last year even if your State has more vacancies
than previous year reason being that even if the vacancies are more
in your state, the competition has only risen and it's better to be at
safe side.

We are giving you the expected good attempts for IBPS RRB 2020 Prelims Exams, you can add +5 or
-5 based on your own state:
Subject IBPS RRB PO IBPS RRB Clerk
Reasoning Ability 32-35 37-39
Quantitative Aptitude 25-30 35-37
Overall 57-65 72-76

70 Adda247 | No. 1 APP for Banking & SSC Preparation


Website: bankersadda.com | sscadda.com | store.adda247.com | Email: contact@bankersadda.com
Most Important Questions | Quantitative Aptitude |
IBPS RRB Prelims 2020
1. The respective ratio of the present ages of Anshu and 6. Rahul purchased 124 notebooks. Sunil purchased 86
her mother is 1 : 2. After 6 yrs the ratio of their ages notebooks. Manish purchased 132 notebooks and
will be 11 : 20. 9 years before, what was the Mona purchased 146 notebooks. What was the
respective ratio of their ages? average number of notebooks they purchased?
(a) 3 : 5 (b) 2 : 7 (c) 1 : 4 (a) 112 (b) 122 (c) 488
(d) 2 : 5 (e) 5 : 7 (d) 98 (e) 102
2. How many different words can be formed with the 7. The labeled price of a product is Rs 750. If it is sold
letters of the word UNIVERSITY so that all the vowels at a 20% discount and the dealer earns a 25% profit,
are together? what is the cost price?
(a) 60840 (b) 60480 (c) 60460 (a) Rs 550 (b) Rs 450 (c) Rs 435
(d) 40680 (e) None of these (d) Rs 480 (e) Rs 520
7
3. The radius of a circular wheel is metre. How many 8. A sum of money fetches Rs 240 as simple interest at
4
the rate of 5 p.c.p.a. after 6 years. What is the
revolutions will it make in travelling 22 kilometres?
principal?
(a) 1500 (b) 2500 (c) 1800
(a) Rs 200 (b) Rs 400 (c) Rs 800
(d) 2000 (e) 2400
(d) Rs 1,200 (e) Rs 1,000
4. A trader marks his goods 30% above the cost price
9. If two men or six women or four boys can finish a
and gives a discount of 15% on the marked price.
work in 99 days, then how many days will one man,
What gain% does he make?
one woman and one boy together take to finish the
(a)10.5% (b)12% (c)10%
same work?
(d)14.5% (e) 16.5%
(a) 54 days (b) 64 days (c) 44 days
5. 30 laborer’s working 7 hours a day can finish a piece (d) 104 days (e) 108 days
of work in 18 days. If the laborer’s work 6 hours a
10. The side of a square is equal to the diameter of a
day, then the number of laborers required to finish
circle. The area of the square is 441 sq. cm. What is
the same piece of work in 30 days will be
the area of the circle?
(a) 15 (b) 21 (c) 25
(a) 112 sq. cm (b) 356.8 sq. cm (c) 346.5 sq. cm
(d) 22 (e) 28
(d) 132 sq. cm (e) 264 sq. cm

Directions (11-15): Study the following graph carefully to answer the questions that follow:
Number of Girls and Boys participating in a rally from five different schools
575
550
525
500
475
450
425
400
A B C D E
Girls Boys

71 Adda247 | No. 1 APP for Banking & SSC Preparation


Website: bankersadda.com | sscadda.com | store.adda247.com | Email: contact@bankersadda.com
11. What is the total number of girls participating in the 20. 80, 122, 168, 226, 288, 362, ?
rally from schools A and C together? (a) 420 (b) 440 (c) 480
(a) 825 (b) 875 (c) 950 (d) 460 (e) 520
(d) 975 (e) 925
Directions (21-25): What will come in place of (?)
12. The number of boys participating in the rally from question mark in the following questions?
school B is what percent of the total number of
children participating in the rally from that school? 21. 12.5%𝑜𝑓 74.4 × 12 + 7 × √39.69 =?
(Rounded off to two digits after decimal.) (a) 112.4 (b) 243.6 (c) 145.3
(a) 48.84 % (b) 47.37 % (c) 49.28 % (d) 155.7 (e) 132.2
(d) 46.46 % (e) None of these 4 1 5
22. (27)3 × (729)3 ÷ (9)2 =?
13. The number of girls participating in the rally from (a) 4 (b) 8 (c) 27
School E is approximately what percent of the
(d) 9 (e) 3
number of boys participating in the rally from the
same school? 1 3
1 3 343
(a) 81 % (b) 106 % (c)122 % 23. (343)3 ÷ (49)2 × 63 ÷ 3 ×√ =?
√729 729
(d) 98 % (e) 114 % (a) 49 (b) 81 (c) 343
14. What is the respective ratio of total number of girls (d) 63 (e) 9
participating in the rally from schools D and C 24. 60% 𝑜𝑓 950 + 35% 𝑜𝑓 880 − 47% 𝑜𝑓 500 =?
together to the total number of boys participating in (a) 643 (b) 523 (c) 713
the rally from schools A and B together?
(d) 673 (e) 663
(a) 23 : 18 (b) 43 : 35 (c) 41 : 38
(d) 21 : 20 (e) 39 : 40 2 1
25. 14 7 % 𝑜𝑓 651 + 6 4 % 𝑜𝑓 1008 − 5 17 % 𝑜𝑓 1156 =?
15

15. What is the average number of girls participating in (a) 86 (b) 88 (c) 49
the rally from all schools together? (d) 72 (e) 98
(a) 500 (b) 460 (c) 525
(d) 505 (e) 650 Directions (26-30): In each of these questions, two
equations (I) and (II) are given. You have to solve both
Directions (16 - 20): What will come in place of the equations and give answer
question mark (?) in the following number series? (a) if x>y
16. 1, 2, 6, 33, 49, 174, ? (b) if x≥y
(a) 255 (b) 284 (c) 210 (c) if x<y
(d) 251 (e) 198 (d) if x ≤y
17. 1728, 1740, 1764, 1800, 1848, 1908, ? (e) if x = y or no relation can be established between x
(a) 1980 (b) 1988 (c) 2008 and y.
(d) 1976 (e) 1955 26. I. x2 – 36 = 0 II. y2 +13y +42 = 0
18. 4, 4, 9, 29, 119, 599,? 27. I. 6x2 +19 x +15 = 0 II. 2y2 +11y + 15 = 0
(a) 1242 (b) 1642 (c) 1824
(d) 3599 (e) 4023 28. I. 6x2 – x -12 = 0 II. 20y2 + 9y -18 = 0
14 3
19. 49, 47, 53, 41, 61, 31, ? 29. I. 2𝑥 + 3𝑦 = II. 3𝑥 − 2𝑦 =
5 10
(a) 75 (b) 73 (c) 71
(d) 79 (e) 81 30. I. 𝑥 2 − 4√2𝑥 + 6 = 0 II. 𝑦 2 − 3√2𝑦 + 4 = 0
Directions (31– 35): The following table shows the no. of calculators manufactured(M) and defective(D) among 4
different companies over the years.
Year Company A Company B Company C Company D
M D M D M D M D
2014 63 11 55 07 86 28 66 11
2015 59 08 42 10 55 14 73 14
2016 60 08 58 8 65 6 78 20
2017 75 10 78 5 67 10 64 9
2018 80 21 82 3 92 12 58 17
2019 54 05 66 12 48 22 50 5
Note : (i) Non defective calculators = total manufactured − defectives
(ii) All the values given are in thousands (000)
72 Adda247 | No. 1 APP for Banking & SSC Preparation
Website: bankersadda.com | sscadda.com | store.adda247.com | Email: contact@bankersadda.com
31. Find the average no. of non-defective calculators (a) 5 : 8 (b) 8 : 13 (c) 8 : 9
manufactured by company A over the years. (Round (d) 8 : 11 (e) 7 : 11
off to nearest integer)
(a)65665 (b) 52997 (c) 51333 34. In which year, total no. of defective calculators
(d) 54667 (e) 58334 produced by all companies together are maximum?
(a)2014 (b) 2015 (c) 2016
32. Total calculators manufactured by company A in
2016 and 2018 together are what percent more than (d) 2017 (e) 2018
calculators manufactured by company C in 2015 and 35. Defective calculators produced by which company in
2016 together. 2015 is 80% of the defective calculators produced by
(a)50% (b) 20% (c) 40% company B in 2015?
2
(d) 16 % (e) 25% (a)Company A
3

33. What is the ratio between total calculators (b) Company C


manufactured by company B in 2015 and 2017 (c) Company D
together to calculators manufactured by company D (d) Can’t be determined
in 2015, 2017 and 2018 together? (e) None of these
Directions (36-40): Date given below shows number of girls in six different schools. Some data is given in absolute value
while some in percentage. Study the data carefully and answer the following questions

F
9% A
E 20%
462

B
594
D
24%
C
15%

36. Find the central angle of girls in school B? (a) 1364 (b) 1386 (c) 1408
(a) 57.6° (b) 64.8° (c) 72° (d) 1430 (e) 1452
(d) 79.2° (e) 86.4° 39. If ratio between number of girls and number of boys
in school ‘F’ is 9 : 8, then find total number of
37. Total number of girls in school ‘D’ is how much more students in school ‘F’.
than total number of girls in school ‘E’? (a) 561 (b) 550 (c) 528
(a) 264 (b) 297 (c) 330 (d) 539 (e) 572
(d) 363 (e) 396 40. Total number of girls in school ‘C’ is what percent
less than total number of girls in school ‘A’?
38. Find the total number of girls in school ‘A’ and ‘D’ 1
(a) 33 % (b) 25%
2
(c) 66 %
3 3
together? (d) 75% (e) 50%

Solutions
1. (d);Let present age of Anshu and her mother be 𝑥 & on solving, 𝑥 = 27 and 𝑦 = 54
𝑦 years respectively. ∴ Required ratio =
18
=
2
𝑥 1 45 5
∴𝑦=2
4!×7!
and
𝑥+6
=
11 2. (b);Required number of words = = 60480
𝑦+6 20 2

73 Adda247 | No. 1 APP for Banking & SSC Preparation


Website: bankersadda.com | sscadda.com | store.adda247.com | Email: contact@bankersadda.com
3. (d);Number of revolutions 17. (a); 1728+12=1740
22000
= 22 7 = 2000 revolutions. 1740+24=1764
2× ×
7 4 1764+36=1800
4. (a); Let C.P. be Rs. 100 1800+48=1848
∴ S.P. =
85
× 130 = Rs. 110.5 1848+60=1908
100
10.5
So, 1908+72=1980
∴ Profit% = × 100 = 10.5%
100 18. (d);4 × 1 + 0 = 4
30×7×18 4×2+1 =9
5. (b);Required number of laborer’s = 6×30
= 21
9 × 3 + 2 = 29
6. (b);Average number of notebooks 29 × 4 + 3 = 119
124+86+132+146 488
= = = 122 119 × 5 + 4 = 599
4 4
599 × 6 + 5 = 3599
7. (d);SP of the product
80 19. (b);49-(1× 2)=47
= Rs (100 × 750) = Rs 600 47+(2× 𝟑)=53
Profit = 25% 53-(3×4)=41
100
∴ CP = 125 × 600 = Rs 480 41+(4×5)=61
61-(5×6)=31
SI×100
8. (c); Principal = So, 31+(6×7)=73
Time×Rate
240×100

5×6
= Rs 800 20. (b);92 − 1 = 80
112 + 1 = 121
9. (e); Tricky Approach
132 − 1 = 168
2 men = 6 women = 4 boys
∴ 1 man = 3 women = 2 boys 152 + 1 = 226
∴ 1 man + 1 woman + 1 boy 172 − 1 = 288
2 11 192 + 1 = 362
= (2 + 3 + 1) boys = 3 boys
So, 212 − 1 = 440
11
∴ M1 D1 = M2 D2 ⇒ 4 × 99 = × D2
4×3×99
3 21. (d);12.5%of 74.4 × 12 + 7 × √39.69 =?
⇒ D2 = = 108 days 1
11 = × 74.4 × 12 + 7 × 6.3 =?
8
10. (c); Side of a square = √Area = √441 = 21 =111.6+44.1 = ?
21
Diameter of circle = 21 cm ⇒ Radius = 2 cm =155.7 = ?
22 21 21 4 1 5
∴ Area of circle = πr² = × × 22. (e); =(27)3 × (729)3 ÷ (9)2 =?
7 2 2
= 346.5 sq. cm =81 × 9 ÷ 243 =?
=3 = ?
11. (e); Required number = 450 + 475 =925
1 3
525 1 3 343
12. (a); Required percentage = 525+550 × 100 = 48.84% 23. (e); (343)3 ÷ (49)2 × 63 ÷ 3 × √729 =?
√729
1 7
13. (c); Required percentage =7 ÷ 343 × 63 ÷ 9 × 9 =? =9 = ?
550
= 450 × 100 = 122%(approx) 24. (a); 60% of 950 + 35% of 880 − 47% of 500 =?
60 35 47
14. (e); Total number of girls participating in the rally = × 950 + × 880 − × 500 =?
100 100 100
from schools D and C together = 475 + 500 =975 =570+308-235 = ?
Total number of boys participating in the rally =643 =?
from schools A and B together = 475 + 525 2 1 15
= 1000 25. (b);14 7 % of 651 + 6 4 % of 1008 − 5 17 % of 1156
Required ratio= 975: 1000 = 39: 40 1 25
=7 × 651 + 400 × 1008 − 1700 × 1156 =88
100

450+550+475+500+550
15. (d);Required average = = 505
5 26. (b);I. x² - 36 = 0
16. (c); 1+𝟏 = 𝟐
𝟑 (x + 6) (x - 6) = 0 ⇒ x = 6, -6
2+𝟐𝟐 = 𝟔 II. y2 +13y +42 = 0
6+𝟑𝟑 = 𝟑𝟑 y² +7y +6y +42 =0
33+𝟒𝟐 = 𝟒𝟗 y(y +7) + 6 (y +7) = 0
49+𝟓𝟑 = 𝟏𝟕𝟒 (y +6) (y + 7) = 0 ⇒ y = -6, -7
So, 174+𝟔𝟐 = 𝟐𝟏𝟎 ∴𝐱≥𝐲

74 Adda247 | No. 1 APP for Banking & SSC Preparation


Website: bankersadda.com | sscadda.com | store.adda247.com | Email: contact@bankersadda.com
27. (a); I. 6x2 +19 x +15 = 0 II. y 2 − 3√2y+4=0
6x² + 10x +9x +15 = 0 y 2 − 2√2y − √2y + 4 = 0
2x(3x + 5) + 3 (3x + 5) = 0
(y-√2)(y-2√2)=0
(2x + 3) (3x + 5) = 0
𝟑 𝟓 y=√2 , 2√2
⇒ x =−𝟐 ,−𝟑
So, no relation between x and y.
II. 2y2 +11y + 15 = 0
52+51+52+65+59+49
2y² +6y + 5y + 15 = 0 31. (d);Required average = ≈ 54667
6
2y (y + 3) + 5 (y + 3) = 0 140−120 2
(y + 3) (2y + 5) = 0 32. (d);Required percentage = × 100 = 16 %
120 3
𝟓
⇒ y = −𝟑, − 42+78
𝟐
33. (b);Required ratio = 73+64+58 = 8 ∶ 13
𝐬𝐨, 𝐱 > 𝐲
28. (e); I. 6x2 – x -12 = 0 34. (a); Maximum defective calculators were produced
6x² - 9x + 8x -12 = 0 in 2014 i.e. 57000
3x (2x – 3) +4 (2x – 3) = 0 8
35. (a); % for company A = × 100 = 80%
(3x + 4) (2x – 3) = 0 10
𝟒 𝟑
⇒ 𝐱 = −𝟑,𝟐 36. (b);Required central angle =
594
× 360 = 64.8°
3300
II. 20y2 + 9y -18 = 0
24
20y² + 24y - 15y -18 = 0 37. (c); Total number of girls in school D = × 3300
100
4y (5y + 6) - 3 (5y + 6) = 0 = 792
(4y - 3) (5y + 6) = 0
3 6 Required difference = 792 – 462 = 330
⇒ y = ,−
4 5
∴ no relation 38. (e); Total number of girls in school A & D together
(20+24)
= × 3300
29. (c); On solving both equation 100
We get, = 44 × 33 = 1452
1 3
x = , and y =
2 5
39. (a); Total number of students in school F
9 17
So, y > x = × × 3300
9 100
30. (e); I. x − 4√2x + 6 = 0
2 = 561
x 2 − 3√2x − √2x + 6 = 0 40. (b);Required % =
20−15
× 100
(x − √2)(x − 3√2)=0 5
20

x=√2 ,3√2 = 20 × 100 = 25%

75 Adda247 | No. 1 APP for Banking & SSC Preparation


Website: bankersadda.com | sscadda.com | store.adda247.com | Email: contact@bankersadda.com
Most Important Questions | Reasoning Ability | IBPS
RRB Prelims 2020
Direction (1-5): Study the following information 8. If all the numbers are arranged in descending order,
carefully and answer the questions given below- then what will be the difference between the second
Eight persons are sitting around a circular table. Some of digit of highest number and third digit of second
them are facing inside the center while some are facing highest number?
outside the center. V sits third to the left of W. S sits (a) 1 (b) 2 (c) 4
second to the left of P. T sits third to the right of S. Both T (d) 5 (e) 0
and Q are facing same direction as V. U sits opposite to Q 9. If ‘1’ is added to all even digits in all the numbers, in
and Q is not an immediate neighbor of S who is facing which of the following numbers will the difference
opposite direction of P. V is not an immediate neighbour between the first and second digit will be less than
of U. R neither sits opposite to T nor P. Both V and W are two?
facing same direction. W is facing inside the center of the (a) 625 (b) 843 (c) 236
circle. While both R and U are facing same direction to
(d) 758 (e) 417
each other but opposite to S.
10. If in each number all the digits are arranged in
1. Who among the following sits immediate right of S?
ascending order within the number, then what will
(a) R (b) V (c) U
be the difference between lowest and highest
(d) P (e) Q
number thus formed?
2. How many persons sits between Q and U when (a) 431 (b) 469 (c) 530
counted in clockwise direction with respect to U? (d) 433 (e) 429
(a) One (b) Three (c) Two
(d) Four (e) No one Directions (11-15): Study the following information
carefully and answer the questions given below:
3. Who among the following faces V?
(a) P (b) R (c) S In a certain code language
(d) T (e) W ‘election survey people’ is written as ‘yo vo na’,
‘biopic law member’ is written as ‘sa ra ta’,
4. Who among the following sit opposite to R?
‘people law lead’ is written as ‘la vo sa’, and
(a) W (b) T (c) S
‘review nation election’ is written as ‘yo ha ja’.
(d) P (e) None of these
11. What is the code for ‘election’?
5. Who among the following sit opposite to the one
(a) ja (b) ha (c) yo
who sits 2nd right of R?
(a) V (b) P (c) Q (d) na (e) None of these
(d) S (e) None of these 12. ‘review nation biopic’ can be written as
Directions (6-10): These questions are based on the (a) ja ha ta
following set of numbers. (b) ta ra ha
(c) ha ja ra
758 625 417 843 236 (d) Either (a) or (c)
6. If in each number first digit is replaced by the second (e) None of these
digit, second digit is replaced by the third digit and 13. What is the code for ‘review’?
third digit is replaced by the first digit, then which
(a) ja (b) yo (c) la
number will be the second lowest?
(d) ha (e) Can’t be determined
(a) 625 (b) 843 (c) 236
(d) 758 (e) 417 14. What does ‘la’ stand for?
7. If in each number, the first and the third digits are (a) law (b) lead (c) survey
interchanged, and then which number will be the (d) nation (e) None of these
highest? 15. What is the code for ‘law’?
(a) 843 (b) 417 (c) 236 (a) ja (b) yo (c) sa
(d) 758 (e) 625 (d) ha (e)None of these

76 Adda247 | No. 1 APP for Banking & SSC Preparation


Website: bankersadda.com | sscadda.com | store.adda247.com | Email: contact@bankersadda.com
Directions (16-20): In each of the questions below are not necessarily in the same order) and all of them are
given some statements followed by some Conclusions. facing north. Each member seated in one row faces
You have to take the given statements to be true even, if another member of the other row.
they seem to be at variance from commonly known facts. E sits 2nd to the right of C and doesn’t faces J. A sit at one
Read all the conclusions and then decide which of the of the ends and diagonally opposite to B. Three persons
given conclusions logically follows from the given sit between B and F, who does not face G. Neither E nor C
statements disregarding commonly known facts. is an immediate neighbor of A. J sits immediate left to D
(a) If only conclusion I follows. but does not faces C. Two persons sit between I and G,
(b) If only conclusion II follows. none of them sits at the end. The one who faces K sits 2nd
(c) If either conclusion I or II follows. right to L.
(d) If neither conclusion I nor II follows.
21. Who among the following faces C?
(e) If both conclusions I and II follow.
(a) G (b) K (c) I
16. Statements: (d) A (e) none of these
All careers are Subject
Some technology is Program 22. How many persons sit to the right of H?
Only a few program are subject (a) No One (b) one (c) two
Conclusions: (d) three (e) four
I: All subject can be program
II: All Careers are technology is possibility 23. Four of the following five form a group, who among
the following does not belongs to that group?
17. Statements: (a) B (b) K (c) C
All syllabus are worst (d) A (e) L
Some worst are not difficult
All difficult are Power 24. If in a certain way A is related to J, K is related to H,
Conclusions: then who among the following is related to D?
I. All difficult are syllabus is a possibility (a) G (b) K (c) H
II. Some power are syllabus (d) F (e) I
18. Statements: 25. Who among the following sit 3rd right to K?
Some Prime are Ultimate
(a) A (b) K (c) E
All ultimate are Package
No prime is Boring (d) C (e) No one
Conclusions: Directions (26-30): In these questions, relationship
I: Some Ultimate is not Boring between different elements is shown in the statements.
II: All Package can never be Boring The statements are followed by conclusions. Study the
19. Statements: conclusions based on the given statements and select the
All profile are Session appropriate answer. Give answer-
Some session are result (a) If only conclusion II is true
Only Session are subject (b) If only conclusion I is true
Conclusions: (c) If both conclusions I and II are true.
I. Some subject are result is possibility (d) If either conclusion I or II is true
II. Some profile is subject is possibility
(e) If neither conclusion I nor II is true.
20. Statements:
All bench are table 26. Statements: A>G<J, I≤D, J<O=K>D.
Only few table are chair Conclusions: I. G>D II. O>I
All chair is wood 27. Statements: P≥W≤Z, W>K, Q>O<Z
Conclusions:
Conclusions: I. W>Q II. Z>K
I. Some wood can be bench
II. All bench can be chair 28. Statements: B≥L<J>O, E>Y<L
Directions (21-25): Read the following information Conclusions: I. B>Y II. J>Y
carefully and answer the questions given below: 29. Statements: T>O<A<Q≥M>N<V
Twelve people are sitting in two parallel rows containing Conclusions: I. T>N II. A>M
six people. In row 1 – A, I, G, K, E and C are seated (but
30. Statements: B>Q≥R, Z>T≥R>J
not necessarily in the same order) and all of them are
facing south. In row 2 – L, H, D, J, B and F are seated (but Conclusions: I. B > J II. Z > Q

77 Adda247 | No. 1 APP for Banking & SSC Preparation


Website: bankersadda.com | sscadda.com | store.adda247.com | Email: contact@bankersadda.com
Directions (31-33): Study the information carefully and 35. How many male members are there in the family?
answer the questions given below. (a) Four
(b) Three
Reena faces west and start his journey from point O and (c) Can’t determined
walks 4 km to reach P, then turn right walks 3km to reach (d) Five
point D. After this, she turns right and walks 5 km. Now, (e) None of these
she turns again towards right and walks 6km, then turn
Directions (36-40): Study the information carefully and
right and walks 1km. answer the questions given below.
31. What is the shortest distance between Point O and Seven persons M, N, O, W, X, Y and Z going to three
Final position and in which direction she is with different classes Acting, Singing and Crafting on different
respect to O? days of the same week with at least two of them in each
(a) 1km, South of these classes. Each of them likes different food items–
(b) 2km, North east Rice, Chocolate, Milk, Candy, Orange, Oats and Banana. X
(c) 3km, South going to Singing class with M. M likes Banana. Those who
(d) 2km, South west went to Acting class do not like Rice and Candy. The one
who likes Chocolate is going only with O in Crafting class.
(e) None of these
The one who likes orange does not go in the same class
32. What is the shortest distance between Point O and with M and O. W does not go to Singing class. W likes
D? Oats. Z and N going in the same class. N does not like Milk.
(a) 5km (b) 4km (c) 3km The one who likes Candy does not go in Singing class.
(d) 6km (e) None of these 36. Which of the following food X likes?
(a) Oats (b) Rice (c) Milk
33. If point M is 4m west of point D, then what is the
(d) Orange (e) None of these
direction of point M with respect to point P?
(a) North (b) South-East (c) North-West 37. Which of the following Class W go?
(d) West (e) None of these (a) Singing
(b) Crafting
Directions (34-35): Read the following information (c) Acting
carefully and answer the given questions. (d) Either (a) or (b)
(e) None of these
There are eight people namely M, N, Q, T, W, X, Y and Z in
a family, all of them are going to attend a marriage party. 38. Which of the following food N likes?
(a) Orange (b) Oats (c) Candy
There are two married couple in the family. Q is the
(d) Banana (e) None of these
mother of N. W is the daughter of M, who is the sister of
X. Q is the only sister of Y. Z is father of Q. T is the 39. Which of the following combination is true?
maternal grandmother of N. M is the wife of Y. (a) N- Orange-Crafting
(b) W-Rice- Crafting
34. How is N related to Z? (c) X-Rice- Acting
(a) Son (d) Y-Milk- Acting
(b) Grand daughter (e) None is true
(c) Grand son 40. Which of the following food Z likes?
(d) Daughter (a) Orange (b) Banana (c) Milk
(e) Can’t determined (d) Candy (e) None of these

Solutions
Solutions (1-5): 1. (c); 2. (b); 3. (d)
4. (c); 5. (b);
6. (a); New arrangement will be 587 256 174 438 362.
7. (d); 8. (c); 9. (c)
10. (a);

78 Adda247 | No. 1 APP for Banking & SSC Preparation


Website: bankersadda.com | sscadda.com | store.adda247.com | Email: contact@bankersadda.com
Solution (11-15): 24. (e); 25. (e);
WORD CODE 26. (a); I. G>D (False) II. O>I (True)
Election yo 27. (a); I. W>Q(False) II. Z>K (True)
Survey na
People vo 28. (c); I. B>Y(True) II. J>Y (True)
Biopic/member ra/ta 29. (e); I. T>N (False) II. A>M (False)
Law sa 30. (b); I. B > J (True) II. Z > Q (False)
Lead La
Review/nation ha/ja Solutions (31-33):

11. (c); 12. (d); 13. (e)


14. (b); 15. (c);
16. (e);

31. (c);
32. (a); OD=√32 + 42 =5km
17. (a);
33. (c);
34. (e);

18. (e);

Since, the gender of N is not defined therefore, it


can’t be determined.
35. (c);

19. (d);

Since, the gender of X and N is not defined


20. (e); therefore, it can’t be determined.
Solutions (36-40):
Person Food item Class
M Banana Singing
N Orange Acting
Solutions (21-25): O Candy Crafting
W Oats Acting
X Rice Singing
Y Chocolate Crafting
Z Milk Acting

36. (b); 37. (c); 38. (a);


21. (e); 22. (b); 23. (b) 39. (e); 40. (c);

79 Adda247 | No. 1 APP for Banking & SSC Preparation


Website: bankersadda.com | sscadda.com | store.adda247.com | Email: contact@bankersadda.com

You might also like